120
Chapter 1 – Introduction 1 of 120 Wills & Trusts Chapter 1 – Introduction 1. The Power to Transmit Property at Death: Justifications and Limitations a. The Right to Inherit and the Right to Convey i. Blackstone – Commentaries (CB p. 1) 1. Wills & testaments are creatures of the civil or municipal laws – gov’t should be able to regulate wills & trusts ii. Locke – Two Treatises of Govt (CB p. 2) 1. Wills & testaments are a personal right, to be governed by people, not governments iii. Hodel v. Irving (p. 3) iv. F/P 1. In 1889, pursuant to a series of land acts enacted by Congress, which divided communal reservations of Indian tribes into individual allotments for Indians and unallotted lands for non-Indian settlement, each male Sioux head of household took 320 acres of land and other individuals took 160 acres. The lands were held in trust by the United States in order to protect the allottees from improvident disposition of their lands to white settlers. 2. The allotment program quickly failed because the Indians leased their allotted lands to white ranchers and farmers, which resulted in parcels being splintered into multiple undivided interests that could not be alienated or partitioned, due to the fact the land was held in trust. 3. To address this problem, Congress enacted the Indian Land Consolidation Act of 1983, which contained an escheat provision. The escheat provision essentially provided that any undivided fractional interest in a tract within a tribe’s reservation or jurisdiction, would escheat (escheat = reversion of property to the state in the absence of legal heirs or claimants) to that tribe and could not be passed by intestacy or devise, if the interest represented two percent or less of the total acreage of the tract and it earned its owner less than $100 in the preceding year before it was due to escheat. 4. The statute became law on January 12, 1983 and it contained no provision for the payment of compensation to the owners of interests covered by the escheat provision. 5. Mary Irving, Patrick Pumpkin Seed, and Eileen Bisonette, appellees, are or represent heirs or devisees of members of the Oglala Sioux Tribe who died in March, April, and June of 1983. But for the escheat provision of the Act, $2,700 which represents 26 escheatable interests in the Cross estate and $1,816 which represents 13 escheatable interests in the Pumpkin Seed estate would have passed, in ordinary course, to appellees or those they represent. 6. Appellees filed suit in the District Court alleging the escheat provision resulted in a taking without just compensation under the Fifth Amendment. 7. The District Court held the statute was Constitutional. 8. The Court of Appeals reversed concluding that while appellees’ had no vested rights in decedent’s property, their decedents had a right to control the disposition of their property at death. The Court held that appellees had standing to invoke that right and that the taking of that right without just compensation to decedents; estates violated the Fifth Amendment. v. I 1. Did the original version of the escheat provision of the Indian Land Consolidation act of 1983 constitute a taking of appellees’ decedents’ property without just compensation? vi. R/A/H 1. HELD: Yes – the escheat provision was an unconstitutional “taking” of decedent’s property w/o just compensation

Wills and Trusts Semester Outline

Embed Size (px)

DESCRIPTION

Semester Outline for Wills & Trusts class

Citation preview

Chapter 1 – Introduction 1 of 120

Wills&Trusts

Chapter 1 – Introduction 1. The Power to Transmit Property at Death: Justifications and Limitations

a. The Right to Inherit and the Right to Convey

i. Blackstone – Commentaries (CB p. 1)

1. Wills & testaments are creatures of the civil or municipal laws – gov’t should be able to

regulate wills & trusts

ii. Locke – Two Treatises of Govt (CB p. 2)

1. Wills & testaments are a personal right, to be governed by people, not governments

iii. Hodel v. Irving (p. 3)

iv. F/P

1. In 1889, pursuant to a series of land acts enacted by Congress, which divided communal

reservations of Indian tribes into individual allotments for Indians and unallotted lands

for non-Indian settlement, each male Sioux head of household took 320 acres of land

and other individuals took 160 acres. The lands were held in trust by the United States

in order to protect the allottees from improvident disposition of their lands to white

settlers.

2. The allotment program quickly failed because the Indians leased their allotted lands to

white ranchers and farmers, which resulted in parcels being splintered into multiple

undivided interests that could not be alienated or partitioned, due to the fact the land

was held in trust.

3. To address this problem, Congress enacted the Indian Land Consolidation Act of 1983,

which contained an escheat provision. The escheat provision essentially provided that

any undivided fractional interest in a tract within a tribe’s reservation or jurisdiction,

would escheat (escheat = reversion of property to the state in the absence of legal heirs

or claimants) to that tribe and could not be passed by intestacy or devise, if the interest

represented two percent or less of the total acreage of the tract and it earned its owner

less than $100 in the preceding year before it was due to escheat.

4. The statute became law on January 12, 1983 and it contained no provision for the

payment of compensation to the owners of interests covered by the escheat provision.

5. Mary Irving, Patrick Pumpkin Seed, and Eileen Bisonette, appellees, are or represent

heirs or devisees of members of the Oglala Sioux Tribe who died in March, April, and

June of 1983. But for the escheat provision of the Act, $2,700 which represents 26

escheatable interests in the Cross estate and $1,816 which represents 13 escheatable

interests in the Pumpkin Seed estate would have passed, in ordinary course, to

appellees or those they represent.

6. Appellees filed suit in the District Court alleging the escheat provision resulted in a

taking without just compensation under the Fifth Amendment.

7. The District Court held the statute was Constitutional.

8. The Court of Appeals reversed concluding that while appellees’ had no vested rights in

decedent’s property, their decedents had a right to control the disposition of their

property at death. The Court held that appellees had standing to invoke that right and

that the taking of that right without just compensation to decedents; estates violated

the Fifth Amendment.

v. I

1. Did the original version of the escheat provision of the Indian Land Consolidation act of

1983 constitute a taking of appellees’ decedents’ property without just compensation?

vi. R/A/H

1. HELD: Yes – the escheat provision was an unconstitutional “taking” of decedent’s

property w/o just compensation

Chapter 1 – Introduction 2 of 120

2. RULE: The States’, and where appropriate, the United States, has broad authority to

adjust the rules governing the descent and devise of property without implicating the

guarantees of the Just Compensation Clause.

3. Since the escheatable rights are not de minimis, nor does the availability of inter vivos

transfer obviate the need for descent and devise, a total abrogation of these rights

cannot be upheld. The regulation virtually amounts to the abrogation of the right to

pass on a certain type of property-a small undivided interest-to one’s heirs.

vii. Notes: Right to Transfer vs Right to Receive

1. Constitutional protections seem to apply only to the decedent’s power to transfer, not

the heir/beneficiary’s right to receive

b. The Policy of Passing Wealth at Death

i. Halbach – An Introduction to Death, Taxes, and Family Property (CB p. 16)

1. Pros: In support of Inheritance

a. In a society based on private property, inheritance is not objectionable (person

should have power to decide how to transfer property; rewards life of hard

work)

b. Inheritance is a natural & proper way to express & reinforce family ties, which

are important to a healthy society & good life

c. Incentive to bring forth creativity, hard work, initiative, and productivity that

benefits others

d. Encourages ppl to save for old age and give to family

e. Encourages families to love, serve, and protect their elders

ii. Bentham – The Theory of Legislation (CB p. 17)

1. ???

iii. Oliver, Shapiro, and Press – Them That’s Got Shall Get: Inheritance and Achievement in Wealth

Accumulation (p. 18)

1. Inheritance is an unearned benefit that produces unequal opportunities

2. E.g. The richest 1% of baby boomers (btwn 1987 and 2011) will get 1/3 of the worth of

estates; The next richest 9% will take the next 1/3.

iv. Ascher – Curtailing Inherited Wealth (CB p. 20)

1. Govt should tax inherited wealth; i.e. federal wealth transfer taxes should curtail

inheritance, and increase equal opportunity, while also raising revenue.

2. This attacks inheritance by healthy, adult descendants – they didn’t earn wealth; “luck”

shouldn’t dictate – inheritance should be allowed only where public policy clearly

justifies it.

3. Ascher’s proposal: All property owned at death, after payment of debts and

administration expenses, should be sold, and the proceeds paid to the US govt.

4. Six Exceptions to Ascher’s Proposal

a. Marital exemption – spouses could continue to provide for each other after

death; amount should depend on the length of the marriage.

b. Dependent Lineal Descendants – Decedents would be allowed to provide for

dependent lineal descendants (the amount allowed would decrease w/ the

descendant’s age)

c. Disabled lineal descendants – Decedents could give generous amounts to

disabled descendants (independent of age?)

d. Lineal Ascendants – Decedent can give unlimited inheritance to lineal

ascendants (e.g. parents, grandparents, etc)

e. Universal exemption – Allows a moderate amount of property either 1) to pass

outside the exemptions or 2) to augment amounts passing under them – i.e.

decedent can leave something to persons of his/her choice, regardless of

whether another exemption was available

f. Charity – Up to a fixed fraction of an estate could go to charity

v. Kristol – Taxes, Poverty, and Equality (CB p. 22)

Chapter 1 – Introduction 3 of 120

1. Problem is: large inherited concentrations of wealth can lead to an aristocracy (the

wealthy in power; undermines democracy)

2. Kristol’s Proposition: Legislate that no large fortune should outlast the lifetime of the

man who made it, but rather that such a large fortune should dissolve into much smaller

fortunes upon his death

a. Make a rule/policy: No individual can inherit more than $1 Million, and any

possessor of a large fortune must distribute it to his children, his relatives, his

friends, anyone, but no one can get more than the maximum legacy (tax-free).

b. Institutional donations would be unlimited

vi. Blum and Kalven – The Uneasy Case for Progressive Taxation (CB p. 24)

1. A great source of inequality of opportunity in our society is “cultural inheritance.”

2. ?

vii. Langbein – The Twentieth-Century Revolution in Family Wealth Transmission (p. 26)

1. Wills/trusts is a dying field

2. The main means of transferring wealth is by paying for education (especially since

skills/knowledge are more meaningful today than e.g. land).

c. The Problem of the Dead Hand

i. “Dead hand control” = Decedent conditions a gift to a beneficiary upon a beneficiary behaving in

a certain way (a.k.a. incentive trust)

ii. R.3d Property (Wills & Other Donative Transfers) § 10.1 – Donor’s Intention Determines the

Meaning of a Donative Document and is Given Effect to the Maximum Extent Allowed by Law

1. The controlling consideration in determining the meaning of a donative document is the

donor’s intention.

2. The donor’s intention is given effect to the maximum extent allowed by law.

3. Note: This favors the decedent’s freedom of disposition

4. Note: (p. 33) – Posner believes that courts should be able to modify the conditions of

wills

iii. Valid Conditions on Gifts

1. Testamentary conditional gifts are valid UNLESS

a. They violate public policy, OR

b. Judicial enforcement would constitute a state action violating constitutionally

protected fundamental rights

iv. Invalid Conditions on GIfts

1. Absolute restraints on marriage

a. Gifts conditioned on the beneficiary not marrying anyone (at least wrt. first

marriages) generally violate the fundamental right to marry, and are VOID.

b. Exception—Partial Restraints: Partial restraints on marriage that impose only

“reasonable restrictions” are NOT contrary to public policy, and are VALID

c. Exception—Temporal/Religion Requirement: Gifts requiring a beneficiary to

marry within a “reasonable amount of time,” even to someone of a particular

religious background, HAVE BEEN HELD VALID

i. Shapira v. Union National Bank (p. 28)

ii. F/P

1. David Shapira, M.D., testator, conditioned his son’s (Daniel

Jacob Shapira, Plaintiff) inheritance under his will upon

Plaintiff being married to, or marrying within seven years of

testator’s death, a Jewish girl with two Jewish parents.

2. Plaintiff filed suit alleging that such a condition was

unconstitutional based upon the premise that the right to

marry is protected by the Fourteenth Amendment to the

Constitution of the United States.

iii. I

Chapter 1 – Introduction 4 of 120

1. Is a condition upon inheritance, which is based on marriage,

is unconstitutional, contrary to public policy, and

unenforceable because of its unreasonableness?

iv. R/A/H

1. No –upholding and enforcing the provision of Dr. Shapira’s

will conditioning the bequests to his sons upon their marrying

Jewish girls does not offend the Constitutions of Ohio or of

the United States.

2. The conditions contained in decedent’s will are reasonable

restrictions. His unmistakable testamentary plan was for his

possessions to be used to encourage the preservation of the

Jewish faith.

3. The condition did not pressure plaintiff into marriage by the

reward of money because the seven year time limit is a

reasonable grace period, which would give plaintiff ample

time for reflection and fulfillment of the condition without

constraint or oppression

2. Religion Requirement

a. Gifts that require a beneficiary to be of a particular religion are generally held

to violate public policy concerning religious freedom, and are INVALID

3. Encouraging separation and/or divorce

a. Gifts that require a beneficiary to separate or divorce before receiving the gift

generally are deemed against public policy and VOID

b. BUT, gifts that provide for a beneficiary only in the event of separation and/or

divorce are not necessarily deemed to encourage divorce.

c. The controlling factor: The decedent’s dominant intent – “encouraging”

separation/divorce vs. “merely providing support in the event of”

separation/divorce

4. Promoting family strife

a. Gifts conditioned upon family members ostracizing and/or not communicating

w/ other fam members generally have been held to violate public policy and

are VOID

5. Property Destruction Directives

a. Generally, testators are not free to direct people to destroy property upon the

testators’ deaths; such directives are generally INVALID

i. Note: Though people may generally destroy property when they are

alive, they can’t generally direct others to destroy property when

they’re dead.

ii. Note: Prof. Strahilevitz’s Proposal for Conditional Right to Destroy

Property at Death:

1. Testators’ directives to destroy property at death should be

held VALID if:

a. During life, the testator put a future interest in the

property up for sale, and

b. The government declined to condemn the future

interest, and

c. The owner turned down the highest bid for the

future interest

v. Remedies for Invalid Conditions

1. When a conditional gift violates public policy, the critical question is whether there is a

“gift-over clause:” a clause that provides where the gift is to go if the condition is not

met

Chapter 1 – Introduction 5 of 120

a. If gift-over clause exists: Usually the courts distribute the gift as per the

express gift-over clause (and not to the beneficiary)

b. No gift-over clause: Usually, the courts give the gift to the beneficiary, free &

clear of any conditions)

vi. Incentive Trusts and the Dead Hand (p. 35)

1. In modern practice, conditional gifts, such as in Shapira (above, p. 3) tend to be made in

trust (known as an incentive trust).

2. Incentive trusts usu. focus on ensuring that the beneficiary doesn’t adopt a slothful or

wasteful life

3. Prime examples of incentives:

a. Pursue an education

b. Moral incentives – those that reflect the settlor’s (testator’s) moral or religious

outlook, or promote a particular way of living.

c. Pursue a productive career – incentives designed to encourage the beneficiary

to have a productive career

4. RULE: Provided that these incentives do not violate public policy, courts generally will

enforce them.

5. Notes: These must be drafted carefully, so that they are not taken advantage of (see

note p. CB p. 36)

vii. Destruction of Property at Death (CB p. 37)

1. See Property Destruction (above, p. 4)

2. Transfer of the Decedent’s Estate

a. Probate and Nonprobate Property

i. Probate Property: property that passes through probate under the 1) the decedent’s will or 2)

by intestacy

1. i.e. A will disposes of the decedent’s probate property only

2. Note: Probate is the Default: Nonprobate property passes pursuant to the terms of the

instrument in question to the transferees identified in the instrument without passing

through the probate system.

ii. Nonprobate Property: Property that passes outside of probate under an instrument other than a

will.

iii. Examples of Nonprobate Transfers

1. Joint Tenancy Property (both real and personal)

a. Joint tenants hold the property in question concurrently. They own it in whole

and fractional shares.

b. When the decedent dies, his/her fractional share vanishes. With multiple

survivors, the shares are recalculated (this is the right of survivorship).

c. Technically, no property interest “passes” to the survivor(s).

d. RULES: Survivor(s) must show death certificate of the decedent

e. Examples – bank accounts, brokerage and mutual fund accounts, real estate

2. Life Insurance

a. Life insurance is an agreement between the insured and the insurance co that,

upon the insured’s death, the co will pay benefits/proceeds to the named

beneficiary or beneficiaries.

b. RULES: Beneficiary must show death certificate of the insured decedent

c. Notes: At common law, life insurance policies were the only type of contract

with a payment-on-death (POD) clause that qualified as a valid will substitute.

The modern trend recognizes all contracts with P.O.D. clauses as valid,

nonprobate transfers exempt from the probate process.

3. Contracts with payable-on-death (POD) provisions (and see Life Insurance, above)

a. A decedent may have a contract with a bank, an employer, or some other

person/corporation to distribute property at the decedent’s death to a named

beneficiary

Chapter 1 – Introduction 6 of 120

b. Examples:

i. Pension plans – often provide survivor benefits

ii. Tax-deferred investment plans (IRAs, 401(k)s, etc.) often survivor

benefits

iii. Brokerage accounts

c. RULES: Beneficiary need only file a death certificate with the custodian holding

the property

4. Interests in trust

a. Testamentary trusts: Property held in a testamentary trust (i.e. trust created

under the decedent’s will) PASSES THROUGH PROBATE

b. Uniform Testamentary Additions to Trusts Act (UTATA) trusts: These also

PASS THROUGH PROBATE.

c. Inter Vivos trusts: Property put in an inter vivos trust (during decedent’s life)

DOES NOT PASS THROUGH PROBATE

d. Note: When the property is in trust, the trustee holds the property for the

benefit of one or more named beneficiaries (see Trusts on p. 69)

5. Legal Life Estates and Remainders

a. When the party who holds a legal life estate dies, the right to possession passes

to the party holding the remainder.

i. BUT the transfer is the result of the original grantor’s division of the

property between the life estate and the remainder, not the result of

the deceased life tenant passing a property interest.

b. Properly created legal life estates and remainders AVOID PROBATE

iv. Who Takes Nonprobate Property

1. The decedent’s nonprobate property goes to the transferees identified in the written

instrument (as long as it properly creates the nonprobate property arrangement).

v. What Happens If the Nonprobate Instrument Fails to Create Proper Nonprobate Transfer?

1. If the property in question does not qualify as nonprobate property, the property

automatically falls to probate (as the default system).

a. Will vs. Intestacy:

i. Will: A properly executed will constitutes an expression of the

person’s intent as to who should take the property when he/she dies

ii. Intestacy: If 1) a decedent does not have a will, or 2) the decedent’s

will does not dispose of all the decedent’s property, then the property

passes via intestacy to the decedent’s heirs.

b. Intestacy is the DEFAULT: If the decedent takes no steps to opt out of

intestacy (e.g. by writing a will), all of the property passes through intestacy

i. 755 ILCS 5/4-14 – the IL intestacy rule

c. How to Opt Out of Intestacy: One can opt out of intestacy by 1) properly

executing a will, or 2) by properly executing a will substitute – i.e. a recognized

nonprobate method of transferring property. (see Will Substitutes, p. 59)

i. Will – goes through probate

ii. Will substitute – does NOT go through probate

b. Administration of Probate Estates

i. The core functions of Probate

1. Provides Evidence of Transfer of Title to the new owners

2. Protects creditors

a. By providing a procedure for payment of debts (probate ensures that creditors

receive notice, and have a chance to present their claims and receive payment)

b. Note also: Probate also extinguishes the claims of creditors who do not

present their claims to the probate court

3. Distributes the decedent’s property

a. The key is: it distributes decedent’s property to those intended

Chapter 1 – Introduction 7 of 120

b. The other key is: this happens AFTER the decedent’s creditors are paid

ii. Probate Terminology and History

1. Testate: Describes a decedent who dies with a valid last will and testament. His/her

property will be distributed according to the last will and testament

2. Intestate: Describes a decedent who dies WITHOUT a valid last will and testament.

His/her property will be distributed according to the state statute on descent and

distribution.

a. Note: The terms “testate” and “intestate” are not mutually exclusive – a will

may dispose of some, but not all of the decedent’s property.. In that case, the

decedent is both testate AND intestate

3. Testator/Testatrix: A male or female (respectively) who executes a valid will

a. Note: today, “testator” is gender-neutral

4. Devise: A gift of REAL PROPERTY under a will

a. Note: Today, “devise” is increasing being used to describe testamentary gifts

of either real or personal property.

b. The word can be used as a noun or verb.

5. Devisee: A beneficiary receiving real property under a will

6. Bequest: A gift of PERSONAL PROPERTY under a will

a. Note: The word can also be used as a verb (bequeath, bequeaths, etc.)

7. Legacy: A gift of MONEY under a will

8. Legatee: A beneficiary receiving money under a will

9. Personal Representative: The person appointed by the probate court to wind

up/probate the decedent’s affairs

10. Executor: The person appointed by the will itself to wind up/probate the decedent’s

affairs, if the decedent dies testate

a. Note: i.e. “Executor” is what we call a ‘personal representative’ who is

appointed by the will, and not the court.

11. Administrator: What the personal representative is called if the decedent either 1) dies

intestate, or 2) dies testate, but does not name an executor

a. Note: Today, the term “personal representative” is being used increasingly,

regardless of whether the decedent died testate or intestate

12. Probate Court: The state court with special jurisdiction over determining who is

entitled to receive the decedent’s probate property.

13. Statute of descent and distribution: The statute that governs how a decedent’s

intestate property will be distributed

14. Heirs: At common law, beneficiaries who received decedent’s INTESTATE REAL

PROPERTY were called heirs. The property was said to “descend” to heirs.

a. Note: this is different than “devisees,” who received testate real property…

15. Next-of-kin: At common law, beneficiaries who received INTESTATE PERSONAL

PROPERTY were called next-of-kin. Property was said to be “distributed” to next-of-kin.

a. Note: This is different than “I don’t know the word.. but see bequest, above”.

iii. A summary of Probate Procedure

1. Opening Probate

a. Who has probate jurisdiction (Primary Jurisdiction)

i. The probate court in the county where the decedent was domiciled at

the time of death has primary (or domiciliary) jurisdiction over the

decedent’s probate estate.

ii. The court has jurisdiction over 1) the decedent’s personal property

and 2) the decedent’s real property located within that jurisdiction

iii. IL Law (755 ILCS 5/5-1 Place of Probate) – see blue sup p. 14

1. See also 755 ILCS 5/5-2 – Situs of personal estate of NON-

RESIDENT

b. Opening the Probate process

Chapter 1 – Introduction 8 of 120

i. Probate is opened by presenting the decedent’s death certificate

ii. The court issues “letters testamentary” appointing an executor or

“letters of administration” appointing an administrator.

1. Majority Rule: Requires notice to interested parties before

selection and appointment of the executor or administrator

iii. Note: Once the court issues its letters, the personal representative is

authorized to begin his or her responsibilities.

1. IL Law - Representative: Any corporation qualified to accept

and execute trusts in IL is qualified to act as representative

(755 ILCS 5/1-3)

2. IL Law – Debtor as Executor: If a debtor owes testator, and

debtor is executor, he still owes testator UNLESS testator

expressly says the debtor is clear in the will (755 ILCS 5/4-15)

c. Ancillary Jurisdiction

i. If real property is located in another jurisdiction, ancillary

administration may be necessary

ii. Ancillary jurisdiction ensures that

1. 1) local creditors in the jurisdiction where the real property is

located receive notice and an opportunity to present their

claims, and

2. 2) there is compliance with that jurisdiction’s recording

system.

2. Will Contests

a. If a party wishes to file a claim challenging the validity of a will offered for

probate, most jurisdictions have statutes requiring the contest to be brought in

a timely manner after probate is opened, or the claim is barred.

3. Probate Administration: Once the court issues its letters, the personal representative is

authorized to begin his or her responsibilities

a. UPC 3-108: No proceeding, formal or informal, may be initiated more than 3

yrs from the date of death. If it’s more than 3 years, intestacy is presumed.

b. Personal Representative’s Powers: Jurisdictions are split as to the PR’s powers

to administer the estate

i. Formal (a.k.a. Notice) Probate: Require supervision by the probate

court

1. UPC 3-401: Formal probate under the UPC is a litigated

judicial determination after notice to interested parties

2. The ct supervises the actions of the PR in administering the

estate

3. The ct must approve the inventory and appraisal of the

estate; payment of debts; family allowance; granting options

on real estate; sale of real estate; borrowing of funds and

mortgaging of property; leasing of property; proration of

federal estate tax; personal rep’s commissions; atty’s fees;

prelim & final distributions; and discharge of the personal rep

4. This is SLOW and expensive

5. Formal proceedings become final judgments if not appealed

ii. Informal (a.k.a. Ex Parte) Probate: The personal rep administers the

estate without going back to court

1. Note: Under UPC, Informal probate is the norm

a. An interested party may file a petition for formal

probate (UPC 3-502)

2. E.g. rep has broad powers of a trustee in dealing w/ estate

property – rep may collect assets; clear titles; sell property;

Chapter 1 – Introduction 9 of 120

invest in other assets; pay creditors; continue any business of

the decedent; and distribute the estate – ALL without going

court approval (UPC 3-715)

3. The personal rep may close the estate by filing a sworn stmt

that he has published notice to creditors, administered the

estate, paid all claims, and sent a statement and accounting

to all known distributes (UPC 3-1003)

4. (we assume the personal rep is a trusted family member)

5. Requirements for Informal Probate (UPC 3-301)

a. The rep petitions for appointment (rep doesn’t have

to give notice of this)

i. The petition contains pertinent info about

the decedent and the names/addresses of

the spouse, children or other heirs.

ii. If a will is involved, the petition also

includes devisees (rx’ers of real property)

iii. If the petition is for probate of a will, the

original will must accompany the petition

c. Personal Representative’s DUTIES:

i. Inventory decedent’s assets: Duty to ascertain and take control of

decedent’s property, which he/she inventories to the probate court

ii. Give notice to and pay creditors:

1. Duty to give notice (usu by publication, or by actual notice) 1)

of the opening of probate and 2) that the creditors are

required to file all claims w/in a set statutory pd or else their

claims will be forever barred. (Statutory pd is governed by

nonclaim statutes)

2. Duty to pay those creditors 1) who present valid claims w/in

the prescribed time pd

3. Duty to file federal and state estate tax returns and, if

necessary, pay any taxes due

4. Probate and “Titled” property: Probate is necessary to transfer title to those assets,

real or personal, that were titled in the decedent’s name.

a. Where the probate asset has a written form of title in the decedent’s name, a

probate court order is needed to transfer title properly.

iv. The Costs of Probate

1. Probate is fucking expensive and slow ; even a quick probate may take up to 2 years

2. Avoiding Probate: Increasingly, people try to avoid probate because it is expensive and

slow. In reality, it is difficult to put ALL property in nonprobate arrangements

a. “Nontitled” probate assets: Probate can be avoided if all of the decedent’s

property is non-titled personal property (e.g. furniture or personal effects).

BUT if the takers (heirs) opt not to probate, then they may be subject to

creditors’ claims.

b. “Small Estate” probate assets: All states have statutes that allow heirs to

avoid probate where the amount of property involved is small (see UPC 3-1201

to 3-1204)

c. Universal Succession (European approach -- and to a limited extent, USA (e.g.

California)): Title to the decedent’s property passes to the appropriate heirs or

residuary devisees automatically and by operation of law without the need for

a personal rep or probate

i. Who pays the creditors?: The heirs/residuary legatees who take title

to the decedent’s assets are then responsible for paying the

Chapter 2 – Intestacy: An Estate Plan By Default 10 of 120

decedent’s creditors and the estate’s tax liability and distributing the

decedent’s property to the appropriate takers.

ii. Louisiana is the only United States state that has adopted universal

succession

iii. Universal succession is in limited form in e.g. CA permits universal

succession for property passing to a surviving spouse)

v. See also IL Statutes

1. 755 ILCS 5/1-1 to 11 (“General Provisions”)

2. 755 ILCS 5/25-1 to 4 (“Small Estates”)

3. 755 ILCS 5/28-1 to 12 (“Independent Administration”)

Chapter 2 – Intestacy: An Estate Plan By Default 1. The Basic Scheme

a. Introduction

i. Intestacy is the norm – roughly half the population dies intestate

ii. Heirs v. Heirs Apparent

1. Heir = a person who SURVIVES the decedent (i.e. the decedent is already dead)

2. Heir Apparent = a person who is designated as the heir of someone who is still alive (i.e.

no decedent yet)

3. Expectancies: An expectancy is an expectation to receive some property at the death of

the decedent.

a. Not a property Interest: Such an expectation by an heir apparent is NOT a

property interest. The heir must survive the decedent to take anything, and

even so, the decedent can defeat the expectancy by transferring property inter

vivos or by executing a will that devises the property to others

b. Transferability: A mere expectancy cannot be transferred at law (because it is

not a property interest).

i. BUT if the heir apparent agrees to transfer his/her expectancy for

valuable consideration, and thereafter tries to avoid enforcement of

the agreement on the grounds that an expectancy is not transferable,

a court of equity will enforce the agreement if enforcement is fair and

equitable under the circumstances.

4. POLICY behind Intestacy Statutes:

a. Carry out the probable intent of the average intestate decedent

i. E.g. favor the surviving spouse

b. Protect the family; i.e. preserve the economic health of the family after a

death.

iii. UPC Intestacy Scheme

1. UPC 2-102 – The Spouse’s Share: The decedent’s surviving spouse gets:

a. 100% of intestate estate if:

i. No descendant or parent of the descendant survives the decedent, OR

ii. All of the dead’s surviving descendants are also descendants of the

surviving spouse AND there is no other descendant of the surviving

spouse who survives the dead

b. The first $300,000 + 75% of any balance of the intestate estate if:

i. No descendant of the dead survives the decedent, but a parent of the

dead survives the dead

c. The first $225,000 + 50% of any balanace of the intestate estate if:

i. All of the dead’s surviving descendants are also descendants of the

surviving spouse AND

ii. the surviving spouse has one or more surviving descendants who are

not descendants of the dead

d. The first $150,000 + 50% of any balance of the intestate estate if:

Chapter 2 – Intestacy: An Estate Plan By Default 11 of 120

i. One or more of the dead’s surviving descendants are not descendants

of the surviving spouse

2. UPC 2-103 – Share of Heirs Other Than Surviving Spouse: Any part of the intestate

estate not passing to a decedent’s surviving spouse under 2-102 (or the entire intestate

if there is no surviving spouse) passes in the following order to the other surviving heirs

(who aren’t the spouse).

a. If there are surviving descendants

i. To the dead’s descendants by representation

b. If there is no surviving descendant:

i. 50/50 to each of the decedent’s parents, OR

ii. 100% to the surviving parent if only one survives

c. If there is no surviving descendant or parent

i. To the descendants of the dead’s parents (i.e. to the dead’s siblings),

or either of them (the parents) by representation

d. If there is no surviving descendant, parent, or descendant of a parent (e.g.

sibling), but the dead is survived on both the maternal and paternal sides by

one or more grandparents or descendants of grandparents (i.e. aunts/uncles)

i. 50% to 1) the dead’s paternal grandparents equally if both survive, OR

2) to the surviving paternal grandparent if only one survives, OR 3) to

the descendants of the dead’s paternal grandparents (i.e. the dead’s

aunts/uncles) if both GPs are deceased (the descendants taking by

representation); AND ALSO

ii. 50% to 1) the dead’s maternal grandparents equally if both survive, OR

2) to the surviving maternal grandparent if only one survives, OR 3) to

the descendants of the dead’s maternal grandparents (i.e. the dead’s

aunts/uncles) if both GPs are deceased (the descendants taking by

representation);

3. UPC 2-105 – No Taker

a. If there is no taker under the provisions of this Article, the intestate estate

passes to the state.

b. Share of Surviving Spouse – Who Qualifies as a Spouse?

i. Usually, the spouse takes first, before anyone else

ii. Policy Issues: (1) Should the surviving spouse take ALL of the deceased spouse’s intestate

property if there are surviving issue (kids?), (2) should the surviving spouse take ALL of the

deceased spouse’s intestate property if there are surviving parents or issue of parents (i.e.

siblings and their issue?)

iii. “Spouse” can be:

1. Actually married people

2. Domestic Partners

iv. “Spouse” DOES NOT INCLUDE

1. Cohabitants (nonmarried couples who live together)

2. Common law marriage (not all jurisdictions recognize common law marriages)

v. Same-Sex Marriage, Domestic Partners, and Intestate Succession

1. Most states do NOT recognize same sex marriages; i.e. they do not allow same-sex

couples to marry, and therefore, they do not recognize spousal-like intestacy rights

2. Civil Unions – some states DO allow spousal-like intestacy rights, but do not allow gay

marriage

vi. Contract Claims (Same-Sex Partners)

1. In some states, a surviving same-sex partner might have a claim against the deceased

partner, based on contract law.

a. Whether the K must be express (either oral or written—as opposed to implied)

varies from jurisdiction to jurisdiction

vii. Defense of Marriage Act: In 1996, Congress enacted the Defense of Marriage Act

Chapter 2 – Intestacy: An Estate Plan By Default 12 of 120

1. Defines “marriage” for federal purposes as applying to only heterosexual couples

2. States that despite the Full Faith & Credit Clause of the Constitution, states are not

required to recognize same-sex marriages contracted in other states.

viii. Putative Spouses: Putative spouses generally do qualify as spouses

1. E.g. “putative spouse” == the couple goes through what at least one of the parties

believes is a valid marriage ceremony, but the marriage is either void or voidable (e.g.

one spouse is already married and not divorced).

2. RULE: As long as one party believes, in good faith, that the marriage is valid, the

spouses qualify as putative spouses and are treated as spouses for purposes of most

intestate schemes.

ix. Married but separated: Spouses who are legally separated generally still qualify as spouses for

the purposes of the intestate distribution scheme.

1. Note: Even if the parties have filed for divorce, the parties remain legally married until

the court enters final judgment or decree of dissolution of marriage.

2. Spousal Abandonment: In some states, if one spouse abandons the other, the

abandoning spouse may be disqualified from inheriting from the other spouse.

x. Survival requirements: To be eligible to receive property from a decedent, a taker must

“survive” the decedent. If the claimant fails to meet the survival requirement, the claimant is

treated as if he/she predeceased the decedent.

1. Scope: Historically, survival requirement applied to anyone claiming a decedent’s

probate testate or intestate property, but not nonprobate property. Modern trend

applies to nonprobate, as well

2. Common law: To qualify as an heir, the party had to prove, by a preponderance of the

evidence, that he/she survived the decedent by a millisecond. Whether a person

survived the decedent is a question of fact.

3. Uniform Simultaneous Death Act (USDA): If there is no sufficient evidence as to who

survived whom, the party claiming a right to take is to be treated as having predeceased

the decedent.

a. Criticism of the USDA: If 2 spouses die together (or near the same time), the 2

families should be grieving together, not suing each other

4. IL Law (755 ILCS 5/3-1): IL Law assumes that the order of deaths is apparent from

evidence. 5/3-1 Governs what happens if the order of deaths cannot be learned from

evidence.

5. Janus v. Tarasewicz (p. 80)

6. F/P

a. Stanley and Theresa Janus returned from their honeymoon to learn that

Stanley’s brother had died unexpectedly. The couple was distraught and

unknowingly took some Tylenol laced with cyanide. Stanley collapsed first,

Theresa a short time later.

b. Although there was conflicting medical evidence, Stanley’s vital signs arguably

disappeared during the ambulance ride to the hospital, and he was pronounced

dead shortly after arrival. Theresa arguably still had a pulse and blood pressure

upon arrival at the hospital, and she lived (on a respirator) for 2 more days

before being pronounced dead.

c. Stanley’s life ins policy named Theresa as a beneficiary; if she failed to survive

him, his mother was named as contingent beneficiary.

d. Stanley’s mother sued the ins co and the administrators of Stanley and

Theresa’s estates for the funds, claiming that there was insufficient evidence

that Theresa had survived Stanley (i.e. the mother should get the $$$)

7. I

a. Did Theresa in fact survive Stanley?

8. R/A/H

Chapter 2 – Intestacy: An Estate Plan By Default 13 of 120

a. HELD: Stanley’s mother loses—there was sufficient evidence to support the

finding that Theresa had survived Stanley

b. RULE: USDA rule – if the title to property depends on the priority of death, and

there is no sufficient evidence that the persons died other than simultaneously,

the property of each person shall be disposed of as if he survived (see p. 84)

c. RULE: Survivorship is a FACT which must be proven by a PREPONDERANCE OF

THE EVIDENCE.

d. There was sufficient evidence that Theresa was alive, with a palpable pulse and

blood pressure, for about 48 hours after Stanley was pronounced dead.

xi. Determining time of death: To determine survivorship, one needs to know when each party

died

1. Common Law: Time of death = when there is an irreversible cessation of circulatory and

respiratory functions

2. Modern Trend: Time of death = when there is irreversible cessation of total brain

function (because circulation & respiration can be maintained artificially now)

3. The Clear and Convincing Evidence Standard: To minimize simultaneous death

litigation

a. RULE: Some states require that a claimant prove by clear and convincing

evidence that he/she survived the decedent

4. UPC 120-Hour Approach: UPC 2-104 and 2-702 provide that:

a. RULE: An heir or devisee or life insurance beneficiary who fails to survive by

120 hours (5 days) is deemed to have predeceased the decedent.

b. The USDA was amended in 1990 to include the 120 Hour rule.

c. Advantages of 120 Hour Rule: It addresses simultaneous deaths even if they

don’t arise from the same disaster

d. Criticisms:

5. IL Law: 755 ILCS 5/3 (blue sup. p. 11)

xii. Failure to meet survival requirement: Whichever standard is applied, if the claimant fails to

meet the survival requirement, the claimant is treated as if he/she predeceased the decedent.

xiii. Wills and Nonprobate instruments:

1. If the written instrument does NOT have its own survival requirement, the statutory

survival requirement applies.

2. If the written instrument DOES have an express survival requirement, it applies.

c. Shares of Descendants (p. 87)

i. If A) there is no surviving spouse, or B) there is a surviving spouse, but he/she does not take all of

the decedent’s property, then property goes to the decedent’s issue equally

ii. After the spouse’s share (if any) is set aside, A) children and B) descendants of deceased children

take the remainder of decedent’s property; everyone else is excluded

1. The descendants of deceased children “represent” the dead child, and divide the dead

child’s share among themselves

2. i.e. if the decedent’s had 3 sons while alive, and one of the sons died before the

decedent did, then the dead son’s descendants “represent” the dead son, splitting his

share among themselves

3. Example of Representation: A dies intestate (and with no surviving spouse). A has 3

children, B, C, and D. C dies before A, but leaves a husband and 2 children.

a. B takes 1/3 of A’s estate (and any of B’s children get nothing, because B got the

intestate share)

b. D takes 1/3 of A’s estate (and any of D’s children get nothing, because D got the

intestate share)

c. C would have taken 1/3 of A’s estate, but C’s dead. So, C’s 2 kids split C’s share,

by representation. C’s husband gets nothing (intestacy usually excludes

children in-law)

iii. IL Law = 755 ILCS 5/2-1 (blue sup p. 4)

Chapter 2 – Intestacy: An Estate Plan By Default 14 of 120

iv. Distribution Schemes

1. English Per Stirpes: Always make the first division of decedent’s property at the first

generation of descendants (whether there are any living takers or not); the dropping

shares then drop by bloodline

a. b. Assume A, B, C, D, E, H, and J all predecease the decedent, who dies with no

surviving spouse, and intestate.

c. Step 1: Under per stirpes, the property is always divided at the first generation

(i.e. divided among decedent’s children), even if everyone in that generation is

dead.

d. Step 2: One share is given to each party who is alive, and one share is given to

each party who is dead but survived by issue.

i. e.g. here, even though A, B, and C are all dead, A, B, and C are all

survived by issue, so each receives a 1/3 share).

e. Step 3: Under English per stirpes, the shares for each party who is dead but

survived by issue drop by bloodline. Each share drops only to the issue of the

predeceased party.

i. E.g. here, A’s 1/3 share is divided equally between E and F (so they

each get 1/6). But, E is dead, so E’s 1/6 drops to K, L, and M, equally

(so each of them gets 1/18).

ii. B’s 1/3 all goes to G. Because G is alive, N gets nothing

iii. C’s 1/3 is divided among H, I, and J. But because H is dead and has no

surviving issue, H gets nothing. So actually, C’s share is split into 2, not

3. I and J each get 1/6. I is alive, I gets 1/6. Because J is dead, J’s 1/6

goes to O and P, who each get 1/12.

f. Criticism: It is possible for descendants of equal degree to the decedent to take

unequally.`

2. Modern Per Stirpes (a.k.a. Per Capita with Representation): Make the first division of

the decedent’s property at the first generation where there is a live taker; the dropping

shares then drop by bloodline.

a. b. Assume A, B, C, E, H, and J all predecease the decedent, who then dies

intestate (with no surviving spouse). Who takes the property?

c. Step 1: Always divide at the first generation where there is a living taker.

i. E.g. here, we start divding at the E, F, G, H, I, J level

d. Step 2: Divide equally among all parties who are 1) alive or 2) dead, but

survived by issue).

Chapter 2 – Intestacy: An Estate Plan By Default 15 of 120

i. E.g. Here, F, G, and I are alive; E, and J are dead, but are survived by

issue; H is dead and has no surviving issue

ii. So, the property is split into 5. F, G, and I each get 1/5.

e. Step 3: Drop the remaining shares by bloodine.

i. Here, E’s issue (K, L, and M) each get 1/15 (a 3-way split of E’s 1/5

share). J’s issue (O and P) each get 1/10 (a 2-way split of J’s 1/5

share).

f. Criticism: It is possible for descendants of equal degree to the decedent to take

unequally.

g. Benefit: The per capita at each generation approach ensures that all

descendants who are equally related to the decedent take equally. This is as

opposed to the per-capita-at-each-generation approach “pools” the dropping

shares (the shares for descendants who are dead but survived by issue).

“Pooling” means that the dropping shares are added together, and then

divided equally among all of the eligible takers at the next generation.

3. Per capita at each generation: Always make the first division of decedent’s property at

the first generation where there is a living taker, and the dropping shares drop by

pooling—combine them and distribute them equally among the eligible takers at the

next generation

a. b. Assume A, B, C, E, H, and J all predecease the decedent, who then dies

intestate and with no spouse.

c. Step 1: Always divide property at the first generation where there is a living

taker.

i. Here, begin dividing at E’s generation.

d. Step 2: One share is given to each party who is alive, and one share is given to

each party who is dead but survived by issue.

i. Here, F, G, and I are alive – they all get 1/5. E and J are both dead, but

survived by issue. H is dead and has no surviving issue. So H gets no

share.

e. Step 3: Pool the dropping shares by adding them together, and then dividing

the total equally among the eligible takers at the next generation.

i. Here, there are 2 dropping shares. Add them together (E’s 1/5 + J’s

1/5 = 2/5). Divide the 2/5 equally among the eligible takers at the

next generation (K, L, M, O, and P – N is not eligible, because her

parent took already). So K, L, M, O, and P each get 2/25.

4. IL Law (755 ILCS 5/2-1): (see blue sup, p. 4)

v. Power to Opt Out: An individual can opt out of a jurisdiction’s default intestate distribution

approach by executing a valid will or nonprobate instrument that expressly provides for an

alternative method of distributing decedent’s estate.

vi. UPC approach: The original UPC (1969 version) adopted the per capita with representation

approach. The revised version has adopted the per capita with each generation approach (UPC

2-106)

vii. Negative Disinheritance (p. 91)

Chapter 2 – Intestacy: An Estate Plan By Default 16 of 120

1. UPC 2-101(b) allows a negative will. The barred heir is treated as if he disclaimed his

intestate share; i.e. he is treated as having predeceased the decedent

a. Common law: Execute a valid will that disposes of all of the decedent’s

property so that nothing passes through intestacy (depriving the heir of any

chance of taking. This is because the heir could inherit against the clear intent

of the will if any property is left to intestacy)

b. Modern trend/UPC approach: Execute a will that merely expresses intent to

disinherit an potential heir, even if some or all of the decedent’s property

passes through intestacy and the heir otherwise would have qualified to take

property. (UPC 2-101(b))

d. Shares of Ancestors and Collaterals

i. If a decedent has no surviving spouse or issue, the property flows “up” the family tree to the

decedent’s ancestors and collateral relatives. There are 3 major approaches to this:

1. Parentelic Approach: The intestate estate passes to grandparents and their

descendants; if no grandparents/descendants, then to great-grandparents and their

descendants, etc.

a. In distributing the property, the per stirpes, per capita, or per capita at each

generation doctrines apply, depending on the default approach in the state.

2. Degree of Relationship Approach: The intestate estate passes to the closest of kin,

counting degrees of relationship

a. Determining the degree of relationship: On the family tree, count the steps

(one for each generation) up from the decedent to the nearest common

ancestor of the decedent and the claimant, and then count the steps down to

the claimant from the common ancestor (The count always goes up.. i.e. don’t

start subtracting when you count down from the nearest ancestor to the

claimant).

3. Degree of Relationship with a Parentelic Tiebreaker Approach:

a. Step 1: Determine the degree of relationship of the possible takers. Those of

closer degree (lower number) take to the exclusion of those of a higher, more

remote relationship.

b. Step 2: If there are multiple takers sharing the lowest degree of relationship,

under the parentelic tiebreaker, those in closer parentelic/collateral lines take

to the exclusion of those in the more remote parentelic/collateral lines.

ii. RULE: When the intestate decedent is survived by a descendant, the decedent’s ancestors and

collaterals do not take.

iii. Definition: Collateral relatives = Persons who are related by blood to the decedent, but who are

not descendants or ancestors (e.g. siblings a.k.a. first-line collaterals, cousins)

iv. UPC Approach – If the decedent is not survived by a spouse, descendant, or parent, then

intestate property passes to brothers/sisters and their descendants. The descendants of any

deceased brothers and sisters (nephews and nieces) take by representation, usually in the same

manner as the decedent’s descendants. (UPC 2-106(c) calls for representation per capita at each

generation).

v. Half-Bloods (p. 96)

1. Half-bloods are relatives who share only one common parent, as opposed to the

traditional relationship where siblings share both parents.

2. At Common Law: Only whole-blood relatives could inherit from intestate parents

3. UPC and modern trend majority: Under UPC 2-107, half-bloods are treated the same as

whole-blood relatives – they can inherit equally (e.g. if A and B have the same mother,

but different fathers, and the mother dies intestate, A and B have equal rights to inherit

from W)

4. IL doesn’t distinguish btwn whole and half-bloods (755 ILCS 5/2-1)

2. Transfers to Children

a. Meaning of “children”

Chapter 2 – Intestacy: An Estate Plan By Default 17 of 120

i. Adopted Children

1. 755 ILCS 5/2-4

2. Hall v. Vallandingham (p. 97)

3. F/P

a. Earl J. Vallandingham, died in 1956 and was survived by his widow, Elizabeth,

and their four children, appellants. Two years later, Elizabeth married Jim

Walter Kilgore, who adopted the appellants. In 1983, Earl’s brother, William Jr.,

died intestate with his sole heirs being his surviving brothers and sisters and

the children of his brothers and sisters who predeceased him. After the

Inventory and First Accounting of Williams estate were filed, appellants alleged

they were entitled to a distributive share of their natural uncle’s estate that

their natural father would have received if he had survived William. The

Orphan’s Court transmitted the issue to the Circuit Court for St. Mary’s County

and the tribunal determined the four natural children of earl were not entitled

to a distribution from William’s estate because of their adoption by Kilgore.

Appellants, unwilling to accept the court’s disposition, appeal.

4. I

a. Whether the trial court erred in denying the appellants the rights to inherit

through their natural paternal uncle, due to the fact appellants were adopted

as minors by their stepfather after the death of their natural father?

5. R/A/H

a. HELD: No. Judgment affirmed.

b. RULE: Under the general rule, the adopting parent steps into the shoes of the

natural parent of the same gender, and the parent-child relationship with the

natural parent is completely severed.

c. The court held that because Est. & Trusts Art. Section:1-207(a) eliminates the

adopted child’s right to inherit from the natural parent, it also abrogated the

right to inherit through the natural parent by way of representation.

d. The right of inheritance was removed by the Maryland Legislature in 1963

when it declared: “Upon entry of a decree of adoption, the adopted child shall

lose all rights of inheritance from its parents and from their natural collateral or

lineal relatives.

e. Absent clear legislative intent, adopted children should be no better off than

nonadopted children and should have only two parents from whom they can

inherit.

ii. Adoption by Relative of Parent (2008 Amendments to the UPC): A parent-child relationship

exists between adopted child and the adoptive parent (UPC 2-118(a)), but not between an

adopted child and the child’s genetic parents (UPC 2-119(a))

1. A parent-child relationship exists between an individual who is adopted by the spouse of

either genetic parent and (1) the genetic parent whose spouse adopted the individual;

AND (2) the other genetic parent (but only for the purpose of the right of the adoptee or

a descendant of the adoptee to inherit from or through the other genetic parent) (UPC

2-119(b))

2. Where a child is adopted by a relative of either natural/genetic parent, or the spouse or

surviving spouse of a relative, the child retains the right to inherit from and through

BOTH natural/genetic parents (UPC 2-119(c))

3. IL Law: 755 ILCS 5/2-4 (blue sup p. 6)

4. R.3d Property (Wills) - § 2.5 (CB p. 100): General rule – adopted child has parent-child

relationship w/ the adoptive parents, not his/her natural parents.

iii. Post-death adoption: Under the 2008 revisions to the UPC, where a child is adopted after the

death of both natural/genetic parents, the child retains the right to inherit through BOTH

natural/genetic parents. (UPC 2-119(d)).

Chapter 2 – Intestacy: An Estate Plan By Default 18 of 120

iv. Adoption of Adults: As a general rule, adopted adults are treated the same as adopted children

for inheritance purposes.

1. Minary v. Citizens Fidelity Bank & Trust Co. (p. 103)

2. F/P

a. Amelia S. Minary, died in 1932, leaving a will that devised her residuary estate

in trust, to pay the income to her husband and three sons, James, Thomas, and

Alfred, for their respective lives. The trust was to terminate upon the death of

the last surviving beneficiary, at which time the corpus was to be distributed to

decedent’s “then surviving heirs, according to the laws of descent and

distribution then in force in Kentucky, and if no such heirs, then to the First

Christian Church, Louisville, Kentucky.”

b. Minary’s husband died, then James died without issue, and then Thomas died

leaving two children. In 1934, the only surviving son, Alfred, married Myra,

respondent, and in 1959 adopted her as his child. The trust terminated in 1963

upon Alfred’s dying without natural issue.

3. I

a. Is respondent is included in the term “my then surviving heirs” according to the

laws of descent and distribution in force in Kentucky?

4. R/A/H

a. RULE: An adult person may be adopted in the same manner as provided by law

for the adoption of a child and with the same legal effect.

b. HELD: The respondent is NOT included in “my then surviving heirs”

c. Although the adoption technically fell within the express terms of the statute,

such an adoption was a subterfuge that thwarted the remote ancestor’s intent.

v. Adoption and Its Impact on Class Gifts

1. UPC 2-705(f) excludes a person adopted after reaching the age of 18 from a class gift

that is 1) GIVEN BY someone other than the adopting parent and 2) GIVEN TO the

adopting parent’s children, issue, descendants, or heirs; UNLESS the adopting parent

was the adoptee’s stepparent or foster parent, or the adopting parent “functioned as a

parent of the adoptee before the adoptee” turned 18. See also R.3d Will 14.5.

2. UPC 2-705(f) excludes a person adopted after reaching the age of 18 from a class gift to

the adoptive parent’s children, issue, descendants, or heirs by someone other than the

adoptive parent unless the adoptive parent was the adoptee’s stepparent or foster

parent, or the adoptive parent “functioned as a parent of the adoptee before the

adoptee” turned 18.

vi. Posthumous Children

1. Typical case = child conceived before, but born after father’s death

a. If posthumously born children are born to a couple that was married during

husband’s life, the child is a descendant of the couple.

2. General Rule: Where it is to the child’s advantage to be treated as “in being” from the

time of conception, rather than from the time of birth, the child will be so treated, if

born alive

3. Common Law: Courts have established a rebuttable presumption that the normal

period of gestation is 280 days (10 lunar months). If the child claims that conception

dated more than 280 days, the burden of proof is on the child.

4. Uniform Parentage Act (UPA § 204) – establishes a rebuttable presumption that a child

born to a woman within 300 (rather than 280) days after the death of her husband is a

child of that husband.

5. 755 ILCS 5/2-3: Posthumous children receive the same share of an estate as if the child

had been born in the decedent’s lifetime.

vii. Nonmarital Children (Children Born out of Wedlock)

1. Common Law: Children born out of wedlock were considered illegitimate and could not

inherit from either the mother or the father.

Chapter 2 – Intestacy: An Estate Plan By Default 19 of 120

2. Modern Trend/UPC Approach: A child has a parent-child relationship with both

natural/genetic parents regardless of their marital status (UPC 2-117).

a. Parent-Child Relationship with Mother: Under the modern trend, a child born

out of wedlock automatically has a parent-child relationship with his/her

natural mother (assuming no surrogate mother) and can inherit from and

through the natural mother

b. Parent-Child Relationship with Father: Inheritance from and through a father

usu requires proof of paternity

i. E.g. Evidence of subsequent marriage of the parents; by

acknowledgement by the father; by an adjudication during the life of

the father; or by clear and convincing proof after his death.

c. 755 ILCS 5/2-2: (see blue sup p. 5) – Note: This statute governs both (1)

children born out of wedlock as testators (e.g. who has rights to inherit from

them), and (2) children born out of wedlock as heirs/devisees/legatees (how

children born of wedlock receive distributions)

viii. Reproductive Technology and New Forms of Parentage

1. Posthumously conceived children (i.e. a mother uses a father’s frozen sperm to

conceive) – Note this is children who are conceived after the father’s death, e.g. by

artificial insemination.

a. Key Issue: Should the posthumously conceived child be treated as a child of

the predeceased natural parent for purposes of distributing his or her estate?

b. Woodward v. Commissioner of Social Security (p. 118)

c. F/P

i. James Woodward married the appellant in 1993. Three years later,

Woodward learned that he had leukemia. He was advised that the

leukemia treatment my leave him sterile. Therefore he decided to

arrange for a quantity of his semen to be preserved. Woodward died

later that year.

ii. In 1995, the appellant gave birth to two children. She applied two

forms of social security, “child’s” benefits and “mother’s” benefits.

iii. The Social Security Administration rejected her claim on the ground

that she had not shown that the twins were the husband’s children

under the meaning of the Act because they were not entitled to The

wife appealed to the US Dist Ct for the District of Mass, seeking decl

judgment to reverse the commissioner’s rulinginherit under the

Massachusetts intestacy and paternity laws.

iv. Dist ct judge certified the question to the Mass Supreme Court

d. I

i. May posthumous children who are the result of artificial insemination

receive social security benefits if their genes can be traced to the

alleged father?

e. R/A/H

i. HELD: In certain circumstances, yes – posthumously conceived

children may enjoy the inheritance rights of "issue" (under Mass’s

intestacy scheme)

ii. RULE: As a threshold matter, the surviving parent must show that the

prospective donor parent:

1. Clearly and unequivocally consented to posthumous

reproduction AND

2. Clearly and unequivocally consented to the support of any

resulting child

3. AND ALSO that the deceased parent is the genetic parent of

the child

Chapter 2 – Intestacy: An Estate Plan By Default 20 of 120

f. Notes: After the donor parent's death, the BURDEN OF PROOF is on the

surviving parent, or the posthumously conceived child's other legal

representative, to prove the deceased genetic parent's affirmative consent to

both requirements

2. Intestacy, and Construction of Wills, Trusts, and Other Instruments

a. Technically, whether a posthumously conceived child qualifies as “child” or

“heir” for purposes of a will, trust, or other written instrument is a question of

the intent of the decedent—not a question of intestate rules. The written

words of the instrument are presumed to be the best evidence of the

decedent’s intent. But often the instrument fails to address the issue. In such

cases, the modern trend is to treat the child as a child of the decedent.

b. In re Martin B. (p. 126)

c. F/P

i. Grantor established 7 trusts for the benefit of his “issue/descendants.”

Grantor’s son died six months earlier, with no children, but before he

died he banked sperm and authorized his wife to use it. Using his

sperm, three years later she gave birth to a son, and then two years

later, to a second son.

ii. The trust gave the trustees the discretion to use the principal for the

settlor’s issue during the grantor’s wife’s life, and upon her death, to

distribute the principal to their issue.

d. I

i. May posthumously conceived children receive trust property?

e. R/A/H

i. HELD: James and Warren (the sons) are “issue” and “descendants” for

all purposes of these trusts.

ii. RULE: Where a governing instrument is silent, children born of this

new biotechnology with the consent of their parent are entitled to the

same rights for all purposes as those of a natural child.

f. UPC Approach: The 2008 revisions to the UPC expressly provide that a

posthumously conceived child is included in a class gift in a trust or will of a

third party (someone other than the predeceased donor parent) as long as

i. (1) the predeceased parent authorized the posthumous use of the

genetic material in a signed writing or there is clear and convincing

evidence of such consent (UPC 2-705(b) and 2-120(f)), AND

ii. (2) the child is living on the distribution date or is in utero within 36

months of, or is born within 45 months of the distribution date (UPC

2-705(g))

3. Surrogate Motherhood and Married Couples (p. 130)

a. UPC Approach: With respect to a child born to a surrogate (a gestational

carrier), UPC 2-121 (2008) provides that:

i. In the absence of a court order to the contrary, the surrogate does not

have a parent-child relationship with the child UNLESS the surrogate is

the child’s genetic mother AND no one else has a parent-child

relationship with the child.

ii. An intended parent of the child (i.e. a person who entered into an

agreement with the surrogate stating that the person would be the

child’s parent) has a parent-child relationship with the child.

4. Assisted Reproduction and Same-Sex Couples (p. 132)

a. UPC Approach to Parent-Child Relationship:

i. A child conceived by assisted reproduction other than gestational

surrogacy is in a parent-child relationship (and thus entitled to inherit

by, from, or through) the child’s birth mother (UPC 2-120(c))

Chapter 2 – Intestacy: An Estate Plan By Default 21 of 120

ii. There can also be a parent-child relationship with another person if

the other person either A) consented in writing to assisted

reproduction by the birth mother with the intent to be the other

parent of the child or B) functioned as a parent of the child within two

years of the child’s birth (UPC 2-120(f))

5. Advancements (p. 133)

a. 755 ILCS 5/2-5: Advancements (blue supp p. 7)

b. Advancements doctrine addresses issue whether inter vivos gifts a decedent

made to an heir should count against the heir’s share of the decedent’s

INTESTATE estate.

i. Common Law: Any lifetime gift given by the testator/decedent to a

child was presumed to be an “advancement” – essentially a pre-

payment of the child’s intestate share. The burden of proof was on

the child to prove that the transfer was intended as an absolute gift,

not to be counted against his share of the estate.

1. Hotchpot: All inter vivos gifts to the child are added back (on

paper) into the parent’s probate intestate estate to create

the “hotchpot.” Then the hotchpot is divided equally among

heirs

2. Ex: D died intestate w/ 3 children, A, B, and C. D gave A

$25,000 in inter vivos gifts. D gave A $50,000, and D gave C

$75,000. Then D died with a $150,000 intestate estate. What

do the kids get?

a. The Hotchpot is $300,000: The Advancements to A,

B, and C add up to $150,000. That is added to D’s

intestate estate of $150,000. So the total hotchpot

is $300,000.

b. Then, A, B, and C each get an even split--$100,000

each total. So, from the estate, A gets $75,000,

because A had already had gotten $25,000 in

advancements (pre-payments) in life. B gets $50,000

from the estate; C gets $25,000 from the estate. So

here Estate + Advancement = $100,000.

ii. Modern/UPC Approach: Inter vivos gifts do not count as an

advancement UNLESS a writing expressly indicates that the donor

wants the gift to be an advancement. UPC 2-109 (see CB p. 135):

Requires writing – see also Emanuel’s p. 46

b. Bars To Succession

i. There are situations when an otherwise eligible taker is barred from taking a share of an

intestate estate.

ii. Homicide: Where a party (a killer) who otherwise is entitled to take from a decedent kills the

decedent, the killer (i.e. the now-DQ’ed taker) cannot take. The equitable principle that one

should not profit from one’s own wrongdoing

1. IL Law (755 ILCS 5/2-6) (blue supp p. 7) – IL law says the same thingsays the same thing

2. In re Estate of Mahoney (p. 145)

3. F/P

a. On May 6, 1961, Howard Mahoney, decedent, died intestate of gunshot

wounds.

b. His wife, Charlotte Mahoney, appellant, was tried for the murder of decedent

and was convicted of manslaughter by a jury in March 1962. Appellant is

currently serving a sentence of not less than 12 nor more than 15 years at the

Women’s Reformatory in Rutland.

Chapter 2 – Intestacy: An Estate Plan By Default 22 of 120

c. Decedent left no issue, but was survived by appellant, and his mother and

father. His father, Mark Mahoney, was appointed administrator of his estate,

which amounts to $3,885.89.

d. The probate Court for the District of Franklin entered a judgment order

decreeing the residue of decedent’s estate, in equal shares, to decedent’s

mother and father.

e. Appellant appeals from the judgment order and decree.

4. I

a. Can a widow convicted of manslaughter in connection with the death of her

husband inherit from his estate?

5. R/A/H

a. HELD: Yes. Decree reversed and cause remanded.

b. RULE: The rule should be drawn between VOLUNTARY and INVOLUNTARY

manslaughter.

i. Here, the conviction did not indicate the degree – the case was

remanded to determine the degree of manslaughter.

c. Legal title passes to the slayer but equity holds him to be a constructive trustee

for the heirs or next of kin of the decedent

d. The probate court was without jurisdiction to impose a constructive trust on

the estate in the hands of the appellant. The jurisdiction over charging the

appellant with a constructive trust on the estate of Mahoney lies in the court of

chancery, and not in the probate court.

e. The principle that one shouldn’t be able to profit thru their wrong must not be

extended to every case in which a killer acquires property from his victim as a

result of the killing.

6. Statutory/UPC Approach: A majority of jurisdictions and the UPC have an express

statute providing that a killer shanll not take from his/her victim (UPC 2-803). Under 2-

803, the killer is treated as disclaimed the property (under UPC 2-1106). Most statutes

treat the killer as having predeceased the victim.

a. NOTE: UPC 2-803 also includes that a killer shall not take even nonprobate

property

7. Intentional and Felonious Killing: The general rule (both case law and statute) is: In

order for killing to bar the killer from taking (either outright or through the constructive

trust), the killing must be intentional and felonious.

a. Manslaughter: Must distinguish btwn voluntary and involuntary manslaughter

(see Mahoney above, p. 21).

b. Self-Defense: Killing in self-defense is not felonious and does not trigger the

doctrine

c. Assisted suicide: Mercy killings and assisted suicides technically are intentional

and felonious and come within the scope of the homicide doctrine.

i. However, whether the doctrine SHOULD include such acts is greatly

debated.

8. Burden of Proof: Whether a killer takes from his/her victim is a CIVIL issue, not a

CRIMINAL issue.

a. A criminal conviction has res judicata effect upon the civil issue (UPC 2-803(g))

i. i.e. res judicata = [Latin, A thing adjudged.] A rule that a final judgment

on the merits by a court having jurisdiction is conclusive between the

parties to a suit as to all matters that were litigated or that could have

been litigated in that suit.

b. BUT an acquittal is not the final word because the burden of proof in a criminal

case if proof beyond a reasonable doubt, while the burden of proof in a civil

case is merely preponderance of the evidence. (i.e. the criminal burden is

higher than the civil burden)

Chapter 2 – Intestacy: An Estate Plan By Default 23 of 120

i. E.g. if the defendant is acquitted on homicide charges, but civilly found

liable for the decedent’s intentional and felonious wrongful death (e.g.

voluntary manslaughter?), the killer is still barred from taking.

9. Remedy:

a. General Rule: If the doctrine applies, then treat the killer as if he/she

predeceased the victim

b. UPC Approach: UPC 2-803 says to treat the killer as having disclaimed the

property (this is not exactly the same as predeceasing); though depending on

statute, the effect of UPC and other statutory approaches might be similar

10. Do the Killer’s Issue Get To Take?

a. Jurisdictions are split as to whether the homicide doctrine applies to the killer’s

issue – (who even cares about this)

b. In Cali, the question hinges on whether or not the victim died testate or

intestate

c. UPC Approach: The UPC treats the killer as if he/she disclaimed the property

(UPC 2-803), which ARGUABLY permits the killer’s issue to take the killer’s

share under anti-lapse and the per stirpes/per capita doctrines (if they would

otherwise qualify to take under those doctrines)

11. Scope of Doctrine (Probate and Nonprobate):

a. UPC 2-803 says the homicide doctrine applies to probate AND nonprobate

property

b. Joint Tenancy – If the victim and killer held property together in jt tenancy,

then by operation of law, the joint tenancy is converted into tenancy in

common.

i. The killer keeps his/her interest, and the victim’s interest is distributed

as if the killer predeceased the victim.

c. Other Statutes: Where the statute expressly covers probate property only,

arguably, a constructive trust should be imposed on the nonprobate property

i. This is based on the equitable principle that one should not benefit

from one’s own wrongdoing. The constructive trust would be in favor

of those who would have taken if the killer had predeceased the

victim.

iii. Abandonment/Elder Abuse: Some states have doctrines that bar a would-be taker from

receiving if the taker is guilty of misconduct short of homicide.

1. Some states bar the taker if he/she is guilty of abandonment

2. Some states bar the taker if he/she is guilty of elder abuse

a. Elder abuse involves acts that amount to physical abuse, neglect, or fiduciary

abuse of the decedent while he/she was an elder or dependent adult

3. Some states bar parents from taking from a child decedent if the parent refused to

support the child

4. Some states (including IL) bar inheritance from children or elderly relatives who were

abused by the heir

5. The Chinese System and Other Conduct-Based Restrictions on Inheritance (p. 151)

a. China’s inheritance scheme punishes bad behavior and rewards good behavior

b. Looks at the conduct of those surrounding the decedent to determine who is

worthy to take, and who is not

c. Cons: The approach is highly fact-sensitive, time consuming, and expensive –

this may create a potential/incentive for fraud

d. Pros: It rewards contributions to the decedent’s welfare, even by those outside

the nuclear family, and penalizes neglectful behavior/lack of care.

iv. Disclaimer: Disclaimer occurs when an heir or devisee declines to take the property they are

entitled to (i.e. a testamentary gift requires 1) intent by the decedent; 2) delivery of the gift (e.g.

Chapter 2 – Intestacy: An Estate Plan By Default 24 of 120

by will?); and 3) acceptance. Waiver is the recipient’s right to decline step 3, acceptance of the

gift)

1. The most common motivations for disclaiming: Avoid taxes (e.g. avoiding estate taxes)

and avoiding creditors

2. How are Disclaimers Treated?: General Rule: The disclaiming party is treated as

though he/she predeceased the decedent. The property is distributed to the next

eligible taker under the applicable rules of the state/jurisdiction.

3. Benefits to Disclaiming: Several reasons

a. Redistribute Property: Disclaimers can be used to adjust who takes and how

much they take

i. Example: D dies intestate, survived by a spouse and 2 children. If the

children are both adults, they may disclaim their interests to increase

the share going to the surviving spouse (e.g. if the children have no

surviving issue)

b. Avoid Gift Tax Consequcnes: If one accepts the property and then gives it to

the next taker in line, the gift may be subject to gift taxes. BUT if one simply

disclaims, the next taker in line receives the disclaimed gift, with no taxes.

c. Avoid Creditors: Generally, creditors are entitled to reach any transferable

property that the debtor holds. If an heir/devisee is facing creditors’ claims,

such that any inheritance or devise would, for all practical purposes, go directly

to the creditors, the heir or devisee can elect to disclaim the property in

question to avoid the property going to the creditors. If the taker disclaims, the

legal significance is that the disclaimer is tantamount to rejecting the gift. If the

gift is rejected, then it was never accepted, so the taker never had a property

interest in the property in question. If the taker never held a property interest

in the property, the taker’s creditors never had a right to reach it… (but the plot

thickens…)

i. Federal Government as Creditor: All of the above sounds nice, but if

the GOVERNMENT is your creditor of the disclaimant (e.g. for tax

purposes or under Medicaid), the property disclaimed often is subject

to the claim of the federal gov’t.

ii. It doesn’t matter how you try to work the system, the government will

get its money from you, or whoever you try to pass the property to.

(see Drye, below).

4. Drye v. United States (p. 155)

5. F/P

a. Irma Deliah Drye died intestate, leaving her son, Rohn Drye (“Drye”), as the

sole heir to her $233,000 estate. Before Irma died, Drye ran up a $325,000 tax

bill – the IRS filed tax liens against all of Drye’s property and rights to property

b. To keep his mother’s estate away from the IRS, Drye disclaimed his interest.

This allowed the entire estate to pass to his daughter, Theresa, who was next in

line under the applicable state intestacy statute

c. Theresa used the funds to set up a trust under which appellant was a

beneficiary – under the Trust, the assets were shielded from creditors seeking

to satisfy the debts of the Trust’s beneficiaries (i.e. shielded the estate against

the IRS)

6. I

a. Was Drye’s disclaimer effective to pass the property to his daughter free of the

federal tax lien?

7. R/A/H

a. HELD: No – because Drye was able to determine who would receive the

property (either himself if he chose to accept, or his daughter if he chose to

Chapter 3 – Wills: Capacity and Contests 25 of 120

disclaim), he held “property or a right to property” – therefore, the property

was subject to the IRS lien

8. Disclaimers to Qualify for Medicaid (p. 156) – refer to “Federal Government as

Creditor,” above

9. Scope of Disclaimer Statutes:

a. BE CAREFUL – see if the statute applies to only to Probate property (the

Traditional Approach) or if it also applies to Nonprobate property (the Modern

Trend)

10. Execution Requirements

a. Most disclaimer statutes have technical rules requiring that the party

disclaiming must do so 1) in writing 2) within 9 months of decedent’s death.

v. See also IL Statutes

1. 755 ILCS 5/2-1 to 9 (“Descent and Distribution”)

2. 755 ILCS 5/3-1 to 2 (“Simultaneous Deaths”)

3. 755 ILCS 5/9-1 to 10 (“Letters of Administration”)

4. 755 ILCS 5/13-1 to 5 (“Public Administrators, Guardians and Conservators”)

5. 755 ILCS 5/26-1 to 2 (“Appeals”)

6. 755 ILCS 5/27-1 to 9 (“Miscellaneous”)

7. 755 ILCS 5/30-1 to 2 (“Repeal—Savings Clause—Effective Date”)

8. 755 ILCS 15/0.01 to 1 (“Safety Deposit Box Opening Act”)

Chapter 3 – Wills: Capacity and Contests 1. Mental Capacity

a. The Test of Mental Capacity

i. R.3d of Property: Wills and Other Donative Transfers § 8.1(b): To be competent to make a will,

the testator must be

1. 1) an adult (age 18 or older) and

2. 2) capable of knowing and understanding:

a. The nature and extent of his/her property

b. The natural objects of his/her bounty

c. The disposition that he/she is making of that property, and

d. Of relating these elements to ano another and forming and orderly desire to

dispose of the property

3. 755 ILCS 5/4-1 says anyone 18 yrs+ and of sound mind and memory can write a will.

b. Policy Justifications for Capacity Requirement

i. Public opinion

ii. Assures testators that the intent expressed when they have capacity will be protected from the

risk that they may lose capacity later in life (e.g. memory loss, etc)

iii. Protects family members (who stand to gain from testator’s gifts)

iv. Protects the testator from unscrupulous 3rd

parties who may try to take advantage of a testator

of weakened capacity (again—protecting intent)

v. Etc.

c. Powers of Appointment Exercisable by Will: A power of appointment may be exercised by anyone 18

years of age, UNLESS the will expressly requires someone older. (755 ILCS 5/4-2)

i. Insurance and Death Benefits Payable to Testamentary Trustee: IL Law allows a trustee to be

named as a beneficiary (755 ILCS 5/4-5)

d. Temporal Application of Capacity Rule: The testator must have the requisite capacity at the time he or

she performs a testamentary act (executes or revokes a will)

i. Lucid Moment: If a person who otherwise lacks testamentary capacity executes a will during a

lucid moment, the will is valid even though the testator lacked capacity for some time before

and/or after executing the will

1. Note: The testator’s capacity immediately before and after executing the will is relevant

to the issue of capacity at the moment of execution.

Chapter 3 – Wills: Capacity and Contests 26 of 120

e. Presumption of Capacity: By default, there is a STRONG presumption that the testator has testamentary

capacity.

i. IL Statute assumes testamentary capacity

f. Standing: General Rule: A party has standing to contest the validity of a will, or provision in a will, only if

that party will financially benefit from a successful challenge.

g. Burden of Proof

i. Majority Approach: Once a proponent offers prima facie proof that a will was duly executed, it

creates a rebuttable presumption that the testator had testamentary capacity, and the burden is

on the contestant to prove a lack of testamentary capacity (see Wilson v. Lane, below p. 26; see

also Breeden v. Stone below, p. 29)

ii. Minority Approach: The will proponent bears the burden of proving testamentary capacity. See

In re Estate of Washburn (below, p. 26).

iii. In re Estate of Washburn (p. 159)

iv. F/P

1. Katherine Washburn, testatrix, executed 3 wills.

a. Will #1 (Oct 1986): Made several $1,000 bequests to several named

individuals; provided that her home, her personal effects, and the residue of

her estate should go to her sister, Margaret Fay, or in default of that, to her

niece, Catherine Colonna (petitioner)

b. Will #2 (Mar 1992): Left $1,000 bequests to certain named individuals; $5,000

to the respondent, her caretaker, and companion; and the residue to the

petitioner

c. Will #3 (Apr 1992): Left $5,000 bequests to the petitioner and another

individual and provided that the respondent receive the residue, which

included testatrix’s home and personal estate

2. Petitioner (Colonna) challenged the testatrix’s testamentary capacity. The probate ct

held a hearing on proof of the will in solemn form.

3. The ct found that the testatrix was suffering from Alzheimer’s diseases at the time of

execution of the April 1992 will – she could not recollect the property she wished to

dispose of, nor could she understand its general nature

v. I

1. Did the testatrix have the testamentary capacity to bestow her property by will?

vi. R/A/H

1. HELD: No, the testatrix did not have testamentary capacity

2. RULE: Every person is presumed sane until there is some evidence shown to rebut that

presumption

3. Petitioner offered sufficient evidence to rebut the presumption of capacity. Medical

testimony established Alzheimer’s disease. Further testimony established testatrix’s

confusion, forgetfulness, and a lack of competency at the time in question.

vii. Wilson v. Lane (p. 161)

viii. F/P

1. The will in question distributed Greer’s property equally to 17 beneficiaries, 16 of whom

are blood-relatives to Greer. The only non-relative beneficiary is Lane, who spent much

of her time caring for Greer before her death.

2. Supporting testimony from drafting atty and other friends established that, at the time

the will was signed, Greer was mentally competent, and that she emphatically selected

every beneficiary named.

3. The caveators attempted to show that Greer was eccentric, aged, and peculiar in the

last years of her life.

4. Jasper County Superior Court found that Greer lacked testamentary capacity at the time

she executed her will, BUT the trial ct granted Lane’s motion for judgment

notwithstanding the verdict.

ix. I

Chapter 3 – Wills: Capacity and Contests 27 of 120

1. Did Greer have testamentary capacity?

x. R/A/H

1. HELD: Trial court affirmed

2. RULE: A person is mentally capable to make a will if she “has sufficient intellect to

enable her to have a decided and rational desire as to the disposition of her property.

3. None of the evidence showed that Greer was INCAPABLE of forming a decided and

rational desire as to the disposition of her property. She may have been ‘peculiar,’ or

‘eccentric,’ but that didn’t make her incapable of making a rational decision.

4. NOTE: “Sufficient Intellect” is somewhat of a weak standard to meet

xi. Dissent (Carley)

1. Though he agrees that Greer had capacity, the JURY found that she did not, and the

court should honor that.

2. When the evidence is construed most strongly in support of the jury’s verdict in favor of

the Caveators , it authorized the finding that Ms. Greer did not have sufficient intellect

to enable her to make a decided and rational decision regarding disposition of her

estate.

h. Comparison of Testamentary Capacity to Other Capacity

i. Testamentary Capacity vs. Capacity to Contract: Contractual capacity requirement is higher

than testamentary capacity

1. Rationale: Contractual capacity is concerned with one improvidently disposing of assets

during one’s lifetime. The risk is that the person may become destitute and depend on

the state for support. The state has a legitimate interest in preventing costs of caring

for people who could have cared for themselves. Testamentary capacity, on the other

hand, is concerned with the level of capacity necessary to transfer one’s assets at the

time of death. The state has less of an interest in the possible consequences of

testamentary transfers

2. Appointment of conservator: If a person lacks contractual capacity, a conservator is

appointed to handle the person’s affairs. Because testamentary capacity is lower than

contractual capacity, the mere appointment of a conservator does not mean that the

person necessarily lacks testamentary capacity. More facts would be necessary to

determine if the person lacked testamentary capacity.

ii. Testamentary Capacity vs. Marriage Capacity: Testamentary is HIGHER than the capacity to

marry. The right to marry is a fundamental right – it has special status that limits the state’s

ability to regulate it.

i. Attorney’s Ethical Duty: A lawyer has an ethical duty to assess the capacity of an individual before

drafting his/her will. It is unethical to draft a will for a person who lacks capacity.

i. But the atty is authorized to rely on his/her judgment in determining testamentary capacity

j. Defects in Capacity: Even if a person has general testamentary capacity, a person may suffer from a

defect in capacity that may invalidate all or part of the will. There are 4 possible defects:

i. Insane Delusion

ii. Undue Influence

iii. Fraud

iv. Duress

k. Remedy: If the testator suffers from a defect that causes him or her to dispose of his/her property in a

way that the testator would otherwise not have, the general rule is the court will strike as much of the

will as was caused by the defect.

i. Capacity Thresholds (p. 166)

1. It takes more ‘capacity’ to will than to contract. Policy = protection because a will

cannot be reviewed by the testator once he’s dead

2. Oddly, legal capacity to make a will requires greater mental competence than to marry.

Policy = if you’re happy, you’re happy. State doesn’t care. Plus, marriage is closer to a

fundamental right, so the gov’t doesn’t want to touch it.

2. Insane Delusion

Chapter 3 – Wills: Capacity and Contests 28 of 120

a. Definition: A delusion is a false sense of reality to which a person adheres despite all evidence and reason

to the contrary

i. Distinction from Psychological Delusion: Insane delusion is a LEGAL PRINCIPLE, not a

psychological one.

ii. Distinction from Mistake: A mistake is merely incorrect knowledge; a mistake is correctible if

the testator is told the truth. In contrast, an insane delusion is not correctible, no matter how

much you tell the testator the truth

1. Common Law: Courts do not reform or invalidate wills because of mistake; but they do

invalidate or reform wills or provisions of wills resulting from an insane delusion

2. Minority View (Any Factual Basis Approach): If there is any factual basis at all for

testator’s delusion, it is not deemed insane

a. Policy Note: This approach protects the testator’s intent – if ANY basis at all

exists for the testator’s view, the jury has no leeway to call it an insane

delusion; this honors intent

3. Majority View (Rational Person Test): A delusion is insane even if there is some factual

basis for it if a rational person in the testator’s situation could not have drawn the

conclusion reached by the testator. (see R.3d of Property: Wills and Other Donative

Transfers § 8.1)

a. Policy Note: This does not protect testator’s intent. If the jury thinks the

testator’s belief is too bizarre, the jury can substitute its own belief, which can

lead to reformation of the will.

iii. Traumatic Events: (We didn’t really talk about this, but…) Although not explicitly part of either

the Minority or Majority tests, case law tends to indicate a rule: If a traumatic event in a

person’s life alters how he/she views the world, the contestant has a better chance of convincing

the court that the testator suffered from an insane delusion

b. Causation:

i. Majority (“but for” causation): For insane delusion contests, in most jurisdictions, the

contestant must show that (1) the testator worked under an insane delusion, AND (2) the will (or

some part of it) was a product of the insane delusion. (i.e. the delusion “materially affected the

disposition”)

1. i.e. but for the insane delusion, the testator would not have made the will as it appeared

(would not have disposed of property as he/she did)

2. Breeden represents the majority approach to causation: In Breeden (below p. 29) , the

court upheld the will because Breeden’s insane delusions did not materially affect or

influence the will’s provisions.

ii. Minority Approach (“Might have caused”): This test requires the contestant to prove only that

the insane delusion “might have affected” the disposition of property (lower/easier standard to

meet)

1. In re Honigman’s Will represents the minority approach to causation: A minority of

courts apply a lower standard for causation – the court denied probate to a will on the

ground that its dispositive provisions “might have been caused or affected” by the

testator’s insane delusion. (see Honigman, below p. 29)

c. In re Strittmater (p. 169)

d. F/P

i. Louisa F. Strittmater, decedent, was born in 1896 and lived with her parents until their death in

1928. her admiration and love of her parents persisted after their death until at least 1934.

However, four years later, she wrote derogatory comments about her mother and father in the

margins of books and on photographs of her parents. In the opinion of the only medical witness,

Dr. Sarah D. Smalley, who was decedent’s physician all of her adult life, decedent suffered from

paranoia of the Bleuler type of split personality. In 1925 decedent became a member of the New

Jersey branch of the National Women’s Party and worked for them as a volunteer in the New

York office from 1939 to 1941. During this period she spoke about leaving her estate to The Party

and on October 31, 1944, she executed her last will, putting this intention into effect. A month

Chapter 3 – Wills: Capacity and Contests 29 of 120

later she died. Decedent’s will was denied admission to probate based on the ground that

decedent was insane. The appellants, The National Women’s Party, appeal.

e. I

i. Whether the decedent’s will was the product of her insanity?

f. R/A/H

i. HELD: Yes, the will was the product of her insane delusion. Judgment affirmed.

ii. Decedent’s disease seemed to have become well developed by 1936. She had been a member of

the Women’s Party for eleven years at that time but the evidence doesn’t show that she had

taken great interest in it. The court believed it was her paranoiac condition, especially her insane

delusions about the male, that led her to leave her estate to the National Women’s Party.

iii. The Master who heard the case in court found that the decedent’s comments she had written in

books and on photos of her parents demonstrated “incontrovertibly her morbid aversion to

men” and “feminism to a neurotic extreme.” The court found that characterization was not

strong enough to conclude the decedent suffered from an insane delusion.

g. Breeden v. Stone (p. 171)

h. F/P

i. Testator Spicer Breeden regularly abused alcohol and cocaine and suffered from delusions, mood

swings, and paranoia, including that everyone was spying on him (including his friends, family,

and repairmen working in the area; and as it turns out, one of his friends, Crow, was an FBI

informant. Whodathunkit?). Testator cut off his TV antennae and his cable service because he

thought the FBI could monitor his conduct through the TV screen. After a weekend of drinking

and getting high, while driving 110 MPH, he hit a another car, killing the driver (testator did not

stop, but switched cars and continued to party).

ii. When the police came to question him about the accident (2 days later), he barricaded himself

inside his house, hand-wrote a will that excluded his family (from whom he was estranged) and

left all of his property to his friend Sydney Stone.

iii. Breeden’s family challenged Stone’s inheritance

i. I

i. Did Breeden suffer from a lack of testamentary capacity (insane delusion)?

j. R/A/H

i. HELD: No, at least not enough to defeat the will. Decedent was of sound mind – i.e. the testator

suffered from an insane delusion, but there was not sufficient evidence that it materially affected

the provisions of his will.

ii. RULE:

1. Rational Person Analysis (Majority): Under the majority approach, it is easy to conclude

that a rational person could not hold the delusions/paranoia against the gov’t, and his

friends and family.

2. Any Factual Basis Analysis (Minority): Under the minority approach, it came out in a

later proceeding that one of the testator’s friends may have been an FBI informant

(which would give a factual basis for decedent’s paranoia against the gov’t). But it is

unclear whether the probate court was aware of this or whether it would have found

that fact alone supported testator’s delusions and paranoia.

iii.

k. In re Honigman’s Will (p. 178)

l. F/P

i. The testator, after coming to believe that his wife was having an affair, left his wife only the

minimum necessary to satisfy her statutory forced share, leaving the rest to his brothers and

sisters.

m. I

i. Insane delusion?

n. R/A/H

i. HELD: Yes – the court DENIED probate to the will because its provisions “might have been”

caused or affected by the testator’s insane delusion

Chapter 3 – Wills: Capacity and Contests 30 of 120

1. NOTE: This is a lower standard than in Breeden – under this analysis, Breeden’s will

probably would have been reformed

ii. RULE: “Might have been”

iii. The Honigman court assumed causation by putting the burden on the proponent to show that an

unnatural disposition, which could arguably be caused by the insane delusion, was not in fact a

product of that delusion.

iv. The brothers and sisters failed to meet this burden – therefore, the wife won.

o. Dead Man’s Statutes (p. 179)

i. NOTE: Dead Man’s Statutes are good law only in a minority of states

ii. Definition: Dead Man’s Statutes prohibit an “interested party” in the will from testifying about a

decedent’s oral statement in support of a claim against the decedent’s estate

iii. Policy/Pros: Dead Man’s Statutes are meant to protect the decedent, because the decedent is

dead; he/she cannot refute the testimony of the surviving party

iv. Cons: But, Dead Man’s Statutes effectively bar even potentially legitimate testimony/claims

against the decedent’s estate.

v. Dead Man’s Statutes are Unpopular: Some states have totally repealed Dead Man’s Statutes.

Basically, they’re not good law. But we still need to study them. Word.

3. Undue Influence

a. Introduction/Definition

i. “Undue Influence” is hard to define

ii. Traditional View: Undue Influence = coercion; when the will of the person who becomes

testator is coerced into doing that which he/she does not desire to do, that is undue influence

iii. R.3d Wills §8.3 – Undue Influence, Duress, or Fraud

1. (a) A donative transfer is INVALID to the extent that it was procured by undue influence,

duress, or fraud.

2. (b) A donative transfer is procured by undue influence if the wrongdoer exerted such

influence over the donor that it overcame the donor’s free will and caused the donor to

make a donative transfer that the donor would not otherwise have made….

3. Notes: The wrongdoer need not be present when the will is executed to have exerted

undue influence

4. Policy Notes: The doctrine of undue influence protects against overreaching by a

wrongdoer seeking to take unfair advantage of a donor who is susceptible to such

influence, based on e.g. age, inexperience, dependence, physical/mental weakness, or

other.

iv. How to Prove Undue Influence (Traditional Approach)

1. Direct evidence (yeah right…): Prima facie case

2. Circumstantial Evidence: For circumstantial evidence to raise an inference of undue

influence, the contestant must prove

a. (1) Susceptibility: The donor was susceptible to undue influence;

b. (2) Opportunity: The alleged wrongdoer had an opportunity to exert undue

influence;

c. (3) Motive: The alleged wrongdoer had a motive for exerting undue influence;

and

d. (4) Causation: There was a result appearing to be the effect of the undue

influence – i.e. the undue influence caused the testator to dispose of his/her

property in a way that the testator would not have done otherwise?

e.

v. Burden of Proof: Under this approach, the party challenging the will bears the burden of proof.

vi. Presumptions and Burden Shifting (Majority Approach): Because direct evidence is rare, and

the DEFENDANT is in the best position to present whatever evidence IS available, most

jurisdictions use a burden-shifting approach to undue influence.

Chapter 3 – Wills: Capacity and Contests 31 of 120

1. Presumption of Undue Influence: If the elements of the burden-shifting doctrine are

met, a presumption of undue influence arises, and the burden shifts to the defendant to

rebut the presumption

2. Rule: The presumption of undue influence arises if: (1) There was a confidential

relationship between the defendant and the testator; (2) the defendant receives the

bulk of the testator’s estate; and (3) the testator was of weakened intellect

3. Estate of Lakatosh (Burden Shifting) (p. 182) (Burden Shifting)

4. F/P

a. In March 1988, Roger Jacobs, appellant, befriended Rose Lakatosh, decedent,

who was then in her seventies and living alone. Appellant assisted decedent

around her house and drove her to various appointments.

b. A few months after they met, appellant suggested that decedent give him

power of attorney. On November 11, 1988, decedent executed a power of

attorney as well as a new will that left all but $1,000 of her $268,000 estate to

appellant. Appellant’s second cousin drafted the will, and a tape recording of

the execution ceremony evidenced that while decedent didn’t entirely lack

competence, she had a “weakened intellect.”

c. (We presume that a family member or someone other than Jacobs challenged

the will)

5. I

a. Did the trial court err in finding that decedent’s will should be revoked because

appellant failed to carry his burden of proving the absence of undue influence?

6. R/A/H

a. HELD: No. Order of the trial court is affirmed – appellant unduly influenced

Lakatosh

b. RULE: (See presumption/burden shifting rule above)

c. The three elements of the test for undue influence were easily met.

i. First, the confidential relationship was established by decedent’s

dependency on appellant as well as the power of attorney.

ii. Second, appellant received the bulk of the estate (he got all but

$1,000 of her $268,000 estate).

iii. Third, decedent’s intellect was weakened as evidenced by the fact she

was an elderly woman unable to prevent the consumption of her

assets by the appellant and she was living in filth with her bills unpaid.

– she had trouble remembering things and had no understanding of

her estate or assets

iv. Jacobs was unable to overcome the presumption of undue influence

here.

7. In re Will of Moses (p. 186)

8. F/P

a. Fannie Traylor Moses, decedent, was married three times and each of her

husbands died. (because she’s a man-eater)

b. During her second marriage she became friends with appellant, Clarence

Holland, an attorney, fifteen years her junior. After her third husband died,

appellant became decedent’s lover as well as her attorney. This relationship

continued for several years until decedent died at age 57.

c. Three years before she died, decedent made a will devising almost all of her

property to appellant. The will was drafted by an attorney, Dan Shell, who had

no connection with appellant, and who did not tell appellant about the will.

d. Decedent’s closest relative was her older sister, who attacked the will on the

ground that of undue influence. The Chancellor found undue influence and

denied probate. Appellant appeals.

9. I

Chapter 3 – Wills: Capacity and Contests 32 of 120

a. Is a presumption of undue influence is overcome when independent advice and

counsel is sought?

10. R/A/H

a. HELD: Not necessarily. The decree of the chancery court will be affirmed.

b. RULE: (See presumption/burden shifting rule above)

c. The attorney’s testimony supports the chancellor’s finding that nowhere in the

conversations with the decedent was it at all discussed the proposed

testamentary disposition whereby preference was given to a non-relative to

the exclusion of her blood relatives.

d. There was no discussion of her relationship with appellant, nor was there

discussion as to who her legal heirs might be, nor was there discussion as to

their relationship to her, after it was discovered she had neither a husband nor

children. It’s clear from the testimony that the attorney-draftsman did no more

than write down, according to the forms of law, what decedent told him.

e. There was no meaningful independent advice or counsel touching upon the

area in question.

f. DISSENT (Robertson): The judgment of the lower court should be reversed and

the decedent’s will should be admitted to probate. There is no testimony that

indicates that appellant even knew of decedent’s will, much les participated in

the preparation and execution of it. Furthermore, the evidence is clear that

decedent executed her will after full deliberation, with full knowledge of what

she was doing, and with the independent consent and advice of an experienced

and competent attorney.

g. OTHER NOTES: The sexual morality of the personal relationship between the

decedent and the appellant is not an issue. However, the intimate nature of

this relationship is relevant to the present inquiry to the extent that its

existence, under the circumstances, warranted an inference of undue

influence, extending and augmenting that which flowed from the attorney-

client relationship.

vii. In re Kaufmann’s Will (p. 191)

viii. Significance: A will is invalid where the evidence shows that the testator did not freely and

voluntarily creates his or her will because another individual exerted undue influence over his

mind.

ix. F/P

1. In about 1949, Kaufman met Walter Weiss at age 39. Weiss did not have any significant

material assets at the time. A year later Kaufman hired Weiss as his financial consultant.

About the same time, Kaufman moved in with Weiss. Kaufman remodeled the top floor

of his apartment building into an office for Walter. Weiss completed most of the

domestic duties of the household, answered the mail and telephone, and paid the bills.

Weiss took charge of both of Kaufman’s bank accounts and investments as if they

belonged to both of them. The two lived together until 1959 when Kaufman died

unexpectedly. Robert executed a will before he died leaving substantially all of his

property to Weiss. He also included a letter to his family stating his appreciation for

having Weiss in his life, hoping that his family would be happy for him.

2. At Kaufman’s death, his brother sought to have his will set aside on grounds of undue

influence after suspecting that Kaufman had a homosexual relationship with Weiss. The

trial court found that Weiss made false accusations against Kaufman’s brother and

caused Kaufman to believe that his family was resentful of his drive for independence.

x. I

1. Does a beneficiary exert undue influence over a testator when the testator could easily

have been taken advantage of, the beneficiary dominates the testator and the testator

relies exclusively on the beneficiary’s knowledge in making dispositions?

xi. R/A/H

Chapter 3 – Wills: Capacity and Contests 33 of 120

1. HELD: Yes.

2. RULE: A will is invalid for reason of undue influence where the evidence shows that (1)

the beneficiary dominated the testator, (2) the testator could be easily taken advantage

of, and (3) the testator relies exclusively on the beneficiary’s knowledge and judgment

concerning the disposition of all material things in the testator’s life.

3. Here the proponent of the will planted in the testator’s mind that his family was

obstructing his goal of independence and made false accusations against the testator’s

brother. The testator was inexperienced, and the proponent exerted undue influence

over him when he disposed of almost his entire estate to the proponent.

xii. Lipper v. Weslow (p. 193)

xiii. F/P

1. Sophie Block, decedent, was married three times. From her first marriage she had one

son, Julian Weslow, who died in 1949, and was the father of decedent’s grandchildren,

Julian Weslow Jr., Julia Weslow Fortson, and Alice Weslow Sale, plaintiffs herein.

2. After the death of her first husband, decedent married a Mr. Lipper. Frank Lipper and

Irene Lipper Dover, defendants, are the sons of their marriage (her kids from her 2nd

marriage).

3. After Mr. Lipper’s death, decedent married Max Block and there were no children from

this marriage.

4. On January 30, 1956, decedent, executed a will wherein she left her estate to her two

surviving children, defendants (on the Lipper side). She explicitly disinherited the

Weslow side (the widow of Julian, and also Julian’s children).

5. Decedent’s will was prepared by her son Frank, an attorney, one of the beneficiaries of

the will, and Independent Executor of the will. The will was witnessed by two former

business associates of the decedent.

6. Plaintiffs contested the will, claiming undue influence on the part of Frank (her son, and

the lawyer who prepared the will)and the jury found that decedent’s will was procured

by undue influence on the part of defendant Lipper. The trial court entered judgment on

the verdict and set aside the will.

xiv. I

1. Was there any evidence of undue influence?

xv. R/A/H

1. No. The cause is reversed and rendered for defendants.

2. The will contained a paragraph that stated that the testatrix disinherited the

grandchildren because they and their mother had been “most unfriendly” to her since

the death of her son, though there was conflicting evidence as to the accuracy of this

assertion.

3. Here the will and the circumstances may raise suspicion, but does not give proof of the

vital facts of undue influence. All of the evidence reflected that decedent was of sound

mind, strong will, and in excellent physical condition. Additionally, subsequent to the

execution of the will decedent told three disinterested witnesses what she had done

with her property in her will as well as the reason for it.

4. Decedent’s will did make an unnatural disposition of her property in the sense that it did

prefer her two children over the grandchildren by a deceased son. However, the

decedent had the right to do as she did and the record contains an explanation from the

decedent herself as to why she chose to do such.

5. A person of sound mind has the legal right to dispose of his property as he wishes, with

the burden on those attacking the disposition to prove that it was the product of undue

influence.

xvi. NOTES

1. Presumption Approach: It is unclear whether the presumption of undue influence

would arise. 1) There was a confidential relationship (Frank, the drafter, was the

testator’s son). 2) It is unclear whether he took the “bulk” of the estate (he got half). 3)

Chapter 3 – Wills: Capacity and Contests 34 of 120

it is questionable whether the testatrix was of weakened intellect. She was 81 yrs old,

but she had also indicated in various forms that she intended to disinherit her

grandchildren.

2. The Interested Drafter Approach: The interested drafter doctrine does not apply if the

atty is related to the testator.

xvii. No-Contest Clauses (p. 198)

1. No-Contest Clauses (a.k.a. in terrorem clauses) provide that a beneficiary who contests

the will shall take nothing, or a token amount, instead of taking whatever was provided

for in the will.

a. Public Policy Considerations: The good: No contest clauses may deter

frivolous suits and protect testator’s intent. The bad: No contest clauses may

shield a party’s wrongful conduct

2. Enforceability:

a. Majority/UPC Approach: A majority of jurisdictions and the UPC refuse to

enforce a no-contest clause if there is probable cause to support the will

contest, whatever the nature of the contest. See UPC 2-517 and 3-905.

i. Rationale : If there is probable cause to support the claim, the risk

that the no-contest clause is being used to shield wrongful conduct is

too great to ignore.

b. Minority Approach: The no-contest clause is unenforceable, regardless of the

amount of evidence supporting the claim, if the claim is one of forgery,

revocation, or misconduct by one active in the procurement or execution of the

will.

xviii. Bequests to Attorneys

1. Presumption of Undue Influence (Atty Drafter): Many courts hold a presumption of

undue influence arises when an attorney-drafter receives a legacy, EXCEPT when the

attorney is related to testator.

2. How to beat the presumption: The presumption can be rebutted only by clear and

convincing evidence provided by the attorney

3. Ethical Considerations: an interested drafter may be subject to ethical discipline.

Model Rule 1.8(c) (of Model Rules of Professional Conduct) says: “A lawyer shall not

prepare an instrument giving the lawyer or a person related to the lawyer … any

substantial gift from a client,…except where the client is related to the donee.”

b. Planning for and Avoiding a Will Contest

i. Anticipating Which Grounds A Will Contest Will Be Brought On: 2 common grounds = 1) Lack of

Capacity and 2) Undue Influence

ii. Warning Signs that A Will Contest is Coming: Things to watch out for:

1. Unusual Disposition: (Perhaps the largest warning sign) e.g. the omission of a close

family member or an unexplainable distinction among family members of equal relation

to the testator

2. Deviation from Plan: An eccentric client’s new testamentary scheme departs radically

from previous plans;

3. Blended Family: The testator has multiple or blended families arising from multiple

marriages;

4. Unfavorable Conditions: The testator imposes the sort of conditions on a bequest that

are likely to cause the beneficiary to bristle;

5. Dispositions to Unfavorable Parties: The testator makes a disposition to a mistress or

other person or group unpopular with the testator’s family

iii. Strategies for Avoiding Will Contests (As Testator/Testator’s Lawyer): The lawyer should take

extra precautions to prevent will contests

1. Statement of Reasons: List the reasons for dispositions in the will (this is weak) --

2. Letter to the Lawyer: Have the client (testator) write a letter to the atty, saying in detail

the dispositions the client wants to make. The lawyer should reply and list the

Chapter 3 – Wills: Capacity and Contests 35 of 120

consequences. Having this trail of evidence shows that the testator was of sound mind

in drafting the will, however it turns out.

3. Video Discussion: Between the testator and lawyer, and in front of witnesses, during

which the testator explains why he wants to dispose of the property in the way written

in the will.

4. Family Meeting: Testator explains his rationale to his/her family

5. Professional Examination: Have the client’s level of capacity examined by a

professional; have written documentation of the capacity

6. No-Contest Clause: Attempt to use a no-contest clause

iv. Other Notes:

1. Fact-Intensive Suits: Although an unnatural disposition does not itself constitute lack of

capacity, at a minimum, it opens the will to attack. This forces the estate to defend

against the claim. Juries sometimes substitute their intent for the testator’s intent – this

is bad; it does not protect the testator’s intent.

v. Using Inter Vivos Trusts Instead of Wills: Using an inter vivos trust increases the chances that

the testamentary scheme will survive a challenge.

4. Fraud

a. Definition: Fraud occurs when 1) the testator is deceived by a deliberate misrepresentation and 2) does

that which he would not have done had the misrepresentation not been made.

b. Rule: The misrepresentation must be made with both 1) the intent to deceive the testator and 2) the

purpose of influencing the testamentary disposition.

i. Note: The fraud must cause the testator to dispose of his/her property in a way that he/she

would not have otherwise

c. 2 Kinds of Fraud: In the Inducement, and In the Execution

i. Fraud in the Inducement: Occurs when a person misrepresents a fact to the testator to 1)

induce the testator to execute or revoke a will, or 2) to refrain from executing or revoking a will,

or 3) to include particular provisions in the wrongdoer’s favor.

1. Key: The misrepresentation does not go to the terms of the will per se, but rather

concerns a fact that is important to the testator and may induce the testator to dispose

of his or her property differently in light of the misrepresentation

ii. Fraud in the Execution: Occurs when a person misrepresents the nature of the document the

testator is signing. E.g. A person tricks another into signing a document that is the signer’s will,

but the signer does not realize it; or when the testator realizes he/she is signing his/her will, but

the person misrepresents some portions of the will.

d. Remedy for Fraud: A provision in a will, made because of fraud, is invalid. The remaining portion of the

will stands as enforceable. BUT the ENTIRE WILL IS INVALID IF the fraud permeates the entire will or the

portions invalidated by fraud are inseparable from the rest of the will.

i. Remedy for Fraudulent Provisions: Strike as much of the will as was affected by fraud; if

necessary, strike the whole will

ii. Remedy for Fraudulent Failure to Revoke: If the fraud causes the testator not to revoke a will

(or clause) that he/she otherwise would have, the appropriate remedy is to strike the will (or

clause) that the testator would have revoked but for the fraud.

iii. Remedy for Fraudulent Failure to Execute: If the fraud causes the decedent not to execute a will

that he/she otherwise would have, although the court will not execute the will for the decedent,

the court can impose a constructive trust on the parties who take the decedent’s probate

property and order the property distributed to the parties who would have taken the property

had the decedent executed the will that the misconduct prevented the decedent from executing.

1. Constructive Trust: The practical effect of this remedy is to give effect to the will that

the decedent did not execute. The constructive trust remedy is rare, but courts have

imposed it, where appropriate, to prevent unjust enrichment by those who would

otherwise receive the decedent’s property.

2. Constructive Trust General Rule: For constructive trust to be appropriate, the unjust

enrichment must result from some party’s misconduct.

Chapter 3 – Wills: Capacity and Contests 36 of 120

e. Puckett v. Krida (p. 209)

f. F/P

i. Laverne Krida and Mattie Ruth Reeves, defendants, were the sole beneficiaries in Nancy Porch

Hooper’s will. The defendants were employed to provide care around the clock care for the

Hooper.

ii. The defendants were hired as nurses to provide around the clock care for the testator. After their

employment, the nurses persuaded the testator that her family wanted to put her in a nursing

home and were misappropriating her funds. (This was not true). The defendants were aware

when they began work that the testator’s biggest fear was going to a nursing home. They

separated the testator from all of her family, friends, and former professional contacts. At

different times, the defendants made false statements to the testator and concealed facts from

her. The defendants listened to Hooper’s phone conversations and told her that her niece was

wasting or misappropriating funds and then reimbursing herself for renting fancy cars and airline

expenses.

iii. The testator entrusted her niece Jean Law to handle her financial affairs. The evidence at trial

showed that Law managed her affairs properly, made every effort to keep the testator out of a

nursing home, and never reimbursed herself for any of the expenses. The trial court also found

that the niece kept meticulous records. The defendants however, had irregular dealings with the

testator’s money.

iv. The defendants isolated the testator and controlled access to her. In regards to the testator’s

professional and business relationships, the defendants (1) made new professional relationships

for the testator and eliminated all former associations, (2) made serious decisions about the

testators real property to avoid contact with her previous realtor, and (3) replaced her long time

tenant with one of their family members. Furthermore, the defendant made the testator’s

neighbors feel unwelcome and threatened the family with a lawsuit.

v. The trial court set aside a deed and a will on grounds of fraud and undue influence finding that

the defendants made misrepresentation to Hooper and controlled her access to legal and

financial professionals.

g. I

i. Whether a presumption of fraud and undue influence is raised where the testator is in a

confidential relationship with people who involuntarily isolate the testator and make

misrepresentations on which the testator relied in making her will?

h. R/A/H

i. Yes. There is sufficient evidence here to support a finding of fraud and undue influence because

the defendants, standing to benefit from the will, had exclusive access to the testator and control

over her physical person. Furthermore, the defendants were in a confidential relationship with

the testator because they were employed as nurses to provide the testator with around the clock

care and one of them was her attorney-n-fact. These facts together with evidence showing the

defendants told false statements which they were aware would cause the testator to exclude her

family from the will.

ii. A court will hold that there is sufficient evidence to support a finding of fraud and undue

influence where the defendants controlled the testator’s physical person and told her lies which

they knew would cause her to exclude certain persons from her will.

5. Duress

a. Definition: Duress is overtly coercive undue influence

b. RULE: A donative transfer is procured by duress if the wrongdoer threatened to perform or did perform a

wrongful act that coerced the donor into making a donative transfer that the donor would not otherwise

have made (see R.3d Wills § 8.3(c)

c. Remedy for Transfers Made By Duress: The law INVALIDATES transfers compelled by duress

d. Latham v. Father Divine (p. 210)

e. F/P:

i. Mary Sheldon Lyon left a will to Father Divine, two corporate defendants, and another defendant

who was one of Father Divine’s followers. Plaintiffs claim that on several occasions after the

Chapter 3 – Wills: Capacity and Contests 37 of 120

execution of her will, she expressed a desire and determination to revoke the will and create

another in which plaintiffs would receive a substantial amount of her estate. Shortly before her

death, Lyon had attorneys draft a new will that named the plaintiffs as legatees for a large

amount, approximately $350,000. However the defendant through false representations, undue

influence, and physical force, prevented the testator from executing the will. Furthermore, the

defendants conspired to kill and did kill the testator without the consent of the testator’s

relatives and before she could execute a new will.

f. I

i. Whether a beneficiary may be liable for undue influence and fraud where he prevents a testator

from executing a will that would give a gift to another by making false representations, using

physical force, and murdering the testator.

g. R/A/H

i. HELD: Yes. A beneficiary is liable for undue influence and fraud where he makes

misrepresentations, uses force, and murders a testator to prevent him from signing a will that

names another person as a beneficiary. In such a case, the beneficiary holds the property in

constructive trust for the intended beneficiaries. Anything short of an equitable remedy would

be unjust.

ii. RULE: When an heir or devisee under a will prevents the testator from making a will or deed in

favor of another, by fraud, duress, or undue influence, such heir or devisee will be deemed a

trustee over the gift in favor of the intended beneficiary.

iii. The validity of a will rests upon testamentary intent and meeting the proper staturory

requirements. When the evidence proves an heir through misrepresentations and physical force

prevents a testator from carrying his testamentary desires, the court will impose a constructive

trust to carry out equity. Though the court will enforce the will, valid on its face, it will not allow

an heir to benefit from such wrongful conduct.

6. Tortious Interference With An Expectancy

a. Definition: Tortious Interference w/ and Expectancy occurs when a third party (not the testator) has

intentionally committed tortious conduct in the testamentary process (undue influence, fraud, or duress).

Others who would have taken but for the misconduct can sue the 3rd

party for tortious interference with

an expectancy.

b. RULE: The plaintiff has to prove 1) the existence of an expectancy; 2) a reasonable certainty that the

expectancy would have been realized but for the interference; 3) that the interference was intentional; 4)

that the interference involved conduct tortious in itself, such as fraud, duress, or undue influence; and 5)

damages resulting from the interference.

i. NOTE: The theory of Tortious Interference MAY NOT be used when the challenge is based on the

testator’s mental capacity

c. Schilling v. Herrera (p. 215)

d. F/P

i. Schilling, the decedent’s brother, sued Herrera, the decedent’s caretaker, claiming tortious

interference with an expectancy.

ii. Decedent’s health began to fail, and she executed a Durable Power of Attorney naming her

brother as her attorney-in-fact. Decedent lived in Fla, her bro in NJ. When the decedent was in a

rehabilitation center, Herrera began to care for her, and Herrera continued to care for her when

the decedent was released, visiting her as needed, and then converting her garage into a

bedroom. Decedent moved in with Herrera (to the new garabe-bedroom). The brother helped

pay for the care.

iii. Herrera convinced decedent to execute a new Power of Attorney naming Herrera, as well as a

new will naming Herrera personal representative and sole beneficiary.

iv. When decedent died, Herrera did not inform the decedent’s brother until after she probated the

will and probate had closed. The decedent’s brother claimed that he called Herrera during this

time but that she intentionally refused to answer or return his calls.

v. The trial court dismissed the brother’s claim because 1) Herrera owed the decedent’s brother no

duty, and 2) the brother had failed to exhaust his probate remedies.

Chapter 4 – Wills: Formalities and Forms 38 of 120

vi. .

e. I

i. Did

f. R/A/H

i. Trial Ct reversed & remanded

ii. HELD: The court ruled that the brother DID state a claim because he alleged that Herrera’s

“fraudulent actions” and “undue influence” caused the decedent to change her will, leaving all of

her property to Herrera and revoking a prior will that left her estate to her brother

iii. RULE: To state a cause of action for intentional interference with an expectancy of inheritance,

the complaint must allege: 1) The existence of an expectancy; 2) Intentional Interference with

the expectancy through tortious conduct; 3) causation (i.e. but for the interference, the testator

would have made the disposition differently); and 4) damages.

iv. It is the testator who was defrauded or unduly influenced by the defendant, not the claimant

(brother).

v. The brother satisfied the rule: 1) He expected to inherit the decedent’s estate upon her death; 2)

Herrera intentionally interfered with his expectancy of inheritance by “convincing” the decedent

(while ill and completely dependent on Herrera) to execute a new last will & testament naming

Herrera as the sole beneficiary; and 3) Herrera’s fraudulent actions and undue influence caused

Schilling damage by preventing him from inheriting the estate.

vi. NOTES: The ct of appeals also ruled that while the claim must be brought in probate court if

adequate releief is available in probate court, and exception exists where the circumstances

surrounding the tortious conduct effectively preclude adequate relief in the probate court. Such

circumstansces existed here because of Herrera’s refusal to answer/return the brother’s calls

g. Anna Nicole Smith and the Probate Exception to Federal Jurisdiction (p. 220)

7. See also IL Statutes

a. 755 ILCS 5/8-1 to 2 (“Will Contests”)

Chapter 4 – Wills: Formalities and Forms 1. Execution of Wills

a. The Functions of Will Formalities

i. Evidentiary: The Wills Act formalities serve an evidentiary function by ensuring that the

document offered for probate truly reflects the testator’s last wishes as to who should take his or

her property.

ii. Protective: The Wills Act formalities serve a protective function by making it more difficult for

fraudulent claims to be brought and by protecting testator’s intent as expressed in the properly

executed will.

iii. Ritualistic: The Wills Act formalities serve a ritualistic function by impressing upon the testator

the finality of the act he or she is performing.

iv. Channeling: The cumulative effect of the Wills Act formalities serve a channeling function by

encouraging individuals to consult an attorney to draft and supervise the execution of their wills,

thereby facilitating the probating the will and decreasing administrative costs.

b. Attested Wills

i. The Function of Formalities

1. Baron – Gifts, Bargains, and Form (p. 223)

2. Lindgren – The Fall of Formalism (p. 223)

a. Policy: Formation/Enforcement of a Will requires more formalities than

contracts – this is to protect the testator from the finality of their decisions

i. Wills: Assume that their seriously intended statement about their

property can’t be trusted. They are so weak, old, feeble, and subject

to pressure that they need extraordinary protection from themselves.

Their spoken words mean nothing. Their writings mean nothing

unless they’re witnessed by two people

Chapter 4 – Wills: Formalities and Forms 39 of 120

ii. Right To Contract: People entering into contracts are generally of

sound mind. Others can rely on only spoken statements. This is

because, generally, people entering into contracts are alive, and will

be alive.

3. Gulliver & Tilson – Classification of Gratuitous Transfers (p. 224)

a. (See above about the Functions of Will Formalities. That’s what this article was

about)

4. UPC §2-502 Execution; Witnessed or Notarized Wills; Holographic Wills (p. 227)

a. (a) Witnessed or Notarized Wills: A will must be

i. (1) In writing

ii. (2) signed by the testator, or signed by someone else in the testator’s

conscious presence and by the testator’s direction

iii. (3) either

1. A) signed by at least 2 individuals, each of whom signed

within a reasonable time after witnessing either (i) the signing

of the will (as described in (2)) or (ii) the testator’s

acknowledgement of that signature; OR

2. B) acknowledged by the testator before a notary public or

other individual authorized by law to take acknowledgements

b. (b) Holographic (Hand-Written) Wills: A will that does not comply with

subsection (a) is VALID as a holographic will (whether witnessed or not

witnessed) if the signature and material portions of the

ii. Writing, Signature, and Attestation: Strict Compliance

1. The Basic Formalities of an Attested Will:

a. IL Law (755 ILCS 5/4-3): Requires writing; signed by testator (or by some

person in his presence and by his direction); and attestation in the presence of

the testator by 2 or more credible witnesses.

b. Writing: There must be a writing. Generally, oral wills are not permitted. (This

is the law in all states, and in UPC 5-502(a).

i. A will need not be on paper; all that is required for a “writing” is a

reasonably permanent record of the markings that make up the will.

(see R.3d Wills § 3.1, cmt i)

ii. Video and Electronic Wills:

1. Videotaped Wills: Generally, a videotaped “will” (e.g.

description of dispositions) is not enough to satisfy that the

will is a signed writing. No courts (to date) have upheld

videotaped wills. The typical videotape scenario lacks

ritualistic function, lacks direct evidence as to whether the

person intended the taped statement to constitute his or her

last will/testament, and has the potential for high costs.

a. BUT!!! a video of the execution ceremony MAY BE

ADMISSIBLE to prove due execution.

2. Electronic Wills: Generally, electronic wills do not satisfy the

signed writing requirement of the traditional Wills Act.

a. BUT!!! NV enacted a statute allowing probate of

electronic wills under strict requirements including

(1) there is a single original file and (2) there is some

way to determine whether the file has been altered

b. ALSO!!! An electronic will might be allowed under

the Substantial Compliance or Harmless Error

doctrines (see below, p. 44)

c. Signature by the Testator

i. Signature = anything the testator intends as his/her signature.

Chapter 4 – Wills: Formalities and Forms 40 of 120

ii. There is no requirement that the testator sign his/her full name. BUT

if a person intends to write his/her full signature, and does not

complete it, the general rule (under strict compliance) is that the

partial signature doesn’t qualify as the person’s signature.

iii. Even a mark (e.g. an “X”) counts as a valid signature if the testator

intends for it to be.

iv. Assisted Signatures: Assisted signatures (e.g. A helps T sign his name,

because T’s hand is too shaky, because of Parkinson’s disease or

something) is valid if the testator intends to adopt the document as a

will (see R.3d Wills § 3.1, cmt j)

v. Signatures by Someone Other Than Testator: If someone else

entirely signs at T’s discretion, and in T’s presence, the will is also valid

(R.3d Wills § 3.1, cmt n)

vi. Acknowledgement by the Testator: Under most statutes, the testator

need not sign in front of the witnesses, as long as the testator

acknowledges, in front of both witnesses at the same time, that the

signature already present on the document is the testator’s signature.

vii. Order of Signatures:

1. General Rule (Traditional Approach): In general, the testator

must sign/acknowledge the will before either of the

witnesses do (see R.3d Wills 3.1, cmt m)

2. Modern Trend: A Witness may sign the will before the

testator signs or acknowledges, as long as no one leaves the

room during the execution ceremony

a. Notes on UPC: The Modern Trend does not exactly

line up w/ UPC because UPC expressly requires

witnesses to sign “after” witnessing the testator

perform (UPC 2-502(a)(3)). BUT if the jurisdiction

has also adopted the UPC harmless error doctrine,

then as long as there is clear and convincing

evidence that the decedent intended the document

to be his/her will, then the order of signing is

irrelevant (UPC 2-503)

viii. Writings Added Below the Signature: WRT attested wills, if writing

appears physically below the testator’s signature, 2 variables must be

analyzed: (1) whether the jurisdiction requires the will to be

“subscribed,” i.e. signed at the “foot or end” of the document; and (2)

temporally, when the writing was added.

1. Subscription Required + Writing Added AFTER Will Signed: If

the will was properly executed and subscribed, then the

original will is valid, and only the writing that was added later

is null and void (assuming it does not qualify as a codicil – a

will that merely amends an existing will – see below, p. 46)

2. Subscription Required + Writing Added BEFORE Will Signed:

If a writing is added BELOW where the signature goes, but it

was added before the will was signed, then technically, the

signature is not at the end of the will. The will fails the

“subscription” requirement

a. Strict Compliance: The whole will is invalid, because

the writing below the signature line fails the

subscription requirement

Chapter 4 – Wills: Formalities and Forms 41 of 120

b. Modern Trend: A court might be willing to simply

strike the provision below the signature and hold

that whatever is above the signature is valid.

3. Subscription NOT Required + Writing Added AFTER Will

Signed: The added writing is not considered part of the will.

The will as it existed when it was executed (signed) can be

given effect, but the writing added later cannot be given

effect unless it qualifies as a codicil.

4. Subscription NOT Required + Writing Added BEFORE Will

Signed: The whole will is valid, including the later writing

added below the signature.

d. Attestation by Witnesses: Most jurisdictions require that the testator sign or

acknowledge (that the signature on the document is testator’s) in the presence

of at least 2 witnesses (usually both witnesses must be present with testator at

the same time)

i. The witnesses must sign the will, and in some jurisdictions, the

witnesses must know that what they are signing is the testator’s will.

ii. IL Law (755 ILCS 5/4-6): If any beneficial legacy/interest is given in a

will to a person attesting to its execution (or to his spouse), the

legacy/interst is VOID to him and all beneficiaries claiming under him,

UNLESS the will is otherwise duly attested by a “sufficient # of

witnesses” other than him.

1. Beneficiaries and creditors may also be witnesses (blue sup p.

13)

iii. Delayed Attestation: Recall: If the jurisdiction’s statute Wills Statute

requires witnesses to sign in the testator’s presence, the witnesses

must sign at the same time as the testator, in the testator’s presence.

BUT, if the statute does not expressly require the witnesses to sign in

the testator’s presence, the Modern Trend/UPC allow witnesses to sign

the will later (delayed attestation), even after the death of the

testator, as long as the witnesses sign w/in a “reasonable period of

time” (see UPC 2-503(a)(3)(A)).

1. What is “Reasonable”: Witnesses should sign while their

recollection of the execution ceremony is fresh enough that

they can remember whether the execution ceremony was

valid.

2. eg: In NY, the statute allows 30 day delay; UPC allows

whatever is “reasonable”

e. The Meaning of “Presence” in Will Execution (p. 233)

i. What does “Presence” mean?

ii. Line of Sight Test: Presence means that the testator does not actually

have to see the witnesses sign, but must be able to see them if he/she

looks.

1. Blind Testator Exception: The testator/witnesses pass the

line-of-sight test if the testator would have been able to see

the witnesses sign from where the testator was

standing/sitting if the testator had the power of sight.

iii. Conscious Presence Test: The witness is in the presence of the

testator if the testator, through sight, hearing, or general

consciousness of events, comprehends that the witness is in the act of

signing.

Chapter 4 – Wills: Formalities and Forms 42 of 120

iv. UPC Approach/Modern Trend: UPC 2-502(a) dispenses altogether

with the requirement that the witnesses sign in the testator’s

presence.

1. Under this approach, there is only “one” presence

requirement: that the testator sign or acknowledge the will

in the presence of the witnesses.

c. Strict Compliance

i. Strict Compliance (Common Law Approach): Requires absolute strict compliance with each Wills

Act requirement, no matter how clear the testator’s intent that the document be his/her last

will. If there is ANY deficiency in the execution ceremony, the document is not a valid will. (see

Stevens v. Casdorph, below)

1. In re Groffman (strict compliance) (p. 228)

2. F/P

a. Groffman died 3 yrs after executing a will at the home of his friends, the Blocks.

If held to be validly executed, Groffman’s widow would share the estate with

her daughter from her 1st

marriage and with one of Graffman’s daughters from

his first marriage. But if the will were invalid, his widow would take the entire

estate in intestacy.

b. Groffman’s widow challenged the will.

3. I

a. Was the will validly executed?

4. R/A/H

a. RULE: English Wills Act of 1837 – Will must be 1) in writing; 2) signed at the

foot/end by either (A) the testator or (B) someone else in his presence and by

his direction; 3) the signature/acknowledgement must be in the presence of 2

or more witnesses present at the same time; 4) the witnesses must attest and

must subscribe the will in the presence of the testator

b. HELD: The court refused to admit the will to probate (even though the ct was

perfectly satisfied that the document was intended by the deceased to be

executed as his will)

c. Though Groffman had 2 witnesses (Block & Leigh), Groffman did not

acknowledge his signature to both witnesses at the same time. Instead, Block

attested in Groffman’s presence, but without Leigh present; and then Leigh

attested without Block present.

d. This fails strict compliance

5. Stevens v. Casdorph (strict compliance) (p. 229)

6. F/P

a. The Casdorphs took Homer Haskell to Shawnee Bank in Dunbar, East Virgina to

execute his will. Haskell asked Debra Pauley a bank employee and public notary

to witness the execution. First, Haskell signed the will. Then Pauley took the

will to two other bank employees, Ms. Waldron and Ms. Ginn to sign as

witnesses. Both ladies testified in their depositions they did not see Haskell sign

and Haskell did not go to with Pauley to the ladies’ work areas. Haskell died on

July 28, 1996. Haskell named Paul Casdorph his executor and left the bulk of his

estate to the Casdorphs. The evidence at the trial court found that everyone in

the bank knew why Haskell was there. Furthermore, the court did not find any

evidence of fraud, coercion, or undue influence. The Stevens’, Haskell’s nieces

who would share in his intestate estate, filed an action to set aside the will.

7. I

a. Is a will is valid where 1) the witnesses do not see the testator sign the will or

acknowledge the will, 2) nor does the testator see the witnesses sign the will or

acknowledge their signature, even though the execution takes place a one

Chapter 4 – Wills: Formalities and Forms 43 of 120

location and the two witnesses are informed that the testator is attempting to

execute his will?

8. R/A/H

a. HELD: NO. A will is not valid if the testator did not sign in it or acknowledge his

signature in the presence of two witnesses, who are together, and sign their

name or acknowledge their signature on the will. The law favors intestacy over

intestacy but this Court has held that a valid will must have testamentary intent

and execution in a manner provided by the code, concurrently

b. DISSENT (Workman): The majority elevates form over substance. Courts should

adhere to the underlying purpose of the statue only to prevent substitution or

fraud. Here, strict adherence to formality defeats the purpose of the statue.

The Court in Wade held that a narrow rigid construction of the statute should

not be allowed to stand in the way of justice.

c. The Court will adhere to statute when deciding whether to probate a will.

Though the Court did not find any evidence of fraud or undue influence, the

evidence did not necessarily rule out the possibility of fraud where the testator

was in a wheel chair and escorted to the bank by the beneficiaries in his will.

9. (Interested witnesses omitted from syllabus)

d. Curing Defects in the Execution of Attested Wills

i. Excusing Execution Defects by Ad Hoc Exception

1. Under Traditional Strict Compliance, almost any mistake in execution invalidates the

will. To avoid this harsh result, some courts excuse or correct an obvious execution

2. In re Pavlinko’s Estate (p. 246)

3. F/P

a. Husband and wife, Hellen and Vasil Pavlinko agreed to leave their property to

each other in the event that either of them died. Hellen Pavlinko signed the will

that was created for her husband and Vasil Pavlinko signed the will that was

created for his wife.

b. Hellen’s will was never offered for probate at her death. The will created for

Hellen but signed by her husband was dated March 9, 1949. Vasil Pavlinko died

February 8, 1957 and Hellen Pavlinko died before her husband on October 15,

1951.

c. The lawyer who created the will and his secretary, Miss Zinkam both signed as

witnesses. Miss Zinkam admitted she did not understand the conversation that

took place between the Pavlinko’s and did not speak their language. The

residuary legatee named in the will in question was Helen’s brother.

4. I

a. Whether a will is valid if it is signed by a person other than the indivudal that is

referenced to in the will as being the creator of the will.

5. R/A/H

a. HELD: No.

b. RULE: The Wills Act specifically requires that a will be in writing and signed by

the testator.

c. Here the will was not signed by the person whom the will describes as being

the creator. Even though the facts show that the husband signed his wife’s will

by mistake and both the spouse and the husband intended to leave their

property to each other, the will cannot be probated because it does not meet

the requirements. Such a holding would leave room to fraudulent claims.

d. The court will strictly adhere to staturory will formation reaquirement that the

testator sign his or her own will unless there is evidence to show that someone

else is unjustly enriched by the purported error.

e. DISSENT (Musmanno): The Pavlinkos’ wills were identical. They left each other

their property and had the same residual legatee. The attorney knew the

Chapter 4 – Wills: Formalities and Forms 44 of 120

language the Pavlinko’s spoke in and versed with them in this language. The

intent of the testator must be determined by looking at the four corners of the

will signed. Because the couple both left their residuary estate to the same

person, the will should be admitted for probate. In addition, the residuary

clause can stand on its own so the whole will should not be submitted for

probate.

6. In re Snide (p. 250) (not assigned)

ii. Curative Doctrines: Substantial Compliance and Harmless Error

1. Substantial Compliance: Even if a will is not executed in strict compliance with the

jurisdiction’s Wills Act formalities, the court is empowered to probate the will IF clear

and convincing evidence shows both that (1) the testator intended this document to

constitute his/her last will and testament, and (2) the will “substantially complies” with

the statutory Wills Act formalities. (see UPC 2-503)

a. In re Will of Ranney (p. 253)

b. F/P

i. During the execution of Russell G. Ranney’s will, the two persons

serving as witnesses did not sign the will. Instead, they signed an

affidavit swearing they had previously witnessed Ranney sign his will.

The lawyer executing the ceremony was not aware that the will itself

needed an attestation clause. He believed that the affidavit alone

without the witnesses’ signature on the will was sufficient. The

Appellate court ruled that the affidavit literally satisfied the statutory

requirements.

c. I

i. ISSUE 1: If a statue allows for an affidavit to be submitted along with a

duly executed will, may the will be probated if the will itself is not

signed by the witnesses.

ii. ISSUE 2: Whether a will if not formally executed in compliance with a

statute may be probated if it substantially complies with the statutory

requirements?

d. R/A/H

i. HELD 1: No. The will may not be probated unless it is signed by

attesting witnesses, even though the witnesses may have signed the

affidavit. Affidavits and attestation clauses serve different purposes.

The legislature expressed an intent that the affidavit be submitted

along with a duly executed will. A will without an attestation clause is

not duly executed.

ii. HELD 2: Yes. A will may be probated if the proponents of the

document can prove by clear and convincing evidence that the

testator intended the document to be his will. Statutory formalities

exist for the purposes they serve. The purpose of a signature is to

provide reliable evidence that the will and its terms reflect the

testamentary intent of the deceased.

iii. The court held that under strict compliance, the will was not properly

executed, but the court went on to adopt the substantial compliance

doctrine and remanded the case for further consideration.

iv. RULE: Courts may still probate a will that substantially complies with

the Wills Act formalities

v. Courts will not allow procedural requirements to defeat the purposes

the statutes are intended to fulfill. The purpose of a signature is to

provide reliable evidence of testamentary intent and attestation

clause help to prevent fraud and undue influence. Here, an affidavit is

reliable. It may be admitted to probate without the witness appearing

Chapter 4 – Wills: Formalities and Forms 45 of 120

in court. Because it is sworn testimony, persons are deterred from

committing fraud because of the penalties for lying under oath.

2. UPC §2-503 Harmless Error (p. 258)

a. Although a document or writing added upon a document was not executed in

compliance w/ § 2-502, the document/writing is treated as if it had been

executed in compliance IF the proponent establishes:

i. (1) by clear and convincing evidence that

ii. (2) the decedent intended the document or writing to constitute

1. (i) the decedent’s will,

2. (ii) a partial or complete revocation of the will,

3. (iii) an addition to or alteration of the will, or

4. (iv) a partial or complete revival of his/her formerly revoked

will or of a formerly revoked portion of a will.

b. Note: The Harmless Error rule is also adopted in R.3d Wills § 3.3

c. In re Estate of Hall (p. 259)

d. F/P

i. Testator and his wife (the Halls) went to their attorney’s office,

modified a draft of their joint will, and signed it on the advice of their

atty that it would constitute a valid will until they signed the final

version. The Halls executed the modified draft, the atty signed it

(there were no other witnesses), and the Halls went home, where they

destroyed their original wills.

ii. After the husband died, the wife applied to informally probate the

Joint Will. The husband’s daughters from a previous marriage

objected to the informal probate and requested formal probate of the

Original Will.

e. I

i. Whether a proponent establishes by clear and convincing evidence

that a decedent intended a second will to be his will where he revokes

all prior wills in his second will and told his wife to destroy his first will.

f. R/A/H

i. RULE: For a will to be valid, typically 2 people must witness the

testator signing the will and then sign the will themselves. However,

Montana statute provides that the document may still be treated as if

it had been executed, given certain circumstances. One circumstance

is that the proponent of the document must establish, by clear and

convincing evidence, that the decedent intended the document to be

the decedent’s will.

ii. HELD: Yes – Although the will was not signed in the presence of 2

witnesses, it is still valid under the harmless error doctrine.

iii. A proponent (the wife) establishes by clear and convincing evidence

that decedent intended a document to be his will where he revoked

his first will in second will and instructed his wife to tear up his first

will. Testamentary intent may still exists where there is evidence that

the testator did not give the will to anyone because he said it was not

finished because the testator may have wanted the will to stand until

his attorney provided for a final will

iv. Clear and convincing evidence of testamentary intent there is

evidence that the testator did not want his first will to be valid and

attempted to execute a valid second will. Furthermore, the testator

gained the attorney’s assurance that the second will would be valid as

it was.

2. Revocation of Wills

Chapter 4 – Wills: Formalities and Forms 46 of 120

a. Revocation in IL (755 ILCS 5/4-7): Will may be revoked only by

i. burning/obliterating (by testator, or by someone in his presence and by his direction & consent)

ii. By execution of a later will (codicil) declaring revocation

iii. By later will to the extent that it is inconsistent with the prior will

iv. By execution of an instrument declaring the revocation, and signed/attested according to IL will

formalities

b. Revocation by Writing or Physical Act

i. UPC §2-507 Revocation by Writing or Act (p. 286)

1. Wills are ambulatory documents; i.e. subject to modification or revocation by the

testator during her lifetime.

2. All states permit revocation of a will in one of two ways: (1) by a subsequent writing

executed with testamentary formalities, or (2) by a physical act, such as destroying,

obliterating, or burning the will.

a. Note: In all states, oral declaration that a will is revokes, with nothing more, is

inoperative.

3. If a properly executed will is not revokes in a manner permitted by statute, the will is

admitted to probate.

4. Revocation by Writing: A will, or any part of a will may be revoked by executing a

subsequent will that revokes the previous will expressly or by inconsistency. (UPC 2-

507(a)(1)). The subsequent writing must be executed with Wills Act formalities.

a. Express Revocation: There is a clear and express statement of the intent to

revoke the prior will. A properly executed instrument that does no more than

express the intent to revoke a prior will is a valid will.

b. Revocation By Inconsistency: The subsequent writing does not expressly

revoke the prior will, but makes a complete disposition of the testator’s estate,

as presumptively replacing the prior will and revoking it by inconsistency. (UPC

2-507(a)(1))

c. Will vs Codicil: If the subsequent will completely revokes the prior will (either

expressly or by inconsistency), the subsequent will becomes the testator’s sole

will. BUT If the subsequent will does not make a complete disposition of the

testator’s estate, it is not presumed to revoke the prior will, but is viewed as a

codicil. The property not disposed of under the codicil is disposed of according

to the prior will.

i. Codicil Execution: The codicil IS A WILL – it must be executed with

Wills Act formalities. However, a codicil is a will that merely amends

an existing will, rather than completely replacing it. (UPC 2-507(a)(1)).

ii. Exception – Codicils to Holographic Wills: Handwritten amendments

(interlineations) to a holographic will constitute a valid holographic

codicil, even if the interlineations do not qualify as a valid holographic

will in their own right.

iii. Mixed Wills and Codicils: Holographic codicils to attested wills are

valid, and attested codicils to holographic wills are valid.

iv. Revocation of codicil/will: Revocation of a codicil does not revoke the

underlying will. Revocation of a will revokes all codicils to it.

v.

5. Revocation by Act: A will or any part of a will is revoked by performing a destructive act

on the will, (1) if the testator performed the act with the intent and for the purpose of

revoking the will or part OR (2) if another person performed the act in the testator’s

conscious presence and by the testator’s direction.

a. Traditional Approach: Destructive acts had to actually affect the written

portions of the will (e.g. you had to rip, burn, etc. the actual text of the will)

b. Modern Approach: Destructive Acts need only destroy the document; not

required to affect the text portion (Note: “Revocatory” or Destructive Acts

Chapter 4 – Wills: Formalities and Forms 47 of 120

include: burning, tearing, canceling, obliterating, or destroying the will or any

part of it (not even requiring the destruction to occur to the actual words of the

will)).

6. Writing as Revocation by Act: The act of writing on a will can qualify as revocation by

act (e.g. the act of writing “VOID” across the will qualifies as a destructive (revocatory)

act.

a. Thompson v. Royall (p. 291)

b. F/P

i. Lou Bowen Kroll asked Judge Coulling and H.P. Brittain to bring her will

and codicil to her home. She told them both in the presence of her

attorney to destroy them. Coulling suggested that instead of

destroying the will and codicil, Kroll should retain the will and codicil in

the event that she decided to execute a new will. Coulling wrote on

the back of the manuscript cover to the will the words, “This will null

and void and to be only held by H.P. Brittain instead of being

destroyed as a memorandum for another will if I desire to make the

same. This 19 Sept., 1932.” Crowell then signed the document. The

same was written on the back of the codicil except the name S.M.B.

Coulling was substituted for H.P. Brittain and signed by Kroll.

ii. The trial court admitted the will and codicil and Kroll’s heirs at law

appeal the decision.

c. I

i. .

d. R/A/H

i. HELD: The writing did not qualify as a revocation by writing because

the notation did not qualify as a valid will (not executed properly—i.e.

not attested because not witnessed and not holographic because the

material provisions were not in the testator’s handwriting).

ii. HELD: The writing did not qualify as a revocation by act because the

handwriting did not touch any of the written portions of the will as

required under the traditional common law approach (see above, p.

46)

iii. A will is not revoked where the testator fails to execute another valid

will or attempt in some way to physically obstruct the will by causing

words or marks to come into contact with the will or destroy the will

itself.

iv. No. A will or codicil is not revoked where the testator attaches a paper

to a will writing “this will null and void” because it was not executed

the way a will is required to be executed.

v. No. The will was not revoked because the words “this will null and

void” did not physically come into contact with the words of the

original will, even though the testator wrote words on a separate

paper attached to the will that declared the will null and void.

Furthermore, the words themselves did not constitute a validly

executed will.

vi. Modern Trend Analysis: The writing in Thompson may qualify as a

valid revocation by act because the act arguably affected some portion

of the will (if the manuscript cover were construed to be part of the

will). The writing may also qualify as revocation by writing – it’s

possible to have delayed attestation…

7. Partial Revocation by Physical Act: Jurisdictions are split – some jurisdictions allow

partial revocation by physical act. But, in some staes, a will cannot be revoked in part by

an act of revocation; it must be revoked by subsequent instrument.

Chapter 4 – Wills: Formalities and Forms 48 of 120

a. Rationale for requiring partial revocation by writing: 1) Canceling a gift to one

person necessarily results in someone else taking the gift, and the “new gift”

can only be made by an attested writing. 2) Permitting partial revocation

revocation by physical act offers opportunity for fraud (e.g. the person who

takes the new gift may be the one who made the canceling marks).

b. Modern Trend/UPC Approach: Many states and the UPC allow partial

revocation by physical act (UPC 2-507), but states are split over how to treat

the revoked gift.

i. MAJORITY: Permit the revoked gift to fall to the residuary and to

increase the residuary, but the partial revocation cannot increase a gift

outside of the residuary.

ii. MINORITY: Hold that the revoked gift may pass via intestacy only.

iii. UPC: The will should be given effect as it reads with the partial

revocation by act, regardless of where that means the revoked gift

goes.

c. Partial Revocation in IL: IL does not specify how partial revocation works.

Assume Majority approach?

8. Revocation by Presumption: If a will was last in the testator’s possession and cannot be

found following testator’s death, a rebuttable presumption arises that the testator

revoked the will by act. If the presumption is not overcome, the will is deemed revoked.

If the presumption Is rebutted, the will is deemed “lost,” and extrinsic evidence is

admitted to prove its terms. If the terms are established, the “lost” will is probated

(jurisdictions are split as to what the burden of proof should be: clear and convincing vs.

preponderance of the evidence; almost any evidence is admissible).

a. Rationale for Presumption: Testators know that their will is an important

document. If the testator takes the will home with him/her, the presumption is

that he/she will safeguard it. If the will is not found after the testator’s death,

the more likely explanation is that the testator revoked it by act, rather than

lost it.

b. The presumption is weak: The presumption that the testator revoked the will

is rather weak. If those challenging the will offer a more plausible explanation

for why the will is found, the issue becomes one for the trier of fact.

c. Duplicate Originals: Duplicate originals are multiple originals of the same will;

each one must be properly executed. A photocopy of an executed will is not a

duplicate original—the testator must properly execute each version of a

duplicate original.

i. Revocation by act or by writing: Affirmative evidence that the

testator properly revoked one duplicate original by act or by writing

automatically revokes ALL duplicate originals.

ii. Revocation by Presumption: The jurisdictions are split over whether

the presumption doctrine applies to revoke all duplicate originals if

the one the testator took home is not found, but a different duplicate

original is found.

1. Revokes all duplicate originals: If the presumption doctrine

applies to one duplicate original, it applies to all duplicate

originals. The reason for this is that revocation by

presumption is a subset of revocation by act. Valid

revocation by act revokes all duplicate originals, so valid

revocation by presumption of one duplicate original revokes

all duplicate originals, even if another duplicate original is

found.

2. Does NOT revoke all duplicate originals: The presumption

doctrine does not revoke duplicate original wills unless NONE

Chapter 4 – Wills: Formalities and Forms 49 of 120

of the duplicate originals are found following the testator’s

death. The reason is that revocation by presumption is based

on the assumption that testators who take their wills home

with them take care to safeguard them. But If there is a

duplicate original (say at the atty’s office), the testator is less

likely to safeguard the duplicate original he/she takes home.

If the will the testator took home is not found, it is just as

likely that the testator lost it as that he/she destroyed it with

the intent to revoke.

d. Harrison v. Bird (p. 287)

e. F/P

i. Daisy Virginia Speer executed a will and named Katherine Crapps

Harrison as the main beneficiary of her estate. The original will

remained with her attorney and a duplicate was given to Harrison. A

few years later, Speer called her lawyer and told him that she wanted

to revoke her will. Then the lawyer or his secretary, in each other’s

presence, tore the will, put the torn pieces in an envelope along with a

letter informing Speer that her will had been revoked. Harrison

attempted to probate her copy of Speers will but the trial court did

not admit it to probate. The Circuit Court affirmed the decision

f. I

i. Whether a presumption arises that a testator revoked her will if (1)

she had possession of the will before death, and (2) the will was not

found among her personal effects at her death, even though a

duplicate of the will exists.

g. R/A/H

i. HELD: Yes, the presumption arises, and the presumption of revocation

was NOT rebutted. The evidence provided by Harrison was not

sufficient to rebut the presumption that Ms. Speer destroyed her will

w/ the intent to revoke it. (Trial Ct Affirmed)

ii. RULE: If there is clear and convincing evidence that the testator had

possession of the will before death, but the will is not found among

the testator’s personal effects after death, a presumption arises that

the testator destroyed the will.

iii. RULE: Furthermore, if the testator destroys the copy of the will in her

possession, a presumption arises that she has revoked her will and all

duplicates, even though a duplicate exists that is not in her possession.

This presumption of revocation is rebuttable, and the burden of

rebutting the presumption is on the proponent of the will.

iv. A presumption arises that a testator revoked her will even if another

person has a duplicate copy of the will, if there is evidence she had it

before she died and it is not found among her personal effects at

death. Though a person may have left copies of his or her will with

other people, the law does not require that the testator insure that

each copy is destroyed in order to revoke a will. The law only requires

evidence that the testator revoked the will itself.

3. Revival of Wills

a. IL Law (755 ILCS 5/4-7(c)): A will that is totally revoked cannot be revived other than by (1) its re-

execution or (2) by an instrument declaring the revival that complies with IL will formalities.

i. Revival of Partially Revoked Will: If a will is partially revoked by an instrument, which is itself

revoked, then the revoked part of the will is revived (it takes effect as if there was never a

revocation).

ii. See also IL Statutes

Chapter 4 – Wills: Formalities and Forms 50 of 120

1. 755 ILCS 5/4-1 to 15 (“Wills”)

2. 755 ILCS 5/5-1 to 3 (“Place of Probate of Will or of Administration”)

3. 755 ILCS 5/6-1 to 21 (“Probate of Wills and Issuance of Letters of Office”)

4. 755 ILCS 5/7-1 to 6 (“Probate of Foreign Wills and Estates of Nonresidents”)

5. 755 ILCS 10/0.01 to 10 (“Uniform International Wills Act”)

b. Revocation by Operation of Law: Change in Family Circumstances

i. Divorce: i.e. the testator, married, executes a will, and then gets a divorce

1. IL Approach: Upon divorce, the testator revokes EVERY legacy interest or power of

appointment given to the former spouse; the testator also revokes any fiduciary office

of the testator’s former spouse (755 ILCS 5/4-7(b))

2. Majority State Law Approach: Divorce revokes any provision in the decedent’s will for

the divorced spouse

3. Minority State Law Approach: Divorce revokes the will only if the divorce is

accompanied by a property settlement

4. Note – Revocation Statutes Apply to Wills Only: These revocation statutes ordinarily

apply only to wills, not to life insurance policies, pensions plans or other nonprobate

transfers

5. Modern Trend/UPC Approach: Revocation applies to wills AND other nonprobate

transfers

6. UPC §2-804 Revocation of Probate and Nonprobate Transfers by Divorce; No

Revocation by Other Changes of Circumstances (p. 306)

a. Divorce revokes any revocable will/portion of a will (i)

disposition/appointments of property; (ii) powers of appointment (of the

spouse or any relatives of the spouse); (iii) nominations of the spouse or any

relative of the spouse to serve in any fiduciary/representative capacity; AND

b. Severs property interests (the spouse no longer has a joint tenancy or a right of

survivorship

c. Effect of Revocation: Treat the spouse as though the spouse (and any relatives)

disclaimed all provisions revoked by this section of the UPC

ii. Marriage (spouse not in premarital will): i.e. the testator (unmarried… or married, I suppose)

executes a will, and then gets married

1. Majority Approach/UPC: The spouse is entitled to his/her intestate share, unless it

appears from the will that (1) the omission was intentional or (2) the spouse is provided

for in the will or by a will substitute with the intent that the transfer be in lieu of a

testamentary provision (see also UPC 2-301)

a. Effect = Revocation: In effect, this kind of statute revokes the will to the extent

of the spouse’s intestate share

2. Minority Approach: The pre-marital will is revoked entirely by marriage. The testator

would need to write an entirely new will.

iii. Marriage (spouse IN premarital will): Spouse may be able to take an elective/forced share of

the decedent’s property in separate property states (see below, p. 69)

c. Birth of a Child: i.e. child born after will executed

i. Majority/UPC Approach: Almost all states have pretermitted child statues – these give a child

born after the execution of a parent’s will, and not mentioned in the will, a share in the parent’s

estate (see UPC 2-302)

1. Effect of Statutes: Pretermitted child statutes result in a revocation of a parent’s will to

the extent of the child’s share.

ii. Minority/Common Law Approach: Marriage followed by birth of children revokes a will

executed before marriage (this position is disappearing)

4. Components of a Will

a. Integration of Wills

i. Introduction: Wills are often written on more than one sheet of paper. The threshold issue is:

what constitutes the pages of a will?

Chapter 4 – Wills: Formalities and Forms 51 of 120

ii. Integration: Doctrine under which all papers present at the time of execution, intended to be

part of the will, are integrated into the will. (R.3d Wills 3.5)

b. Republication by Codicil

i. Introduction: Publication of a will is the testator’s statement to the witnesses, by words or by

action, that a document is the testator’s will.

ii. Republication by Codicil: A will is treated as re-executed (re-published) as of the date of the

codicil.

1. RULE: A will is treated as if it were executed when its most recent codicil was executed,

whether or not the codicil expressly republishes the will, UNLESS treating as such would

be inconsistent with the testator’s intent. (R.3d Wills 3.4)

iii. Potential Conflicts: Republication by Codicil can have unintended consequences

1. Example: Testator revokes Will 1 by Will 2, but then executes a Codicil to Will 1. Then

Will 1 is republished, and Will 2 is revoked by implication (squeezed out)

2. Policy: Because of this, the doctrine of Republication by Codicil should be applied only

where updating the will carries out the testator’s intent (i.e. not automatically)

iv. Requires Existing Valid Will: Republication applies only to a prior validly executed will.

1. Compare to Incorporation by Reference (below, p. 51): IBR can apply to incorporate

into a will language or instruments that have never been validly executed.

2. Exception: A few jurisdictions (e.g. NY do not recognize Incorporation by Reference – in

those states, some cts will stretch Repub by Codicil to re-execute and re-publish an

invalid underlying will, but only if that will went through a valid execution ceremony but

is invalid for some other reason

v.

c. Incorporation by Reference

i. Definition: UPC 2-510 Incorporation by Reference: A writing in existence when a will is

executed may be incorporated by reference if the language of the will (1) manifests this intent

and (2) describes the writing sufficiently to allow it to be identified

ii. IL Law (755 ILCS 5/4-9): Unless made by either (1) the testator or (2) someone in the testator’s

presence and by his direction/consent, an (A) addition to a will or (B) an alteration, substitution,

interlineation, or deletion of any part of a will, which does not constitute revocation, has no

effect.

1. Note: Alterations to wills MUST follow IL will formalities.

iii. Clark v. Greenhalge (Incorporation by Reference) (p. 310)

iv. F/P

1. Nesmith executed a will in 1977 that named Frederic T. Greenhalge as the executor. The

will also made Greenhalge her principal beneficiary to all of their tangible personal

property upon her death except the items that she “designated by a memorandum left

by her and known to Greenhalge, as in accordance with her known wishes to be given to

others at her death.” Nesmith kept a plastic covered notebook in her drawer and named

it “List to be given Helen Nesmith 1979.” One of the entries among others read, “Ginny

Clark farm picture hanging over fireplace.” Several times Nesmith told her home care

nurses that the painting should go to Virginia Clark. Nesmith also told Clark that she

wanted her to have the painting.

2. Nesmith executed two codicils to her 1977 will, one on May 30, 1980 and one on

October 23, 1980. The codicils amended certain bequests and deleted others while

ratifying the will. Nesmith also created a document called “MEMORANDUM” with

Greenhalge in 1972 and identified it as a list of items of personal property prepared with

Miss Helen Nesmith upon September 5,1972 for the guidance of myself in the

distribution of personal tangible property.

3. Greenhalge was the executor for Nesmith’s estate. As executor he distributed Nesmith’s

property in accordance with the will as amended, the 1972 memorandum as amended

in 1976, and certain provisions of the notebook. Greenhalge did not give the painting to

Clark because he wanted to keep it. Greenhalge was aware of the notebook and its

Chapter 4 – Wills: Formalities and Forms 52 of 120

contents but made no attempt to determine the validity of the gift of her farm scene

painting to Clark. Nesmith’s notebook was in existence at the time of the execution of

the 1980 codicils. The probate judge ruled that the notebook was a part of the will and

Greenhalge appealed.

v. I

1. May a document be incorporated by reference into a will if the will refers to the

document, even though it may not be in the same form as stated in the will, but serves

the same function as the document stated in the will, and was in existence at the time

the codicils to the will were created?

vi. R/A/H

1. HELD: Yes. A notebook that gives the executor guidance in distributing the testator’s

estate may be incorporated by reference to a will that includes the language of a

memorandum that serves as a guide to the executor on the distribution of her estate.

2. RULE: A document may be incorporated into a will by reference if the will makes

reference to the document, the document was in existence at the time that the will was

created, and is the document is sufficiently identifiable in the will.

3. The cardinal rule in the interpretation of wills is that the intention of the testator shall

prevail.

4. It appears from the language used in her will that Nesmith intended to retain the right

to alter and amend her will without formally amending the will itself.

5. The language in the will, “a memorandum” does not preclude the existence of more

than one memorandum. It also does not preclude the existence of a document in the

form of a notebook from being included in the will. The fact that it was not labeled as

such does not mean that it was not intended by the testator to be an instruction as to

how to distribute her property at death. Since the testator retained the right to amend

and alter her will after execution, the notebook is sufficiently described since it guides

the executor in distributing her estate at death and the notebook was in existence at

death.

6. A document may be incorporated by reference if the will refers to the document and

the document was in existence with codicils were made to the will, even if it did not

exist at the time the original will was created. Also, the item is sufficiently described in

the will even if it is not specifically referred to if it serves the same purpose as is

indicated in the will.

vii. Simon v. Grayson (p. 315)

viii. F/P

1. Grayson executed a will making reference to a document that would be dated March

25, 1932 and would tell his executors how to give away a gift of $4,000. The letter was

found at Grayson’s death but it was dated July 2, 1933. The letter instructed the

executors to give $4,000 to Ester Cohn. Grayson also executed a codicil dated November

25, 1933.

ix. I

1. Whether a document may be incorporated by reference to a will if was created after the

will but before a later codicil was executed.

2. Whether a document is sufficiently identified by a will if it bears a date differenet from

the date that is referenced to in the will

x. R/A/H

1. HELD 1: Yes. A document may be incorporated by reference in the will where it is in

existence at the date of a codicil to the will.

2. HELD 2: Yes. A document written in July of 1933 can be validly incorporated into a will

and codicil with its latest date at November of 1933 since it informs the testator’s

executors how to pay $4,000 and the document tells the executors to give $4,000 to a

beneficiary named Ester Cohn.

Chapter 5 – Construction of Wills 53 of 120

3. RULE: A document may be incorporated into a will that was in existence at the time a

later codicil was made but not in existence at the time the will was written. A will makes

sufficient reference to a document outside of the will to be incorporated by reference

even though the will states the date on the document is different to the date referenced

in the will as long as the document serves the same function as the one stated in the wil.

4. Because a codicil republishes a will, a document meets the time requirements of the

incorporation by reference doctrine if it was in existence at the time the codicil was

written. Furthermore, the document referred to does not have to specifically match the

description in the will, as long as the will describes the substance of the document and

its purpose.

xi. UPC §2-513 Separate Writing Identifying Devise of Certain Types of Tangible Personal Property

1. Introduction: UPC Tangible Property List allows a testator to give away his/her tangible

personal property via a list not executed with Wills Act formalities, even if the list is

created AFTER the will is executed, as long as the will expressly states such intent.

2. Explanation: The doctrine modifies incorporation by reference by waiving the

requirement that the document be in existence at the time the will is executed, as long

as the document only disposes of the testator’s tangible personal property.

3. Johnson v. Johnson (p. 317) (not assigned)

Chapter 5 – Construction of Wills 1. Mistaken or Ambiguous Language in Wills

a. Introduction: Goal of construing wills is to give effect to the testator’s intent (R.3d Wills 10.1)

b. Admissibility of Extrinsic Evidence (Validity vs Construction):

i. Evidence Offered to Prove VALIDITY of Will: Generally admissible

ii. Evidence Offered to Prove CONSTRUCTION (Interpretation) of Will: Generally inadmissible

c. Traditional Approach: No Extrinsic Evidence, No Reformation

i. Majority Approach: 2 rules bar admission of evidence to vary the terms of the will

1. Plain Meaning Rule (aka No Extrinsic Evidence Rule): Extrinsic evidence may be

admitted to resolve some ambiguities, but the evidence cannot be introduced to argue

that a different meaning was intended, apart from the “plain meaning” of the words of

the will

2. No Reformation Rule: Reformation is an equitable remedy that, if applied to a will,

would correct a mistaken term in the will to reflect what the testator intended to say.

The No Reformation Rule bars this.

a. Policy: Not allowing reformation forces the court to interpret the words that

the testator actually used, not the words that the testator is purported to have

intended to use.

ii. Mahoney v. Grainger (Construction of Wills) (p. 336)

iii. F/P

1. Sullivan executed a will and instructed her attorney to leave all of her real and personal

property to about twenty-five first cousins equally. Her prior will left her assets to two

of her first cousins. The will was duly executed. The residuary clause read, “All the rest

and residue of my estate, both real and personal property I give, devise, and bequeath

to my heirs at law living at the time of my decease, absolutely , to be divided among

them equally, share and share alike …” The trial judge ruled that the term heirs at law

only applied to her maternal aunt, Frances Hawkes Greene and not her twenty-five first

cousins. Certain of the first cousins appealed form the decree disimissing the petition

for distribution to them.

2. Issue. Whether extrinsic evidence that a testator intended to dispose property to

beneficiaries not named in the will may be admitted when a beneficiary can be

ascertained from the face of the will?

iv. I

Chapter 5 – Construction of Wills 54 of 120

1. May extrinsic evidence that a testator intended to dispose property to beneficiaries not

named in the will be admitted when a beneficiary can be ascertained from the face of

the will?

v. R/A/H

1. HELD: No.

2. RULE: Extrinsic evidence is not admissible when the beneficiary of a will can be

identified on the face of the will. (No Extrinsic Evidence Rule)

3. The words used in the will, “heirs at law living at the time of my decease” undoubtedly

refer to the testator’s aunt and not her cousins. The testator’ only heir at law was her

aunt. Extrinsic evidence would only be admissible to help to determine the meaning of

testamentary language that its not clear in its application to the facts.

4. The Court does not want to redraft a testator’s will. Therefore, it will not allow extrinsic

evidence to change the plain face meaning of a disposition under a will.

vi. How to Handle Ambiguities (Traditional Approach)

1. Introduction: 2 kinds of ambiguities

2. Patent Ambiguities: Appears on the face of the will.

a. Example: Clause 1 in T’s will leaves the “disposable portion of my estate” to T’s

daughter, A. Clause 2 in T’s will leaves “my entire estate” to T’s daughters, A

and B.

b. RULE: (Note – this is still the Traditional View – for Modern view, see below):

Extrinsic Evidence is not admissible to clarify a patent ambiguity. Court must

use the 4-corners approach, even if as a result, the will/devise fails, and the

property passes by intestacy.

3. Latent Ambiguities: Manifests itself only when the terms of the will are applied to the

testator’s property or designated beneficiaries.

a. Type 1 (Equivocation): Occurs when a will clearly describes a person or thing,

and two or more persons or things exactly fit that description

i. E.g. “To my niece, Alicia,” if the testator has 2 nieces named Alicia

ii. RULE: Extrinsic is admissible (and necessary)

b. Type 2: No person or thing exactly fits the description, but two or more

persons or things partially fit.

i. RULE: Extrinsic evidence is admissible (and necessary)

4. Personal Usage Exception: (A form of latent ambiguity) If the testator has always

referred to a person by a name other than the person’s true name (e.g. nickname), and

the testator uses that name in the will, extrinsic evidence is admissible to show that (1)

the testator always called the person by that name and (2) the person called by the

nickname is the person who is supposed to take the gift

a. The court treats the nickname that the testator gave to the object/person as if

it were the proper name

d. Modern Trend: Correcting Mistakes Without The Power To Reform Wills

i. The Plain Meaning Rule is repudiated: Extrinsic evidence is admissible to establish the

circumstances around the testator at the time he/she executed the will; this helps the ct

determine testator’s intent

ii. How to Handle Ambiguities (Modern Approach)

1. No Distinction Between Patent and Latent Ambiguities: Extrinsic evidence is allowed

to help interpret ambiguities, regardless of form

iii. Reformation Is NOT Allowed, but “correction” is: Courts are allowed to correct mistaken terms

to conform the will to the actual intent of the testator.

iv. Policy Underlying Modern Trend: Allowing evidence makes room for the doctrines of undue

influence, testamentary capacity (including insane delusions), duress, and fraud.

v. Arnheiter v. Arnheiter (Reformation of Will/Correction of Mistakes) (p. 343)

vi. F/P

Chapter 5 – Construction of Wills 55 of 120

1. Guterl executed a will and directed her executrix to, “sell my undivided one-half interest

of premises known as No. 304 Harrison Avenue, Harrison, New Jersey,” and use the

proceeds to establish a trust for each of the decedent’s two nieces. Guterl did not own a

home at 304 Harrison Avenue but at 317 Harrison Avenue.

vii. I

1. Is a testator’s description of her property in a will sufficient where she names the street,

city and state where her house is located but does not identify the correct street

number?

viii. R/A/H

1. HELD: Yes. Here the testator’s disposition is valid because even though the street

number was not correct, she owned one-half one piece of property on the street.

Because she only devised one-half of the property, which was what she owned, the

property is sufficiently identified

2. RULE: “Mere erroneous description does not vitiate.” The court may use evidence to

interpret mistaken terms in the will, to give effect to the testator’s intent. However the

court does not have power to correct or reform a will.

3. The identification of property in a will does not have to be exact, but only needs to

clearly fit the actual property that exists. The disposition will be valid if despite a

description that does not fit the remaining description clearly identifies the property.

e. Openly Reforming Wills for Mistake

i. Introduction: The “No Reformation Rule” is under attack (in Erickson v. Erickson, not assigned,

the court explicitly rejected the rule).

ii. UPC §2-805 Reformation to Correct Mistakes (p. 351)

1. The court may reform the terms of a governing instrument, even if unambiguous, to

conform the terms to the transferor’s intention

2. It must be proved by clear and convincing evidence that the transferor’s intent and the

terms of the governing instrument were affected by a mistake of fact or law, whether in

expression or inducement.

iii. Langbein – Curing Execution Errors and Mistaken Terms in Wills (p. 351)

1. R.3d Wills 12.1: Allows courts to reform mistaken terms in a will

2. Policy Reasons: (1) The rise of the nonprobate system; (2) Experience in other

jurisdictions; (3) growing embarrassment that failure to cure well-proved mistakes

inflicts unjust enrichment; and (4) concern to spare lawyers from needless malpractice

liability.

iv. Fleming v. Morrison (p. 356)

v. F/P

1.

Butterfield created a will that disposed all of his personal and real property to Mary

Fleming. Before Sidney S. Goodridge signed his will as a witness, Butterfield told him

that the will was fake, and that he only created the instrument to undue Fleming to

“sleep” with him. Under statutory law, a will must be attested by three witnesses. Two

other witnesses had already attested the will.

2. The Probate Court admitted the will to probate.

vi. I

1. May extrinsic evidence be admitted to contradict that the will has been signed by

persons signing as witnesses?

vii. R/A/H

1. HELD: Yes. And also, the will was invalid. Extrinsic evidence is admissible to prove that

one witness was not truly signing as a witness to a will. The testator told a witness that

the will was fake and composed for another motive. Because a will in this jurisdiction

requires three witnesses and one of them did not truly believe that the testator was

signing his will, the will is not valid.

Chapter 5 – Construction of Wills 56 of 120

2. RULE: Extrinsic evidence is admissible to contradict the statements in a will that it is a

will, that it had been signed by a person named as the testator, and attested and

subscribed by persons signing as witnesses.

3. The Court admitted the evidence because it related to the belief of one of the

witnesses, not the validity of the provision to the beneficiary. Therefore the will was not

admitted to probate. The issue was not on concerning the testator’s intent to make a

gift to a beneficiary.

2. Death of Beneficiary Before Death of Testator

a. Introduction: If a devisee does not survive the testator, the devise lapses (fails).

b. IL Law (755 ILCS 5/4-11 Legacy to a Deceased Legatee): (See blue sup p. 13)

c. Default rule = All gifts made by will are subject to requirement that the devisee survive the testator,

unless the testator specifies otherwise.

d. Anti-lapse Statutes: Most states have anti-lapse statutes that, under specified circumstances, substitute

another beneficiary for the predeceased devisee (see Antilapse Statutes below, p. 57)

e. Common Law Rules Re Lapsed Devises (Default Rules):

i. Specific or General Devise: (1) If a specific or general devise lapses, the devise falls into the

residue.

ii. Residuary Devise: (1) If a residuary devise lapses, heirs of the testator take by intestacy. (2) If

only a share of the residue lapses (e.g. when one of two residuary devisees predeceases the

testator), at common law, the lapsed residuary share passes by intestacy to the testator’s heirs

rather than to the remaining residuary devisees.

1. Note: This rule—the no residue of a residue rule—is followed in Estate of Russell (see

below, p. 56

2. But note: In most states, this rule has been overturned by statute or judicial decision.

iii. Class Gift: If the devise is to a class of persons, and one member of the class predeceases the

testator, the surviving members of the class divide the gift.

iv. Void Devise: Where a devisee is already dead at the time the will is executed, or the devisee is a

pet or some other ineligible taker, the devise is void.

1. The same default rules govern the disposition of a void devise as govern a lapsed devise.

v. Estate of Russell (p. 359)

vi. F/P

1. Thelma Russell executed a valid holographic will that said in pertinent part, “I leave

everything I own Real & Personal to Chester H. Quinn & Roxy Russell.” Russell also

disposed of jewelry and money to Georgia Nan Russell Hembree. Russell’s heirs offered

extrinsic evidence to prove that Roxy Russell was a dog.

2. The ct ruled that the fact that Roxy Russell was a dog was a latent ambiguity, and

extrinsic evidence was admissible to establish that fact. The trial court held that Russell

intended Quinn to receive all of her personal and real property and that the gift to the

testator’s dog merely reflected a wish that Quinn care for the dog. Hembree appeals the

decision.

vii. I

1. Whether a gift lapses under the anti-lapse statute if the gift is void?

2. Whether extrinsic evidence is admissible because the language of a will could

reasonably signify two or more meanings?

viii. R/A/H

1. RULE: No-Residue of a Residue (see above)

2. HELD: Dogs are not eligible takers, so the gift to Roxy failed. The gift to Roxy was in the

residuary clause; the court applied the “no-residue-of-a-residue” rule, and held that

Roxy’s half fell to intestacy to the testator’s heirs.

3. A gift lapses under an anti-lapse statute if the gift is void. Testamentary gifts to animals

are void. The gift to the dog lapses under the anti-lapse statute.

4. Note: Extrinsic evidence is not admissible to prove the testator’s intent because the

language of the will is not reasonably susceptible to one or more meaning. Here the

Chapter 5 – Construction of Wills 57 of 120

testator left her property to a person and to her dog. The language did not state that

the testator was making a gift to a person for the benefit of the dog. The will on its face

makes a gift to the dog. The language is not precatory in nature. Extrinsic evidence is not

admissible to prove the testator’s intent. BUT it IS admissible to correct the

interpretation of who Roxy was (latent ambiguity)

f. Modern Trend/UPC Approach (Lapsed Devises): As long as any part of the residuary clause is valid, that

part catches whatever part of the residuary clause fails

i. Policy: The Modern Trend reasons that if the testator included a residuary clause, the testator’s

intent was for ALL of the testator’s property to pass via the will, and for nothing to pass through

intestacy. (UPC 2-604; R.3d Wills 5.5, cmt. O)

g. Antilapse Statutes

i. Introduction: Anti-lapse statutes do not prevent lapse; they merely substitute other

beneficiaries (usu. Descendants) for the dead beneficiary if certain requirements are met.

ii. Typical Antilapse Statute: If a devisee is (1) of a specified relationship to the testator and (2)

predeceases the testator, but is survived by descendants who survive the testator, then the

descendants are substituted for the predeceased devisee UNLESS the will expresses contrary

intent

iii. Scope of Anti-Lapse Statutes:

1. Traditional/Majority Approach: Anti-lapse statutes apply to wills only

2. Modern Trend/Minority: Anti-lapse doctrine applies to wills, as well as some will

substitutes—trusts, insurance policies, and POD contracts, but NOT JOINT TENANCIES

iv. Policy/Theory (Presumed Intent): The idea is, for certain predeceasing devisees, the testator

would prefer a substitute gift to the devisee’s descendants rather than for the gift to lapse.

v. Contrary Intent: The presumption of testator’s intent is rebuttable (i.e. the anti-lapse statutes

yield to a contrary expression of the testator’s actual intent). But the will must explicitly provide

for the testator’s intent if it is contrary to the default anti-lapse rules.

1. Words of Survivorship are not (on their own) enough to establish contrary intent: e.g.

words of survivorship, e.g. in a devise to an individual ‘if he survives me,’ or in a devise

to ‘my surviving children’ are NOT a sufficient indication of contrary intent, in the

absence of additional evidence.

a. Policy: By not specifically providing a substitute gift to the devisee’s

descendants, the testator’s intent is deemed to be that the gift should pass

through anti-lapse. (UPC 2-603; R.3d Wills 5.5, cmt h)

vi. UPC §2-605 Antilapse; Deceased Devisee; Class Gifts (p. 365)

1. Devisee must meet the following rules:

a. Devisee must be a grandparent or lineal descendant of a grandparent of the

testator

b. Devisee must either (1) be dead at the time of execution of the will, (2) fail to

survive the testator, or (3) be treated as if he predeceased the testator.

2. If the conditions in step 1 are met, then the issue of the deceased devisee take (inherit)

in place of the deceased devisee, if they meet the following conditions:

a. Issue must survive the testator by 120 hours

3. How much do the issue take (Requisite Degree of kinship)?

a. Issue take equally if they are all of the same degree of kinship to the devisee

b. If they are of unequal degree to the devisee, then those of more remote

degree take by representation.

4. One who would have been a devisee under a class gift if he had survived the testator is

treated as a devisee for the purposes of this section, whether his death occurred before

or after the execution of the will.

vii. Enforcement of Anti-lapse Statutes

1. Common Law: The anti-lapse doctrine applied only to lapsed gifts, not to void gifts

Chapter 5 – Construction of Wills 58 of 120

2. Modern Trend/UPC: Anti-lapse doctrine applies to any qualifying beneficiary who

predeceases the testator, regardless of whether the beneficiary dies before or after

execution of the will (see UPC 2-605)

viii. Ruotolo v. Tietjen (Anti-Lapse Statute) (p. 367)

ix. F/P

1. Testator devised half of his property to “Hazel Brennan of …, if she survives me.” Hazel

was his stepdaughter, a beneficiary covered by the state anti-lapse statute. She died 17

days before the testator, survived by a daughter.

x. I

1. Did the language “if she survives me” constitute an express contrary intent to the

application of the anti-lapse statute?

xi. R/A/H

1. HELD: Standing alone, the bare words-of-survivorship language (“if she survives me”)

was NOT enough to establish contrary intent

2. RULE: UPC approach (i.e. anti-lapse statute applies here) – see UPC 2-605 above

3. The anti-lapse was adopted to overcome the harsh effects of lapse and therefore should

be applied broadly and liberally, placing the burden of proving contrary intent on those

who oppose the application (i.e. the drafters of the will).

4. Such language is merely boilerplate in many wills; there was no express gift-over in the

event the beneficiary predeceased.

xii. Words of Purchase, Words of Limitation, and the Meaning of “And” vs “Or” (p. 374)

3. Changes in Property After Execution of a Will

a. Introduction: Scenario = A will includes a specific devise of an item of property, but the testator sells or

gives the item away before dying. The question is, what, if anything, should the beneficiary take?

b. Types of Devises:

i. Specific Devise: A disposition of a specific item of the testator’s property

1. Eg: “My 3-carat diamond ring given to me by my Aunt Jane”; or “Blackacre farm”

ii. General Devise: Testator intends to confer a general benefit and not give a particular asset

1. Eg: A legacy of $100,000 to A. If there is not $100,000 in cash in the testator’s estate at

death, the legacy is not adeemed (presumed revoked) – other property must be sold to

satisfy A’s general legacy

iii. Demonstrative Devise: Hybrid—a general devise, yet payable from a specific source

1. Eg: to B, “The sum of $100,000 to be paid from the proceeds of sale of my Apple stock.”

If the testator owns sufficient Apple stock at death, the executor must sell the stock to

get the money to give to B. But if testator doesn’t have $100,000 worth of Apple stock,

the devise is not adeemed; other property must be sold to raise the full $100,000.

iv. Residuary Devise: Conveys the portion of the testator’s estate not otherwise effectively devised

by other parts of the will.

1. Eg: A devise to A of “all the rest, residue, and remainder of my property and estate.”

c. Ademption by Extinction

i. Specific devises of real property are subject to the doctrine of ademption by extinction.

ii. Ademption by extinction applies ONLY to specific devises, not to general, residuary, or

demonstrative devises.

iii. Identity Theory: If a specifically devised item is not [found or identified as being] in the

testator’s estate, the gift is extinguished (the item is adeemed). The court will NOT take any

extrinsic evidence as to why the item cannot be found.

1. Exceptions:

iv. Intent Theory: If the specifically devised item is not in the testator’s estate, the beneficiary my

nonetheless be entitled to the replacement for, or cash value of, the original item—if the

beneficiary can show that this is what the testator would have wanted. Otherwise, the item is

adeemed.

v. UPC §2-606 Nonademption of Specific Devises; Unpaid Proceeds of Sale, Condemnation, or

Insurance; Sale by Conservator or Agent (p. 387)

Chapter 6 – Nonprobate Transfers and Planning for Incapacity

59 of 120

1. A specific devisee has a right to the specifically devised property in the testator’s estate

at death (etc) (UPC 2-606) – see CB p. 387)

2. If specifically devised property is sold or mortgaged (e.g.), then the specific devisee has

the right to a general money devise equal to the net sail price, the amount of unpaid

loan, condemnation award, or basically, whatever.

d. Stock Splits and the Problem of Increase (CB p. 390)

e. Satisfaction of General Pecuniary Bequests

i. Satisfaction (Ademption by Satisfaction) applies when the testator makes a transfer to a devisee

after executing the will (CB p. 390)

f. Exoneration of Liens (CB p. 391)

g. Abatement

i. Intro: If testator gives away more in his/her will than he/she has to give, the doctrine of

abatement provides for which gifts are to be reduced first

ii. General Approach: Based on presumed testator’s intent, residuary clause is reduced first, then

general gifts, and specific gifts last

1. Criticism: Reducing the residuary gift first arguably is against the testator’s intent.

Often the testator presumes that the residuary clause will be the biggest gift and saves

it for the most important beneficiary. If testator is married, usually residue is left to

surviving spouse.

iii. Minority/UPC: Adopt the general approach but also include a provision giving the courts

flexibility to alter the order of abatement where it appears inconsistent w/ the testator’s overall

testamentary wishes. (UPC 3-902)

iv. IL Law (755 ILCS 5/24-3): Abatement = Specific legacies pro rata, then general legacies pro rata

Chapter 6 – Nonprobate Transfers and Planning for Incapacity 1. Introduction to Will Substitutes

a. Langbein – The Nonprobate Revolution and the Future of the Law of Succession (p. 394)

i. Will substitutes do not pass through the probate system. They are the core of the nonprobate

system

ii. The Will Substitutes (4 Main Substitutes)

1. Pure Will Substitutes: When pure will substitutes are created, they are functionally

indistinguishable from a will. Each reserves to the owner complete lifetime dominion,

including the power to name and change beneficiaries until death.

a. Life Insurance

b. Pension Accounts

c. Bank, Brokerage, and Mutual Fund Accounts

d. The Revocable Inter Vivos Trust

2. Imperfect Will Substitutes: Also serve to transfer property at death without probate;

(but they are functionally distinguishable from wills?)

a. Common-law Joint Tenancy

b. Joint tenancies in real estate

c. Joint tenancies in securities

iii. The Hidden Causes of the Nonprobate Revolution

1. Primary Cause = avoidance of probate process

iv. Traditional/Common Law Approach to Will Substitute Formalities: Because will substitutes are

governed by other areas of law (e.g. contract law, property law, etc), the rules and doctrines of

those areas of law control (and not the law of wills)

v. Modern Approach to Will Substitute Formalities: Modern trend is to subject the will substitutes

to wills-related construction doctrines.

1. Policy: People use will substitutes to avoid the costs, hassles, and delays of probate, not

to avoid the wills-related formalities. The wills formalities are better suited than other

Chapter 6 – Nonprobate Transfers and Planning for Incapacity

60 of 120

to handle issues that arise as far as creating a document that purports to transfer

property at the time of death.

2. Will Substitutes and the Wills Act

a. Revocable Trusts/Inter-Vivos Trusts

i. Introduction: A trust is nothing more than another way to make a gift. (Conceptually, the key to

understanding a trust is to remember that it is a bifurcated gift – see Trusts in Chapter 8, starting

on p. 69)

ii. Parties to a Trust: There are 3 parties to a trust: A Settlor (a.k.a. grantor); a trustee (or

numrerous trustees?); and the beneficiaries.

1. Title to the Property is Bifurcated: Legal title is given to the trustee, who holds and

administers the property for the benefit of the beneficiaries. Equitable title is given to

the beneficiaries. The donor is called the settlor.

2. RULE: The trustee can be one of the beneficiaries, but the same person cannot be the

sole trustee and also the sole beneficiary. Policy =If the sole trustee were also the sole

beneficiary, then the trustee would owe no duties to anyone but himself.

iii. Inter Vivos vs Testamentary Trusts: Inter Vivos Trust = The trust is created during the settlor’s

life. Testamentary Trust = the trust is created by will (see 755 ILCS 5/4-4)

1. Inter Vivos Trusts May be Revocable or Irrevocable: That’s all

2. A Testamentary Trust is Always Irrevocable: Nuff said

3. A Revocable Trust is Always Inter Vivos: This is because, for a trust to be revoked, the

grantor must be alive.

iv. Creation of Revocable Trusts:

1. Via Deed of Trust: Settlor transfers the property to be held in trust to the trustee.

When the settlor dies, the trust property is then distributed or held in further trust,

depending on the terms of the instrument.

a. Traditional Approach: A revocable trust created by deed of trust requires with

Wills Act Formalities, or else it is ineffective.

b. Modern Approach: All jurisdictions allow a revocable trust created by deed of

trust to create a nonprobate transfer on death. (see R.3d Trusts §25)

2. Via Declaration of Trust: Settlor declares himself to be the trustee of certain property

for the benefit of himself during his life, with the remainder to pass to others at his

death. While alive, the settlor may revoke the trust and the right to the trust income;

and as trustee

3. Farkas v. Williams (Validity of Inter Vivos Trust Created by Declaration) (p. 398)

4. F/P

a. Farkas purchased stock on four different occasions, each time taking title in his

name “as trustee for Richard J. Williams.” Concurrently with each purchase,

Farkas signed four declarations of trust, where he conveyed himself the life

interest, remaninder to Williams, and retained power to revoke by selling the

stock.

b. Farkas died intestate. His heirs claimed the inter vivos trusts were invalid

testamentary dispositions that failed to comply with Wills Act formalities.

5. I

a. Should the creation of trust by declaration, that doesn’t comply with Wills Act

formalities, be held valid?

6. R/A/H

a. HELD: Yes – the trust is valid

b. It appeared as though Farkas did intend to give Williams an interest in the

property – 1) he set himself up as trustee of the stock for the benefit of

Williams.; The stock was registered in his name as trustee for Williams. 2) He

manifested an intention to bind himself to having his property pass upon his

death to Williams, unless he changed the beneficiary or revoked the trust.

Chapter 6 – Nonprobate Transfers and Planning for Incapacity

61 of 120

c. Some interest passed inter vivos to Williams even though the trusts were

revocable, and in the alternative, the process Farkas went through to create

the inter vivos trusts adequately served the functions underlying the Wills Act

formalities.

7. Theoretical/Policy Perspectives on Inter Vivos Trust Creation (Wills Formalities

Required?):

a. Classic Trust: Assume the settlor, trustee, and beneficiaries are all different

people. Also assume the trust is irrevocable. Then the settlor must transfer

title to the trust inter vivos (during iife). The trust then holds legal title to the

property, not the settlor.

v. UTC §603 Settlor’s Powers; Powers of Withdrawal (p. 403)

1. (a) While a trust is revocable (and the settlor has the capacity to revoke the trust), rights

of the beneficiaries are subject to the control of, and the duties of the trustee are owed

exclusively to, the settlor.

2. (b) During the period the power may be exercised, the holder of a power of withdrawal

has the rights of a settlor of a revocable trust under this section to the extent of the

property subject to the power.

3. Standing

a. Linthicum v. Rudi (Beneficiaries’ Standing To Challenge Revocable Trust

Amendments During Settlor’s Life) (p. 403)

b. F/P

i. The settlor’s revocable trust named her brother and sister-in-law as

successor trustees on her death or incapacity and also made them the

primary beneficiaries on her death. The settlor then amended the

trust naming her late husband’s nephew as successor trustee and sole

beneficiary. Brother and sister-in-law sued to set aside the

amendment on grounds of lack of capacity and undue influence.

c. I

i. Issue: whether revocable trust beneficiaries have the right (standing)

to challenge amendments to trust, when made by settlor during

settlor’s lifetime.

d. R/A/H

i. HELD: Beneficiaries of a revocable trust have no standing to contest

amendments to the trust made during the settlor’s life

ii. At best, the beneficiary’s interest is contingent and unenforceable

during the settlor’s lifetime. The beneficiaries’ interest in the trust

property vests only at the settlor’s death.

vi. Revocability:

1. Traditional/Majority Approach: Inter vivos trusts are presumed to be irrevocable

unless the terms of the trust expressly state that the trust is revocable

a. Policy: Gifts are irrevocable, and because trusts are simply another way of

making a figt, trusts should be irrevocable unless expressly state otherwise.

2. UTC/Modern Trend: (Reverses the traditional approach) A settlor may revoke or amend

an inter vivos trust, unless the trust expressly states that it is irrevocable

a. Policy: UTC approach is arguably more consistent w/ a typical settlor’s intent;

but the traditional rule is true to the origin of trust law—the law of gifts.

vii. Revocation by Particular Method Expressed in Trust Instrument: When a trust sets forth an

express, particular method of revocation, ONLY that method of revocation is valid

1. Policy: The settlor’s intent controls. If the settlor sets forth a particular method of

revoking a trust, it is presumed that the settlor intends that to be the only way to revoke

the trust.

Chapter 6 – Nonprobate Transfers and Planning for Incapacity

62 of 120

2. Subsidiary Law of Wills: The revocable trust is the most will-like of all will substitutes.

Courts have applied subsidiary rules from the law of wills, such as abatement or

ademption.

a. Abatement: Where there is not enough trust property to satisfy the provisions

calling for distribution on death of the settlor (abatement).

b. Ademption: the trust does not include a specific item of property that is to be

distributed to a particular beneficiary (ademption)

3. Traditional Approach:

a. In re Estate and Trust of Pilafas (Revocation of Trust by Express Method) (p.

414)

b. F/P

i. Pilafas created a trust in which eight nonprofit organizations were to

receive a portion of the trust property upon his death. The remaining

portion was to go to his wife and other stated relatives. He amended

the trust twice and simultaneously executed a will at the same time

that he amended the second trust.

ii. The attorney did not retain the originals of Pilafas’s will but to the best

of his knowledge, gave the originals of the trust agreement, the

amendments and the will to Pilafas.

iii. Pilafas kept meticulous records but his will and the trust with

amendments could not be found among his personal things at death.

Pilafas had a room filled with important documents including

photographs and old divorce papers. After Pilafas died, his son sought

a determination that Pilafas revoked his trust agreement and will.

iv. The trial court held that Pilafas had revoked his will and trust. The

remainder beneficiaries appealed the decision of the trial court that

Pilafas revoked his inter vivos trust and will and died intestate.

c. I

i. Because the will and trust documents were last known to be in

decedent’s possession, but the documents could not be found at his

home, would a presumption of revocation apply to one or both

documents?

d. R/A/H

i. HELD: The inter vivos trust was not revoked, and remained valid.

ii. RULE: A will is presumably revoked if it was in the testator’s

possession, but not found among his personal effects at his death.

iii. RULE: If a settlor reserves the power to revoke a trust in a particular

manner and under certain circumstances, he may only revoke the

trust in the stated manner and under the same circumstances as

stated in the trust.

iv. The trust instrument expressly specified the manner in which

decedent could revoke his trust (only through a document in writing

given to the trustee). Because the decedent did not issue a document

to the trustee, which would revoke/amend the trust, the trust is still

valid.

4. UTC Approach/Modern Trend: UTC provides that where the trust sets forth a particular

method of revocation, it should NOT be construed as the EXCLUSIVE method UNLESS

the trust provision EXPRESSLY makes it exclusive. Substantial compliance with the

particular method of revocation is sufficient. (see UTC 602(c)(1) – (2)). A will executed

after the trust, which specifically refers to the trust or the power to revoke, can revoke a

revocable trust IF the trust terms do not specify an exclusive method of revocation AND

if the will is not revoked. (UTC 602(c)(1)).

Chapter 6 – Nonprobate Transfers and Planning for Incapacity

63 of 120

viii. Revocation – No Particular Method Expressed in Trust Instrument: If the trust is revocable, but

silent as to the method of revocation, the power may be exercised in any manner that

adequately expresses the intent to revoke. The trust can be revoked (1) by writing (even if the

writing does not qualify as a will), (2) by act (destructive act + intent), (3) by presumption

(arguably), and even (4) orally (UNLESS real property is involved).

1. Divorce:

a. Traditional Approach: An inter vivos revocable trust is not revoked by divorce,

whilc a will is automatically revoked by operation of law (e.g. divorce).

b. Modern Approach: Treat inter vivos revocable trusts like wills, and apply the

revocation by operation of law doctrine and automatically revoke the

provisions in the trust that favor the ex-spouse.

c. Note: The UTC seems to have implicitly adopted the modern trend through its

rules of construction.

ix. Rights of Creditors of the Settlor: General Rule = one should not be able to shield one’s assets

from one’s creditors. If one has a property interest in a trust, one’s creditors should be able to

reach that property interest. (Note: The rights of creditors of beneficiaries other than the settlor

are different, and are covered in Chapter 8 – Trusts).

1. State Street Bank and Trust Co. v. Reiser (p. 416)

2. F/P

a. Dunnbeier created a revocable inter vivos trust and reserved the power to

amend and revoke, and to direct the disposition of principal and income during

his lifetime. He transferred to the trust the stock of 5 closely held corporations.

After that, Dunnebier obtained an unsecured loan for $75,000. During the loan

application process, Dunnebier represented to the bank that he held

controlling interest in the 5 closely held corps. (The court found that he was

not fraudulent in this representation).

b. Shortly afterward, Dunnebier died. His probate assets wer insufficient to pay

his creditors. The bank sued to reach the assets in the revocable inter vivos

trust.

3. I

a. Whether a creditor may reach the assets of a deceased settlor’s trust if he

created a trust during his lifetime and reserved the right to amend and revoke,

or direct payments to himself, even though the trust has living beneficiaries.

4. R/A/H

a. HELD: To the extent that Dunnebier had power over the assets in the

revocable inter vivos trust during life time, those assets should be available to

creditors following his death.

i. The court required the creditors to exhaust the decedent’s probate

assets first.

b. RULE: Creditors may reach a deceased debtor’s trust income if he created a

trust during his lifetime and reserved the right to amend and revoke, or to

direct disposition of principal and income. The creditors may recover an

amount that is not satisfied by the estate, and not greater than that which the

settlor could have used for his own benefit during his lifetime.

c. A creditor may reach the property of a settlro’s trust if he reserved the rights to

amend and revoke, or direct payment to himself during his lifetime. It violates

public policy for an individual to have an estate to live on but not an estate to

pay his debts with. The creditors may reach the assets of the trust to the extent

that the debt is not satisfied by the estate. The creditors may not reach any

amount that the settlor could not have used for his personal benefit during his

lifetime.

Chapter 6 – Nonprobate Transfers and Planning for Incapacity

64 of 120

d. Where there is evidence that a settlor creates a trust that is not solely intended

for a purpose other than the settlor’s own gain, the court will not allow that

trust to be used to avoid paying a debt.

5. Notes on Creditors’ Rights to Non-Trust Nonprobate Assets:

a. Creditors of Joint Tenants: Cannot reach the jointly held property after the

joint tenant’s death because the decedent’s interest is treated as having

vanished.

b. Creditors of Life Insurance Policy Holders: Usually cannot reach the proceeds

if they are payable to a spouse or child.

c. Creditors of Retirement Account Holders: Same as for Life Insurance.

b. Payable on Death Contracts and Other Nonprobate Transfers

i. Life Insurance – Introduction: POD contracts (most commonly life insurance) are contracts by

which the insured pays into a policy, and the insurance company agrees to pay the benefits

under the policy to a beneficiary designated in the policy if the insured dies while covered. Life

Insurance Policies

ii. Creation (Wills Act Formalities Required?): POD contracts essentially transfer property from the

transferor (decedent) to a beneficiary. So they’re “like” wills. Life insurance policies need not be

created with Wills Act Formalities.

1. Common Law: Under common law, life insurance was the ONLY POD contract that

could be created w/o Wills Act formalities.

a. In re Estate of Atkinson (Pay-On-Death Contracts – Common Law Approach)

(p. 407)

b. F/P

i. Decedent made a deposit in a local bank and took the certificate of

deposit as follows: “Walter S. Atkinson, P.O.D. Mrs. Patricia Burgeois.”

He created 2 other accts w/ the same language, but designating two

other P.O.D. beneficiaries.

ii. After his death, his wife claimed her statutory share of his probate

estate and argued that the monies in all three accounts should be

included in his probate estate as invalid attempts at non

c. I

i. Are the P.O.D. clauses on the C.O.D’s valid nonprobate transfers?

d. R/A/H

i. HELD: The monies in the accounts were invalid attempts at

nonprobate transfers. (i.e. this conforms to the common law – non-

life insurance POD contracts are invalid transfers).

ii. Even though the contracts clearly intended to transfer money to the

named beneficiaries, the state did not recognize such POD accounts as

valid nonprobate transfers.

2. Modern Trend/UPC Approach: Any & all POD contracts/instruments may be treated as

will substitutes

a. Estate of Hillowitz (Pay-on-Death Contracts – Modern Approach) (p. 409)

b. F/P

i. Hillowitz was a partner in an investment club. A provisioning in the

partnership agreement transferred his partnership interest to his wife

in the event of his death. The executors of Hillowitz’s estate

challenged the agreement claiming that it was an invalid testamentary

disposition. The District Court held that the agreement was valid but

the Appellate reversed. Hillowitz’s widow appealed.

c. I

i. Whether a partnership agreement that transfers a partner’s interest

to a beneficiary upon his death is valid even though it does not comply

with the statute of wills.

Chapter 6 – Nonprobate Transfers and Planning for Incapacity

65 of 120

d. R/A/H

i. HELD: Yes. Partnerships are third party beneficiary contracts. They are

similar to other instruments which are contractual in nature but need

not conform to the statute of wills to dispose of gifts at death.

Examples of such agreements include (1) a contract to make a will, (2)

an inter vivos trust in which the settler reserves a life estate, and (3)

an in insurance policy.

ii. RULE: A partnership agreement that provides for a payment of the

partner’s interest to a beneficiary upon his death is valid even though

it does not comply with the statue of wills.

iii. The courts will enforce agreements where the agreement is not solely

testamentary. The Court is likely to enforce contractual agreements

where the parties undertake a business venture separate from the

testamentary disposition, and beneficiaries have reason to expect a

gift.

3. Revocability: The transferor who creates the POD clause is presumed to have the right

to cancel or change the POD clause (i.e. Beneficiaries of a POD clause do not receive an

irrevocable property interest inter vivos).

4. Construction (under the Law of Wills): The modern trend/UPC approach generally

applies the wills-related rules to will substitutes, including life insurance policies. The

wills-related rules are primarily the rules of construction that arise out of changes that

can occur btwn when the instrument was created and when the transferor/testator

dies.

a. Cook v. Equitable Life Assurance Society (Revocation of POD by operation of

law – Traditional Approach) (p. 420)

b. F/P

i. Douglas Cook purchased a life insurance policy and named the

appellant (Doris) as the beneficiary. Approximately two years later,

Cook and Doris (appellant) divorced, and he married Margaret.

ii. The insurance policy expressly provided that the owner of the policy

may change the beneficiary by written notice to the company.

Douglas never gave written notice to the ins co to remove Doris, but

after marrying Margaret, he executed a holographic will that expressly

stated that he was giving the ins policy to his wife Margaret and his

son.

c. I

i. May a testator change the beneficiary of his life insurance policy

through a will even though it does not comply with the prescribed

method in the insurance policy?

d. R/A/H

i. HELD: No – The divorce did not revoke the contractual provisions in

favor of Doris (ex-spouse) – ct awarded the life insurance proceeds to

Doris as the contractual beneficiary designation controlled.

ii. RULE: Beneficiaries of a life insurance policy may not be changed by a

will if the policy contract provides a specific method for changing

beneficiaries.

iii. Strict compliance with insurance policy requirements is necessary to

change a beneficiary under the policy. The insurer, the insured, and

beneficiary should be able to rely on the certainty that policy

provisions relating to the naming and changing of beneficiaries will

control

iv. Courts will protect the expectation interest of a beneficiary under a

policy. Because the testator remarried, his first wife would not have

Chapter 6 – Nonprobate Transfers and Planning for Incapacity

66 of 120

known that he had changed her as the beneficiary because he

changed it in his will and not with the Society

e. Common Law: Revocation by operation of law applies to wills only

f. UPC/Modern Trend: Revocation by operation of law applies to all will

substitues, including life insurance.

g. UPC/Modern Trend Survival Requirement: While the UPC applies an express

survival requirement to life insurance contracts, it is silent w.r.t. general

contracts with P.O.D. clauses. (UPC 2-104, 2-702, 6-101).

iii. Langbein – The Nonprobate Revolution and the Future of the Law of Succession (p. 410)

iv. UPC §6-101 Nonprobate Transfers on Death (p. 411)

1. A provision for nonprobate transfer on death in an insurance policy, contract of

employement, etc. is NONTESTAMENTARY

2. Transfer on Death Deeds: New development in law—some states allow Transfer on

Death Deeds; there isn’t yet a uniform/model law for it. Key characteristics are: (1)

Deed must be executed and recorded inter vivos, but it does not take effect until the

grantor’s death; (2) Deed is revocable during grantor’s life (usu by recording another

deed that revokes the initial deed); (3) transfer is effective immediately upon the

grantor’s death and avoids probate.

v. Superwills: If a will is permitted to change the terms of a will substitute, the will is called a

“superwill.”

1. General Rule: Most jurisdictions have rejected the idea of a superwill

2. UPC: The UPC adopts the superwill doctrine only if the contract permits the beneficiary

of the policy to be changed by a subsequently executed will (UPC 6-101). The UPC is

silent as to what the rule should be if the K does not addres the issue

3. Restatement: See Casebook p. 423.

c. Pension and Retirement Accounts

i. Introduction: Pension plans vary greatly, but they typically involve the creation of a property

right in a fund of money, to be used by the retiree upon retirement. The plans usually allow the

retiree to designate a third party beneficiary who shall receive whatever is left, if the retiree

should die before the plan proceeds are exhausted.

ii. Langbein – The Twentieth-Century Revolution in Family Wealth Transmission (p. 424)

iii. Defined Benefit vs Defined Contribution Pension Plans (p. 425)

1. Defined Benefits Plan: Usually, defined benefits plans are funded by employers; there

is no individual account; the employee is entitled to receive a fixed benefit (e.g. a

percentage of their highest annual salary) for the remainder of his/her life – an annuity.

a. Annuities: An annuity is a stream of income for the remained of one’s lifetime,

paid monthly at a fixed amount. Annuities can be purchased separately or are

an option as to how one can receive benefits from a pension plan.

i. Annuities shift the risk of long life (i.e. long-time payouts) to the

company/provider. This is the flip of how it is w/ life insurance. This is

the common plan for gov’t providers (but it is declining in popularity

among the private sector).

b. Joint and Survivor Annuity: Guarantees fixed payments not only for the life of

the employee, but also for the life of the spouse.

2. Defined Contribution Plan: The employer, or employee, or both make contributions to

a specific pension account for the employee (e.g. 401(k)). Upon retirement, the

individual has rights to withdraw from the account, subject to various distribution rules.

Because the employee owns all of the funds in the account, the dcp often leads to lump-

sum payouts on the death of the worker and her spouse.

3. Federal Regulation: Private pension plans are heavily regulated by federal statutes—

most notably the Employee Retirement Income Security Act of 1974 (ERISA).

a. General Rule: Federal Law pre-empts state law on time-of-death issues.

b. Egelhoff v. Egelhoff (Pension Plans) (p. 426)

Chapter 6 – Nonprobate Transfers and Planning for Incapacity

67 of 120

c. F/P

i. The Petitioner was married to David A. Egelhoff. Egelhoff was

employed by Boeing who gave him a life insurance policy and pension

plan that were both governed by ERISA. The Petitioner was designated

as a beneficiary under both plans. Afterwards, they divorced, and the

husband Egelhoff died in an automobile accident two months later.

Egelhoff’s children of a previous marriage challenged her status as

beneficiary because a state law revokes all nonprobate testamentary

gifts to former spouses.

d. I

i. Does ERISA preempt a state statute that revokes the payment of a

non-probate asset to a former spouse?

e. R/A/H

i. HELD: Yes – The federal Employee Retirement Income Security act

preempts a state statute which revokes the payment of a non probate

asset to a former spouse because the Washington statute interferes

with the federal statute’s s goal to administer a nationally uniform

plan.

ii. RULE: Wash Statute: Divorce automatically revokes the beneficiary

designation in favor of an ex-spouse in all revocable nonprobate

arrangements.

iii. RULE: ERISA expressly pre-empts all state laws insofar as they may

relate to any employee benefit plan.

iv. DISSENT (Breyer): The state law imposes a mere administrative

burden on the ERISA statute at the expense of other substantive state

goals. This Court has held that the fact that state law poses some

burden on the administration of ERISA plans does not necessarily

require pre-emption. ERISA’s ultimate goal is to protect employee

benefits and the state law seeks to transfer an employee’s pension in

the manner they wanted to receive them. In this case the Court

permits a divorced wife to receive a windfall at the expense of the

testator’s children. The logic of this Court would also extend to state

cases involving slayer statues that prohibit a husband who kills a wife

from receiving benefits as a result of a wrongful death.

v. The ERISA statute commands that a plan shall, “specify the basis on

which payments are made to and from the plan.” If administrators are

forced to act in accordance with the state statute, they will have to

comply with the varying statutes of all 50 states and wait on litigation

before processing a payment. This delay conflicts with the legislature’s

goal of minimizing the administrative and financial burdens placed on

beneficiaries.

vi. The state law directly interferes with the purpose of ERISA because it

would cause beneficiaries to endure lengthy litigation before receiving

their payments and the administrators would have to comply with

several states with conflicting laws in order to distribute payments.

d. Multiple-Party Bank and Brokerage Accounts

i. Introduction: The various types of multiple-party bank and brokerage accounts include (1) Joint

and Survivor, (2) POD, (3) Agency or Convenience, and (4) saving (Totten trust) accounts.

ii. Problem with Multiple-Party/Brokerage Accounts: Sometimes, depositors intend for some

other type of account than Joint & Survivor; but banks used to only allow people to open Joint &

Survivor accounts. So the courts have to figure out what kind of account was actually intended

Chapter 6 – Nonprobate Transfers and Planning for Incapacity

68 of 120

1. Definition (Joint & Survivor): With joint & survivor accounts, either A or B has the right

to draw on the account, and the survivor solely owns the balance of the account, which

is non-probate.

2. POD disguised as Joint & Survivor: A might open a joint acct with B, intending only that

B receive the balance upon A’s death.

a. Common Law: Recall that at common law, POD accounts were invalid as

nonprobate transfers (UNLESS they were life insurance policies)

b. Modern Trend/UPC: POD accts are one of many POD arrangements permitted

as valid nonprobate transfers.

3. Agency or Convenience Acct disguised as Joint & Survivor: A might intend that B have

power to draw on the account during A’s life only for the convenience of A (e.g. to help

A pay bills), but not for other purposes, and not for B to receive anything at A’s death.

iii. Extrinsic Evidence to Prove Depositor’s Intent: Because banks and brokerage houses routinely

forced depositors interested in created various types of multiple-party accounts to use only one

type (the joint tenancy account), even if that wasn’t the depositor’s intent, courts allow extrinsic

evidence to prove depositor’s intent.

1. Burden of Proof: Rule: If the depositor executes paperwork that expressly states that

the account is a jt tenancy account, the paperwork creates a presumption that the

account is a “true” jt tenancy account. To overcome the presumption, move

jurisdictions require clear and convincing evidence of a different intent.

2. Varela v. Bernachea (Multiple-Party Accounts – Depositor’s Intent) (p. 432)

3. F/P

a. Bernachea, a retired atty, fell in love w/ Varela. After a year of traveling

together, Varela moved into his condo in Fla, and he paid all of her expenses.

Bernachea added Varela to his bank account as a joint tenant w/ right of

survivorship. Varela received a check card for the account, which she used

freely. Bernachea then suffered a heart attack, and while he was in the

hospital, Varela wrote a check that transferred the entire $280k in the acct into

an account in her name alone.

b. When Bernachea returned from the hospital, he demanded the bank return the

money to the original acct. The bank complied and initiated proceedings to

settle ownership status. Bernachea alleged that he did not have donative

intent, and that Varela had restricted access to the acct because she only had a

fcheck card and not paper checks.

4. I

5. R/A/H

a. HELD: Varela was entitled to half of the money in the acct.

b. When a jt bank acct is established with the funds of one person, a true joint

tenancy is presumed and may be rebutted ONLY by clear and convincing

evidence to the contrary.

c. A check card with no limit on its withdrawal abilities had the same significance

as a paper check, so Varela’s access to the acct was unrestricted.

d. Bernachea had claimed that he did not understand the meaning of a right of

survivorship (even though he was an attorney)—this fails to provide clear and

convincing evidence that Bernachea did not have donative intent.

iv. UPC Approach: The UPC provides that, inter vivos, it is presumed that the parties to a multiple

party account own in proportion to their contributions, and that upon the death of any party, it is

presumed that there is a right of survivorship.

1. The Presumptions control: The presumptions control the distribution of the money in

the account unless clear and convincing evidence of a contrary intent exists.

2. (See UPC 6-201 – 6-227)

e. Joint Tenancies in Realty

Chapter 7 – Restrictions on the Power of Disposition: Protection of the Spouse and Children

69 of 120

i. Introduction: Joint Tenancy and Tenancy by the Entirety are common/popular methods of

avoiding the cost and delay of probate. Upon the death of one joint tenant or tenant by the

entirety, the survivor owns the property absolutely, free from the decedent’s interest in the

property.

ii. Three Key Features of Joint Tenancy:

1. Equal Interests: The creation of a jt tenancy in land gives the joint tenants equal

interests upon creation.

a. Unlike jt tenancies in personalty (e.g. bank/brokerage accts), jt tenancies in

land require the agmt of all jt tenants to take most important actions.

b. A person who transfers land into a jt tenancy cannot, during like, revoke the

transfer and cancel the interest given the other jt tenant. (In this sense, a jt

tenancy is an imperfect will substitute because it is not revocable).

2. No Devise of Shares By Will: A joint tenant cannot devise her share by will. If a joint

tenant wants someone other than the other joint tenant to take her share at death, she

must sever the joint tenancy during life, converting it to a tenancy in common. This is

because, under the common law vanishing theory of joint tenancy, once the decedent

dies, there is no interest for the decedent’s will to operate on.

3. Creditors’ Rights: A creditor of a joint tenant generally must seize the joint tenant’s

interest, if at all, during the joint tenant’s life. In almost all states, the joint tenant’s

interest vanishes at death, leaving nothing for the creditor to reach.

3. Will Substitutes and the Subsidiary Law of Wills

a. R.3d §7.2 Application of Will Doctrines to Will Substitutes (p. 413 – not assigned, but cover it anyway)

4. See also IL statutes

a. 755 ILCS 25-1 to 2 (“Lifetime Transfer of Property Act”)

b. Skim 755 ILCS 27/1 to 100 (“Illinois Residential Property Transfer on Death Instrument Act”)

c. 755 ILCS 30/1 to 2 (“Third Party Beneficiary Contract Act”)

d. 735 ILCS 5/2-1403 (“Judgment debtor as beneficiary of trust”)

e. 205 ILCS 625/1 to 5, 10, 15 (“Illinois Trust and Payable on Death Accounts Act”)

f. 760 ILCS 25/1 (“Disclaimer Under Nontestamentary Instrument Act”)

Chapter 7 – Restrictions on the Power of Disposition: Protection of the Spouse and

Children 1. Introduction to Marital Property Systems

a. Separate Property (English Common Law):

i. Property: Husband and wife own separately all property that each acquires (including his/her

earnings). If one spouse is the wage earner, while the other works in the home, the wage-

earning spouse will own all of the property acquired during marriage (but gifts to the home-

spouse belong to him/her).

ii. Gifts: Gifts to either spouse are theirs separately

iii. Death of One Spouse: Upon the death of one spouse, under the elecetive share doctrine, the

surviving spouse has a right to claim a share of the deceased spouse’s property, regardless of the

terms of the deceased spouse’s will.

b. Community Property:

i. Property: While property acquired before marriage (and gifts acquired during marriage) by

either spouse are each spouse’s separate property, all earnings acquired (and any property

acquired w/ such earnings) during marriage by either spouse are community property. Each

spouse has an undivided ½ interest in each community property asset.

ii. Gifts: Gifts to either spouse are their separately.

iii. Death of Spouse: If one spouse dies, the community is dissolved – each community property

asset is divided in half. The surviving spouse’s half is his/hers immediately and outright (this

ensures that each spouse has a share of the marital property regardless of which spouse

Chapter 7 – Restrictions on the Power of Disposition: Protection of the Spouse and Children

70 of 120

acquired it). The deceased spouse’s half goes into probate where he or she can devise it to

whomever she/he wishes.

iv. Policy: The spouses are considered partners – any property acquired as a result of time, energy,

and/or labor of either spouse is considered owned by the partnership.

2. Rights of Surviving Spouse to Support (i.e. Right to Receive Support from Deceased Spouse):

a. Social Security: All workers in the USA have to pay into Social Security. SS pays benefits to (1) survivors

of predeceased workers, (2) the dependents of Social Security beneficiaries, and (3) (less importantly for

the purpose of wills & trusts), people who have already retired or are disabled. Only a surviving spouse

can receive the worker’s survivor’s benefit (a stream of income for life). The worker spouse cannot

transfer the benefit to anyone else.

i. Waiver: Surviving spouse cannot waive

ii. Divorce: A divorced former spouse of the worker has a right to benefits if the marriage lasted 10

years or longer

b. Private Employee Pension Plans: Under ERISA, a surviving spouse must have survivorship rights in the

worker spouse’s retirement benefits (usually an annuity; i.e. a stream of income for life—that is, support).

Unlike social security, under ERISA a surviving spouse can waive his or her rights in the worker spouse’s

private pension plan.

i. Waiver: Surviving spouse can waive, but waivers are not favored – ERISA has strict rules, e.g.

pre-nuptial agreements cannot waive ERISA-covered pension rights.

c. Homestead: Homestead Law is designed to secure the family home to the surviving spouse and minor

children, free of the claims of the decedent’s creditors.

i. Probate Homestead: Surviving spouse usu has right to occupy the family home (or perhaps

family farm) for his lifetime. This right is usu in addition to any other rights the surviving spouse

has in the decedent’s estate. In some states, the homestead must be established by the

decedent during life, usu by filing declaration of homestead in some public office; in others,

probate ct has power to set aside real property as a homestead.

d. Personal Property Set-Aside: The surviving spouse is entitled to claim certain tangible personal property

items, regardless of the deceased spouse’s attempts to devise them (i.e. if set-aside meets limitations &

conditions, the decedent usu has no power to deprive the surviving spouse of the exempt items). Items

are free from creditors’ claims (and usually include household furniture and clothing; may include a car

and farm animals). Details of the right vary state to state (some states have a statutory list of tangible

personal property to which the surviving spouse is entitled; other states have a monetary limit on how

much the surviving spouse may claim).

i. UPC 2-403: Sets the limit at $15,000, subject to the cost of living adjustment formula in 1-109

e. Family Allowance: Family allowance = statutorily authorized allowance for maintenance and support of

the surviving spouse (and often of dependent children). In some states, max allowance is fixed by statute.

In others, a “reasonable allowance” tied to the spouse’s standard of living is allowed. (See also UPC 2-404

and 2-405) – see p. CB 475

f. Dower and Curtesy: At common law, the main method of providing spousal support was either dower or

curtesy

i. Dower: A widow had dower in all of her husband’s qualifying land. Dower entitles the widow to

a life estate in 1/3 of her husband’s qualifying land.

1. When Dower Attaches: Dower attaches at the LATER of 1) the moment the husband

acquires title to land or 2) upon marriage.

2. When Dower Vests (completes): Dower is inchoate (incomplete) during the husband’s

life, and becomes complete at husband’s death. Once dower has attached (even

inchoate), the husband cannot unilaterally terminate it by transferring the land. No

purchaser (bona fide or not) can cut off the wife’s dower without her consent.

ii. Curtesy: At common law, the husband had a support interest in his wife’s lands, called curtesy.

It was the same as dower, but the husband was given a life estate in the entire parcel, not merely

1/3.

1. When Curtesy Attaches: Curtesy only attached if the husband and wife had children

Chapter 7 – Restrictions on the Power of Disposition: Protection of the Spouse and Children

71 of 120

iii. Modern Trend: Virtually all jurisdictions have abolished dower and curtesy in favor of the

elective share.

iv. IL Law – IL doesn’t have Dower/Curtesty (ILCS 755 5/2-9)

v. Note on Transferring Real Property: To avoid any possible claims, both spouses should sign any

deed transferring real property, even if the title is held in one spouse’s name, to ensure that no

dower or curtesy interest may be asserted after the transfer.

3. Rights of Surviving Spouse to a Share of Decedent’s Property

a. The Elective Share: All but one of the separate property states give the surviving spouse an elective share

of the decedent’s property (a.k.a. a “forced share”). It is called “elective share” because typical statutes

provide the surviving spouse w/ an election – either A) take under the decedent’s will or B) renounce the

will and take a fractional share of the decedent’s estate.

b. A Typical Elective Share: Typical elective share is 1/3 of all of the decedent spouse’s probate property,

plus certain nonprobate transfers (i.e. less of a share than in the community property approach).

c. Policy Issues to Consider: (1) WHAT credit, if any, should the non-wage earning spouse receive for

contributing to the partnership and enabling the wage-earning spouse to focus on earning money, and (2)

WHEN should that credit be recognized?

d. Policy Justifications for elective share:

i. #1 (the Partnership Theory): The surviving spouse contributed to the decedent's acquisition of

wealth and deserves to have a portion of it

1. Did they, though?? -- what if the higher-earning spouse earns WAAAAYYYY more than

the other?

ii. #2 (the Support Theory): Provide the surviving spouse w/ adequate support

1. e.g. possible to get a really low amount of support?

e. Personal Right: Is the election personal to the surviving spouse? i.e. can someone else (e.g. personal

representative) make the election?

i. Example Scenario: H dies, leaving a will that excludes W. Before W exercises her right of

election, she dies. Should W’s personal rep be allowed to renounce H’s will and take a forced

share? If yes, then W’s elective share would pass to W’s heirs/devisees. If no, then all of H’s

property will pass to H’s devisees.

ii. Majority/UPC Approach: Election right is personal – e.g. personal rep cannot act on surviving

spouse’s behalf. (see UPC 2-212(a))

iii. IL Law (755 ILCS 5/2-8) (blue sup p. 10)

iv. Incompetent Spouse: If the surviving spouse lacks the capacity to decide whether to exercise

the elective share, a guardian of the spouse can decide “in the best interests” (with the probate

court’s approval). UPC Approach to Incompetent Spouse: If the elective share is exercised for

an incompetent spouse, the share of the elective share that exceeds the share the spouse was

taking under the deceased spouse’s will is placed in a custodial trust, with the surviving spouse

having a life estate, and the remainder in the devisees under the will

f. Life Estates: It is not uncommon for the first-spouse-to-die to leave the surviving spouse only a life estate

in all or part of his/her property. Many reasons for this—one major reason is tax benefits (see p. CB p.

482)

i. Estate Taxes: The decedent’s estate tax depends on the size of his/her estate.

1. Example Scenario: Husband is the wage earner; he dies first with virtually all of the

property in his name.

2. Community Property Approach: The husband owned only half of the property,

resulting in a lower estate tax.

3. Separate Property Approach: The husband owned nearly all the property, resulting in a

higher estate tax.

a. Policy: Even if the husband left all of his property to his wife, the wife is worse

off under separate property because the after-tax amount passing to her is less

ii. Marital Deduction: Congress adopted the estate tax marital deduction to help the surviving

spouse (typically the wife) under the two property approaches.

Chapter 7 – Restrictions on the Power of Disposition: Protection of the Spouse and Children

72 of 120

1. Definition: Marital deduction provides that no estate taxes apply to any and all

transfers from the deceased spouse to the surviving spouse, even if the transfer is the

whole estate.

2. Rule: The decedent must transfer a life estate interest (typically this is done in trust and

is called a QTIP trust).

3. Policy: This protects the spousal transfer and also prevents states from earning easy

income from such transfers

iii. Same-Sex Couples: To date, no court has extended the elective share doctrine to include same-

sex couples who lived in a spousal-like relationship. But with states increasingly recognizing

same-sex marriages, such partners should have the right to claim the elective share.

g. What Property Is Subject to the Elective Share: Elective share statutes gave a surviving spouse a

fractional share in the decedent’s “estate.” The question is: does “estate” also include “nonprobate

property?”

i. Traditional Approach: The definition of “estate” in elective share meant “probate estate” only

ii. Modern Doctrinal Considerations: Should nonprobate property be included in the elective

share?

1. The “Illusory Transfer” Test: The most widely adopted judicial response to the

nonprobate avoidance problem. Ask whether the inter vivos property arrangement that

permits the property to avoid probate is really an inter vivos transfer, or whether the

deceased spouse retained such an interest in the arrangement that the transfer is more

testamentary than inter vivos. If the transfer is deemed illusory, then the transfer is

valid, but the property in question is included in the decedent’s estate subject to the

elective share

2. The “Intent to Defraud” Test: Focus on the deceased’s state of mind: Did the deceased

spouse intend to defraud his/her surviving spouse by creating the nonprobate property

arrangement in question?

a. Subjective Approach: Did the deceased actually intend to defraud the

surviving spouse of his/her right to an elective share in the property by creating

the nonprobate transfer?

b. Objective Approach: Focus on a variety of factors: (1) The amount of property

in question relative to the party’s overall property; (2) when the nonprobate

arrangement was created relative to the party’s death and relative to the

party’s marriage; (3) how much of an interest the deceased spouse retained;

etc.

3. The “Present Donative Intent” Test: Focus on whether the deceased spouse really had

a “present donative intent” at the moment he/she created the nonprobate transfer.

(One could argue this is very similar to the intent to defraud test, just worded

differently)

4. Sullivan v. Burkin (Elective Share and Trust Property) (p. 488)

5. F/P

a. Sullivan executed a deed of trust where he transferred to real estate to himself

as the sole trustee. The net income of the trust was payable to Sullivan during

his lifetime and the trustee could pay to him all part of the principal of the

estate as he might request by writing. Sullivan retained the right to revoke the

trust at any time.

b. At his death, the successor trustee was to pay the principal and any

undistributed income equally to defendants, George Fr. Cronin, Sr., and Harold

J. Cronin if they should survive him. The beneficiaries survived Sullivan.

c. Sullivan executed a will in where he stated that he intentionally neglected to

make any provision for his wife, appellant, Mary Sullivan, and his grandson,

Mark Sullivan. He directed the residue of the trust be paid to the trustee of the

inter vivos trust.

Chapter 7 – Restrictions on the Power of Disposition: Protection of the Spouse and Children

73 of 120

d. When Sullivan died, the appellant sought a determination by the court that the

trust property should be considered a part of his estate.

6. I

a. Whether an intervivos trust with a remainder interest is an invalid

testamentary disposition if the settlor retained broad power to modify or

revoke the trust, receive income and invade principal during his lifetime?

7. R/A/H

a. HELD: The court is a valid inter vivos trust. But assets in an inter vivos trust

created during marriage would be subject to the elective share if the deceased

retained a power to revoke or general power of appointment.

8. Bongaards v. Millen (Elective Share and Trust Created by 3rd

Party) (p. 492)

9. F/P

a. Jean Bongaards was the life tenant of a trust established by her mother. The

trust consisted of a 1.4 million apartment building. had the power to terminate

the trust at any time during her life and the corpus would have been paid to

her. Ten days before her death, Jean appointed the trust remainder to her

sister Nina. Bongaard’s specifically disinherited her husband in her will.

Bongaard’s husband petitioned the court to include the trust property in his

wife’s estate.

10. I

a. Whether a surviving spouse may invade a deceased spouse’s trust and take

their elective share, if the trust was established by a third party?

11. R/A/H

a. HELD: No, a surviving spouse may not invade a deceased’s spouse’s trust and

take their elective share if the trust was created by a third party. A third party

has no obligation sot support someone else’s property. Property owned by a

third party has never been a part of someone else’s spouse’s elective share

“estate.”

b. RULE: A surviving spouse may include the property of a trust within their

deceased spouse’s estate if the trust was created by third party for the benefit

of the deceased spouse

c. Most trusts are upheld that make testamentary dispositions at death but do

not comply with the Statue of wills on a theory that the beneficiaries have a

legitimate expectancy interest. However surviving spouses do not have a

legitimate expectancy interests in their deceased’s spouse’s trust benefits.

12. Revocable Trusts: In most states, a revocable trust created by the decedent spouse is

included in determining the surviving spouse’s elective share (R.3d Wills 9.1)

13. Conflict of Laws: What to do if the decedent’s elective share property is in a different

state?

a. UPC: The laws of the state where the decedent was domiciled at the time of

death control whether the surviving spouse is entitled to receive an elective

share in real property located in another state (UPC 2-202(d))

h. Statutory Responses to Elective Share (Expanding the Definition of Estate):

i. 1969 UPC Amendments - Augmented Estate: The surviving spouse is entitled to the decedent

spouse’s “Augmented Estate”

1. Augmented Estate: Augmented estate includes (1) decedent’s probate estate and (2)

certain nonprobate and gratuitous inter vivos transfers made during marriage (but NOT

life insurance proceeds):

a. any transfers where the deceased spouse retained the right to possession or

income from the property

b. any transfers where the deceased spouse retained the power to revoke or the

power to use or appoint (dispose of) the principal for his/her own benefit

c. any joint tenancies with anyone other than the surviving spouse

Chapter 7 – Restrictions on the Power of Disposition: Protection of the Spouse and Children

74 of 120

d. gifts to 3rd

parties within 2 years of the deceased spouse’s death in excess of

$3,000 per done per year, AND

e. property given to the surviving spouse either inter vivos or via nonprobate

transfers (including life estates in trusts).

f. Note: UPC approach includes a community property component

ii. 1990 UPC Amendments - Marital Property Approach: The augmented estate combines the

property of both spouses and gives the surviving spouse a share of the combined, “augmented”

estate that depends on the duration of the marriage

1. Sliding Scale: 1990 UPC Amendment abandons fixed share approach – the elective

share percentage increases depending on the length of marriage. The surviving spouse

starts out entitled to 3% of the couple’s marital property. The share increases roughly

3% each year until the spouse is entitled to 50% of the couple’s marital property after 15

years of marriage. (UPC 2-203)

2. Increase in Scope of Marital Property (vs Community Property): Unlike in community

property, the 1990 UPC Amendments include both (1) property acquired before

marriage and (2) gifts acquired during marriage in property that may be subject to the

elective share

a. Under community property, property acquired before marriage, and gifts

acquired during marriage are that spouse’s separate property, and the

surviving spouse has no rights in that property (unless the spouse commingles

it with community property)

i. Must the Surviving Spouse Accept a Life Estate?

i. They used to be forced to, but now (1993 UPC Amendments) the surviving spouse is not charged

against her elective share if she renounces the life estate

4. Migrating Couples and Multistate Property Holdings

a. Introduction: Migrating Couples cause problems for the spousal protection doctrine because of potential

conflicts of law: (1) real property is governed by the laws of the state where it is located, (2) personal

property is characterized at the time it is acquired as either separate or community property based on the

laws of the spouses’ domicile at the time of acquisition, and (3) the “at the time of death” spousal

protection that a surviving spouse is entitled to depends on the spouses’ domicile at the time of death of

the first spouse

b. Moving from Separate Property to Community Property State: The spousal protection scheme is

community property, BUT the characterization of the couple’s already-owned property does not change

(from separate to community) because the couple moved into a community property state.

i. Rule: The spousal protection scheme is community property. BUT the characterization of the

couple’s already-owned property does not change (from separate to community) because the

couple moved into a community property state.

ii. Example: H + W live in a separate property state. All of the couple’s marital property was

acquired by H, so the property is H’s separate property. They move to a community property

state, and H dies. W is entitled to 50% of their community, but they have no community

property because they have not acquired property in the new state.

iii. Quasi-Community Property: Quasi-community property is separate property that would have

been characterized as community property if the couple had been domiciled in a community

property jurisdiction if the couple had been domiciled in a community property state when the

spouse acquired the property.

1. Rule: When a spouse with quasi-community property dies, the quasi-community

property is treated like community property for distribution purposes. The surviving

spouse immediately receives a ½ interest in the quasi-community property that is his or

hers outright. The deceased spouse can devise only ½ of QCP.

iv. Order of Deaths: QCP gives the non-wage-earning spouse property rights in the property

acquired during the marriage by the other spouse only if the wage-earning spouse dies first.

Chapter 7 – Restrictions on the Power of Disposition: Protection of the Spouse and Children

75 of 120

1. QCP is not the same as community property – QCP applies only to the property owned

at death by the deceased spouse, not by the surviving spouse (i.e. QCP actually applies

to separate property).

2. If non-acquiring spouse dies first: he/she has no right to devise any of the surviving

(QCP-owning) spouse’s property (even if the surviving spouse has property that would

have been characterized as QCP if he/she had died first).

c. Moving from Community Property to Separate Property State:

i. RULE: The spousal protection scheme is the elective share. The non-wage earning spouse

receives his/her half of the community property outright, and the deceased spouse’s half goes

into probate. Surviving spouse can then claim an additional 1/3 or ½ interest in the deceased

spouse’s probate property

ii. Example: Before moving, H + W live in a community property state. All of H + W’s marital

property is acquired by and titled in the wage-earning spouse’s name. But the property is

treated as comm prop (each spouse owns ½ share). Couple moves to sep prop state; H dies. W

gets ½ by community property and H’s half goes into probate. W is also entitled to an additional

1/3 to ½ through elective share (i.e. W can double-dip in spousal protection).

5. Rights of Spouse Omitted from Premarital Will:

a. Introduction: Issue arises when a testator writes a will, then gets married, and then dies without ever

adding the spouse to the will

b. Omitted spouse presumption: Where the testator (1) marries after executing his or her will and (2) dies

w/o revising or revoking the will, a presumption arises that the testator ACCIDENTALLY disinherited the

spouse (i.e. testator meant to amend the will to provide for the spouse, but died before doing so).

i. Rebuttable Presumption: The presumption is rebuttable – to rebut, the challenger must show

either (1) the failure to provide for the new spouse was intentional, and that intent appears in

the will; (2) the testator provided for the spouse outside the will and the intent that the transfer

outside the will be in lieu of the spouse taking under the will is established by any evidence,

including oral statements y the testator and/or the amount of the transfer; or (3) the spouse

validly waived the right to share in the testator’s estate

ii. In re Estate of Prestie (Omitted Spouse – Presumption) (p. 516)

iii. F/P

1. Maria and W.R. Prestie were married in Las Vegas in 1987, but they were divorced two

years later. Over the years, they maintained a good relationship, and in 2000, when

W.R. became ill, Maria moved into his home to take care of him.

2. In 2001, W.R. amended an inter vivos trust he had established in 1994 to provide for a

life estate for Maria in his condominium.

3. Shortly thereafter, the couple was married. W.R. passed away nine months later.

W.R.'s son, the trustee and beneficiary of the trust, said that W.R.'s amendment to the

inter vivos trust rebutted the presumption of revocation of W.R.'s will as to Maria.

4. The ptf (the son and primary beneficiary under the decedent’s estate plan) asked the

court to expand the scope of the doctrine judicially to take into consideration the

decedent’s revocable trust. The decedent amended his revocable trust just a few weeks

before he married to grant his new spouse a life estate in his real property.

5. Son argued that the new spouse should not qualify as an omitted spouse because the

deceased spouse provided for her in his revocable trust

iv. I

1. Whether an amendment to an inter vivos trust could rebut the presumption that a will

is revoked as to an unintentionally omitted spouse.

v. R/A/H

1. HELD: An amendment to an inter vivos trust does NOT serve to rebut the presumption

that a will is revoked as to an unintentionally omitted spouse."

2. "[T]he only evidence admissible to rebut the presumption of revocation for the

purposes of NRS 133.110 is a marriage contract, a provision providing for the spouse in

the will, or a provision in the will expressing an intent to not provide for the spouse."

Chapter 7 – Restrictions on the Power of Disposition: Protection of the Spouse and Children

76 of 120

[citation omitted]. Since there was not a marriage contract, and there was nothing in

the will providing for the spouse or expressing an intent not to do so, the will was

revoked as to Maria.

c. Common Law/Minority: A premarital will was revoked on (1) marriage outright or (2) marriage following

the birth of a child.

d. Modern Trend/Majority: The premarital will is valid/left intact. However, state statutes give a surviving

spouse a spousal intestate share.

e. UPC: UPC 2-301 (see casebook p. 518) UPC grants an omitted spouse the right to receive no less than

his/her intestate share of the deceased spouse’s estate from that portion of the testator’s estate (if any)

that is not devised to a child of the testator or the child’s descendants (directly or through anti-lapse) if (1)

the child is not a child of the surviving spouse, and (2) the child was born before the testator married the

surviving spouse.

i. Effect of UPC: If the testator devises all of his probate estate to his child or descendants from a

prior relationship or marriage, the surviving spouse will not receive an omitted spouse’s share,

despite otherwise meeting the requirements of an omitted spouse.

f. Inter Vivos Trusts Treated as Wills for Omitted Spouse Statutes: In some states, the omitted spouse

doctrine arises only if the marriage occurs after execution of all the deceased spouse’s wills AND

revocable trusts, and the surviving spouse’s share is of the property included in the probate estate and

revocable trusts.

i. E.g. in Estate of Prestie, the court refused to consider the gift in the revocable trust in applying

the rebuttable presumption doctrine, because the state statute made express reference only to

the decedent’s will.

g. Malpractice Liability (see Emanuel’s p. 201)

6. Rights of Descendants Omitted from the Will

a. IL Approach: By default, if a child is born after the execution of the will, the child is entitled to receive the

portion of the estate he would have gotten had the testator died intestate (and all legacies shall abate

proportionately for that purpose)

i. Exception: If provision is made in the will for a child born after the will is executed, then that

provision controls

ii. Exception: If, from the will, it appears that the testator intended to DISINHERIT the child, the

child gets nothing. IL rolls like that, son.

b. Protection from Intentional Omission

i. Majority Approach: In all states except Louisiana, a child or other descendant has no statutory

protection against intentional disinheritance by a parent

c. Protection from Unintentional Omission

i. Introduction: The omitted child doctrine applies where a testator executes a will; some time

later, has a child; and dies without revising or revoking the will. The issue is whether the testator

meant to omit the child or not. “Pretermission” statutes are designed to prevent the

unintentional disinheritance of descendants.

ii. If the Will Provides for “Children Born After Execution”: If the testator writes a clause indicating

that children who are not born at the time of execution (but are anticipated/contemplated to be

born afterwards), generally those children do not qualify as omitted children (they may be

receiving a class gift)

iii. Children Alive At Will Execution: Some states extend the omitted child statute to include both

(1) children born after the will was executed and (2) children born before the will was executed,

but who weren’t named in the will

1. Affirmative Disinheritance Required: If the omitted child statute covers living children,

most courts require “affirmative disinheritance” – specific reference to the child.

Blanket statements, that the testator has no children or that no children are to take

under the will, usually aren’t enough to disinherit the child. (see Anna Nicole Smith note

on CB p. 534)

Chapter 7 – Restrictions on the Power of Disposition: Protection of the Spouse and Children

77 of 120

iv. Omitted Child Presumption: Where the testator has a child after executing his/her will, and

then dies w/o revising or revoking the will, a presumption arises that the testator meant to

amend his/her will to provide for the new child, but died before getting around to it.

1. Rebuttable Presumption: The presumption that the testator accidentally disinherited

his/her child is rebuttable.

a. To Rebut the Presumption: Traditional Rule—Presumption can be rebutted

ONLY by showing that either: (1) the failure to provide for the new child was

intentional and that intent appears from the will; OR (2) the testator provided

for the child outside of the will and the intent that the transfer outside of the

will be in lieu of the child taking under the will is established by any evidence,

including the amount of the transfer; OR (3) the testator had one or more

children when the will was executed and devised substantially all of his/her

estate to the other parent of the omitted child

b. Two types of omitted (or pretermitted) child statutes: “Missouri” type and

“Massachusetts” type

i. “Missouri Type” Statute: Under the “Missouri” type of omitted child

statute, the intent to omit the child must be determinable SOLELY

from the terms of the will. Extrinsic is not admissible.

ii. “Massachusetts” Type Statute: Extrinsic evidence IS admissible to

help determine whether the omission of the child was intentional.

v. Omitted Child’s Share: If the presumption of unintentional failure to provide for the new child is

not rebutted, the typical omitted child statute gives the omitted child his/her share of the

testator’s probate estate.

1. Gray v. Gray (Omitted Child’s Share) p. 528

2. F/P

a. John Gray executed a will while married to Mary, and although he had two

children from a previous marriage, he left all of his estate to Mary. 3 years

later, John and Mary had a son, Jack. 5 yrs later, John and Mary divorced. John

died w/o changing his will

b. Under Alabama’s revocation by operation of law doctrine, Mary was treated as

though she predeceased John (due to the divorce), so she did not take.

c. Jack petitioned the probate court to determine if he was entitled to a share of

John’s estate under Alabama’s omitted child doctrine.

3. I

4. R/A/H

a. HELD:

b. RULE: Usually, if a child is born after the execution of a will, and the will fails to

provde for the child, the omitted child is entitled to a share of the testator’s

estate.

i. An exception exists – an omitted child is NOT entitled to a share of the

estate if “when the well was executed, the testator had one or more

children and devised substantially all his estate to the other parent of

the omitted child”

c. Although the presumption arose that the omission was accidental, it could be

overcome by the exception that the testator had one or more children when

the will was executed, and he left substantially all of his estate to the other

parent of the omitted child (i.e. John meant to omit Jack from the will).

d. It did not matter that John’s other children were from a different marriage; the

exception still applies (Jack gets nothing).

e. DISSENT: The exception was meant to apply where the child could look to

inherit down the road from the other parent who took from the decedent.

Here, where the other parent was ineligible to take (due to divorce), the

exception should not apply.

Chapter 7 – Restrictions on the Power of Disposition: Protection of the Spouse and Children

78 of 120

vi. Overlooked Child vs Omitted Child: Some states have expanded the traditional omitted child

doctrine to include a living child who is omitted because (1) the testator does not know about

the child, or (2) the testator mistakenly believed the child was dead

1. General Rule: The overlooked child receives his/her intestate share, just like an omitted

(pretermitted) child

vii. UPC Approach: UPC 2-302 (see CB p. 531): If a testator fails to provide for any of his/her

children born or adopted after the execution of the will, the omitted after-born or after-adopted

child receives a share in the estate (NOTE: I am not typing out the whole rule. See the book)

1. UPC Applies to both Adopted and After-Born Children: The UPC expressly provides

that it applies to children born or adopted after execution of the will (UPC 2-301(a))

2. Intent to Omit: UPC dictates that evidence that the failure to provide for the child was

intentional must come FROM THE WILL (2-203(b)(1)) – NOTE: Unlike the UPC omitted

spouse doctrine, the UPC omitted child doctrine does not broaden the scope of the

evidence that can be used to prove intent to omit a new child. (Compare omitted

spouse doctrine above, p. 75)

3. Other Children: UPC

4. The Omitted Child’s Share (How Much Do They GET??): The omitted child’s share

depends on whether the testator has other children living at the time he/she executes

the will.

a. Testator Had No Children When Executing Will: The omitted child receives

his/her intestate share, UNLESS (1) the testator devised all or substantially all of

his/her estate to the other parent of the omitted child AND (2) the other parent

survives the testator and is entitled to take (in which case the omitted child

gets nothing). (UPC 2-302(a)(1))

b. Testator Had One or More Children Living When Executing Will: If the will

devised property to one or more of the then-living children, then the omitted

child’s share (1) is taken out of the portion of the testator’s estate being

devised to the then-living children, and (2) should equal the share or interest

the other children are receiving, had the testator included all omitted children

with the children receiving shares and given each an equal share. (Gifts to the

then-living children are to abate pro-rata (UPC 2-302(a)(2))

c. Overlooked Child: The UPC considers ONLY a child overlooked because the

testator thought him/her dead when executing the will. UPC treats those

children exactly the same as adopted or after-born children. However, The

UPC DOES NOT consider children overlooked simply because the testator did

not know (e.g. illegitimate children) (UPC 2-302(c))

5. Omitted Issue of Deceased Child: Most omitted child statutes cover omitted children

only. But some explicitly provide that they apply ALSO to the omitted issue of a child

who predeceased the testator. The omitted issue of the child who died before the

testator (e.g. the omitted grandchildren of the testator) take their intestate share (does

this mean that the issue evenly split the predeceased child’s share? Per stirpes? Etc?)

viii. Nonprobate Transfers and Pretermitted Child Statutes

ix. Introduction: With the increasing popularity of nonprobate modes of transfer, esp the use of

revocable inter vivos trusts as will substitutes, a new issue has arisen: Do the pretermitted heir

statutes protect a child omitted from a non-probate mode of transfer such as a revocable trust?

x. Modern Trend (Minority Approach?): A few states have modified the omitted child doctrine to

recognize that increasingly, the inter vivos revocable trust is being used much like a will. In those

states, the omitted child doctrine arises only if (1) the birth occurs after execution of all deceased

spouse’s wills and revocable trusts AND (2) the omitted child’s share is of the property included

in the combined probate estate and revocable trusts.

xi. Majority Approach: The pretermitted-heir statutes apply only to wills.

xii. Kidwell v. Rhew (Pretermitted Child Statutes and Revocable Inter Vivos Trusts) (p. 536)

xiii. F/P

Chapter 8 – Trusts: Introduction and Creation 79 of 120

1. Irene Winchester created a revocable, inter vivos trust. She transferred real property to

the trust, and named her daughter, Margie Rhew, as successor trustee upon Irene’s

death.

2. Irene never executed a will. Another daughter, Rhenda Kidwell, petitioned the ct to

apply Arkansas’ pretermitted-heir statute to the revocable trust. Kidwell argued that

the pretermitted-heir statute should apply to dispositions made by testamentary will

substitutes, such as an inter vivos trust.

xiv. I

1. Should the pretermitted-heir statute apply to dispositions made by testamentary will

substitutes, such as an inter vivos trust?

xv. R/A/H

1. HELD: No (Majority View).

2. RULE: Pretermitted-heir statutes apply only to wills, not trusts.

3. Kidwell cited no convincing authority that would compel the court to conclude that the

pretermitted-heir statutes apply to trusts.

4. A will and a trust are different things legally. A will is a disposition of property to take

effect on the death of the testator. A trust is a fiduciary relationship in which one

person holds title to a property subject to an equitable obligation to keep/use the

property for the benefit of another. The terms are not interchangeable. Thus, the

statutes, which speak only in terms of “execution of a WILL” do not apply to trusts.

7. See Also IL Statutes

a. 755 ILCS 5/15-1 to 4 (“Spouse and Child Awards”)

Chapter 8 – Trusts: Introduction and Creation 1. Introduction: A trust is a fiduciary agreement by which a settlor transfers legal/title ownership of trust property

(the trust res or trust corpus) to trustees, who hold/manage/use the property for the benefit of the beneficiaries,

who have equitable interests in the property.

2. Trust Purpose: Trusts can be used for many purposes (trusts are the most flexible legal instruments available).

The main point of trusts is to separate the benefits of ownership (for the beneficiaries) from the burden of

ownership (for the trustees).

3. Trust Structure: Prototypical trust structure is that A transfers property to B for the benefit of C (and possibly

others). That is a trust. A is the settlor; B is the trustee; and C is the beneficiary

a. Example (Gift vs. Trust): Gift � A gives B $1,000. This is a classic inter vivos gift. B can do whatever she

wants with the $1,000. Trust � A gives B $1,000, to use for the benefit of C during C’s lifetime; then on

C’s death, any remaining principal is given to D. This is a classic trust.

b. 5 Common Trusts:

i. Revocable Trusts: O declares herself trustee of property for the benefit of O for life, and then on

O’s death, to pay the principal to O’s descendants. O retains the power to revoke the trust.

1. S

4. Basic Trust Rules: These are the main rules:

a. The same party can wear all 3 hats: i.e. One person can be settlor, trustee, and beneficiary, as long as

there is another trustee or beneficiary.

i. Merger: If the same party is both trustee and beneficiary, and there is no other trustee or

beneficiary, the legal title and equitable title are said to merge, and the trust is terminated (i.e.

bifurcation of legal and equitable titles is essential to a trust).

b. A trust is not created until it is funded: A trust is not funded until property is transferred to the

trust/trustee. Where the trustee is a 3rd

party (someone other than the settlor), property MUST be

transferred to the trustee, with the intent that the trustee hold and manage it for the benefit of someone

else.

i. Note: Executing a deed of trust does not constitute funding.

c. A trust will not fail for lacking a trustee: Where the settlor clearly expresses the intent to create a trust

of provides for funding, a trust will not fail for want of a trustee.

Chapter 8 – Trusts: Introduction and Creation 80 of 120

i. Trustee Unable/Declines to Serve, or Settlor Forgets to Appoint Trustee: If a trustee declines to

serve (a named trustee is not required to serve but rather must accept the appointment), dies, or

is unable to continue, or if the settlor forgot to name a trustee, the court will appoint a successor

trustee.

ii. If the court appoints a trustee: If the court appoints a trustee, and the trustee accepts the

position, due to the fiduciary duties inherent in the office, the trustee can leave the position only

with court approval or the consent of all beneficiaries.

iii. If the will creates the trust, but fails to name the trustee: The general rule is to appoint the

executor as trustee.

iv. Exceptions: If the court concludes that the powers given to the trustee were personal, to be

exercised only by that particular trustee (e.g. the settlor intended the trust to last only as long as

the named trustee actually served as trustee), the court will DECLINE to appoint a successor

trustee, and the trust will fail. BUT this exception is narrowly construed, and rarely applied.

d. Co-Trustees must agree on action: If a private trust appoints co-trustees, the general rule is that all the

trustees must consent to any proposed action. An individual trustee or subgroup of trustees cannot act

alone. BUT the trust instrument, however, can provide that the trust can act upon a majority vote of the

trustees.

i. UTC Approach: The UTC rejects the common law rule and permits action based on the vote of a

majority of the co-trustees (no requirement to specify majority vote in the trust instrument).

5. The Parties to a Trust

a. The settlor – The person who creates the trust.

i. Inter Vivos Trust: Settlor creates the trust during his/her life

ii. Testamentary Trust: Trust is created by will

1. Note: If a trust is created by will, then of course, the settlor can’t be the trustee.

iii. Trust Creation: Trusts may be created by

1. Declaration of Trust: Settlor declares that he holds certain property in trust – this

occurs if the settlor is also the trustee

2. Deed of Trust: Settlor transfers property to another person as trustee

a. RULE: If the settlor is not the trustee, then deed of trust or actual delivery of

the trust property to the trustee is REQUIRED

b. The trustee – The person/people who own legal title to the trust property.

i. A trust may have one or more trustees

ii. TRUSTEE RULES:

1. For the trust to be valid, the trustee must owe equitable duties to someone other than

himself/herself (i.e. the same person cannot be the sole trustee and also the sole

beneficiary)

2. A trust will not fail for want of a trustee:

a. If the settlor intends to create a trust, but fails to name a trustee, the court will

appoint one (R.3d §§31, 34)

b. If a will names someone as trustee, but the named person refuses the

appointment or dies while serving as trustee, and the will does not provide for

a successor trustee, the court will appoint a successor trustee.

3. The trustee MUST have active duties to perform. If the trustee has no duties at all, the

trust is said to be “passive” or “dry,” the trust fails, and the beneficiaries get legal title to

the trust property (i.e. the beneficiaries take what the trustee had).

iii. Bifurcated Ownership

1. The trustee owns legal title to the trust property (a.k.a. trust corpus). The beneficiaries

own equitable/beneficial interests.

iv. Legal Entity Status of Trustee: The trust is not a freestanding legal entity with the power to sue,

be sued, and transact in its own name. Instead, the trustee sues, is sued, and transacts in his

capacity as such.

v. Rights of Creditors of Trustees

Chapter 8 – Trusts: Introduction and Creation 81 of 120

1. Creditors of the trustee as an individual: Although the trustee has legal title to the

trust property, a personal creditor of the trustee has no recourse against the trust

property (R.2d Trusts §308).

2. Creditors of the trustee as a trustee: Creditors of the trustee as trustee (i.e. a creditor

who interacts with the trustee in regard to trust property) has recourse against the trust

property, but not the trustee’s personal property (UTC § 1010)

vi. Duty of Trustee

1. Trustee is held to a fiduciary standard of conduct. This includes:

2. Duty of loyalty: Trustee must administer the trust solely in the interest of the

beneficiaries (self-dealing is sharply limited and often prohibited altogether)

3. Duty of Prudence: Trustee is held to an objective standard of care in managing the trust

property

4. Subsidiary Duties:

a. Duty of Impartiality Between Classes of Beneficiary: Treat all classes of

beneficiaries the same (e.g. high-income beneficiaries, remainderpersons, etc.)

b. Duty Not To Commingle: Don’t commingle trust property with trustee’s own

property

c. Duty to Inform and Account: To the beneficiaries

5. Penalties for Failing to Meet Duties: A trustee who breaches her fiduciary duties may

be denied compensation, subjected to personal liability, and removed as a trustee

6. Rejecting Trustee Appointment:

a. Common Law: Once a person accepts appointment as trustee, the person

cannot be released from office unless all of the beneficiaries consent, or by

court order.

b. UTC Approach: Allows for trustees to resign with 30 days’ notice to all

interested parties (UTC 705)

vii. Individual vs Corporate Trustees

1. Trustee may be an individual or a corporation

2. Settlors often name co-trustees (e.g. individual and corporate)

3. Delegation: Trustees may delegate to experts on matters outside the trustee’s

competence

a. Modern Approach: Delegation is allowed

4. Individual: Usually costs less than corporate trustee, and usually has a strong sense of

settlor’s wishes.

5. Corporate: e.g. banks with a trust dept experienced in portfolio mgmt. and trust

administration. Usually higher fees (cost of expertise, deep pockets, and institutional

safeguards)

6. Trustee Compensation: Note: The rules governing trustee compensation entitle the

trustee to “reasonable compensation.”

a. Right to Contract for Trustee Compensation: The default rules of

compensation may be displaced by contrary agreement; corporate trustees

usually insist on such agreements

b. Traditional Rule of Trustee Compensation: Award the trustee an annual

commission set by a statutory formula (usually a % of the trust corpus, or

percentage of trust income, or some combo of the two).

c. Modern Rule of Trustee Compensation: UTC 708 and R.3d Trusts § 38 entitles

the trustee to “reasonable compensation”

7.

c. The beneficiaries – the people who own beneficiary title to the trust property

i. Rights of Beneficiaries:

1. If the trustees breach duties, beneficiaries have a claim against trustees personally

Chapter 8 – Trusts: Introduction and Creation 82 of 120

a. Wrongful Disposure: If the trustee wrongfully disposes of the trust property,

the beneficiary can recover the property (unless the property has come into

the hands of a bona fide purchaser for value).

b. If the trustee disposes of trust property and acquires other property with the

proceeds of sale, the beneficiaries can enforce the trust on the newly acquired

property

6. Remedial Trusts: Remedial Trusts arise by operation of law and thus are not subject to the requirements to create

a valid trust. They are used as equitable remedies by courts to order one party who is currently holding property

to transfer the property to another party whom the court concludes has a stronger equitable claim to the

property.

a. Resulting Trust: A resulting trust arises when a trust fails on whole or in part. Courts use it to require the

party holding the property (usu. The trustee) to return the property to the settlor (or the settlor’s estate if

the settlor is dead). i.e. the Resulting Trust is an equitable reversionary interest.

i. Scenario 1: Where an express trust fails or makes an incomplete disposition (R.3d Trusts §8)

ii. Scenario 2: Where one person pays the purchase price for property and causes title to the

property to be taken in the name of another person who is not a natural object of the bounty of

the purchaser (R.3d Trusts §9)

b. Constructive Trust: Constructive trusts are used to prevent unjust enrichment.

i. Traditional Approach: The courts required (1) a confidential or fiduciary relationship between

the transferor and the transferee, (2) a promise (express or implied) by the transferee, (3) that

the transferor transferred property to the transferee in reliance on the promise, and (4) that the

transferee refuses to honor the promise (which constitutes unjust enrichment)

ii. Modern Trend: Courts tend not to emphasize the elements themselves, but they do focus on

the equitable notion that the constructive trust is used to prevent unjust enrichment.

7. Trust vs Legal Life Estate

a. Legal Life Estate: Life tenant possesses and controls property (i.e. life tenant has legal ownership of

property

i. Formation: Property “to A for life, remainder to A’s descendants”

ii. Problems Arise:

1. Cannot Sell Fee Simple: Legal life tenant cannot sell a fee simple unless the life estate

instrument explicity grants such power to the life tenant. Otherwise, life tenant must

get permission from ALL remainderpersons and reversioners, or get judicial approval.

2. Obtaining A Mortgage or Leasing Property: Potential lenders/lessees will be wary of

working with land owned by a life tenant who will eventually die (and then possession

of the land will go to someone else.

3. Waste: e.g. cutting timber, taking down a still-usable building on the property may give

rise to waste – remainderpersons might have rights to injunctions or damages.

4. Risk of Creditors: If the life tenant gets into debt, the creditor can seize the life estate

and sell it.

b. Trust (aka Equitable Life Estate): Trustee, not the life beneficiary, has legal title to the trust property

i. Formation: Property “to X in trust for A for life, remainder to A’s descendants.”

ii. Trusts Cure/Mitigate the Problems with Life Estates: (see CB p. 555)

1. Risk of Creditors: Trusts can be put out of reach of creditors (unlike legal life estates)

8. Trust vs Debt: A trust is not the same as a debt—the requirement of an identifiable trust res distinguishes a trust

from a debt

a. Trust: A trustee holds a fiduciary duty to hold a specified property for the benefit of another. The trust

property must be kept separately from the trustee’s own funds

b. Debt: Involves a personal obligation to pay a sum of money (not necessarily a specifically identified

property) to another.

c. Key Analysis: Whether the recipient of the assets is entitled to use them as his own and commingle them

with his own assets. (R.3d Trusts 5, cmt k)

9. Creation of A Trust

Chapter 8 – Trusts: Introduction and Creation 83 of 120

a. Overview: 4 core requirements to create trust: (1) settlor’s intent to create trust; (2) funding – property

transferred to the trust/trustee; (3) the beneficiaries must be ascertainable; and (4) writing (i.e. the trust

act does not require writing, but the law of wills and/or the statute of frauds (if the trust involves real

property) requires writings)

b. Intent to Create a Trust: The grantor must manifest an intention to create a trust relationship (UTC

402(a)(2), R.3d Trusts 13). No specific words are required—only intent.

i. Lux v. Lux (Intent to Create Trust) (p. 557)

ii. F/P

1. Philomena Lux died testate devising the residue of her estate to her grandchildren. In

the residuary clause of the will, the testator stated that her estate “shall be maintained

for the benefit of said grandchildren and shall not be sold until the youngest of said

grandchildren has reached twenty-one years of age.”

2. The Court determined whether the language in the will created a testamentary trust.

iii. I

1. Did the language in the will devise the property to testator’s grandchildren outright, or

in trust?

iv. R/A/H

1. HELD: The will created a trust.

2. RULE: A trust is created when legal title to property is held by one person for the

benefit of another. No particular words are required to create a testamentary trust (the

absence of words such as trust or trustee is immaterial where the requisite intent of the

testator can be found).

3. RULE: General rule (will executor as trustee): Unless either (1) a contrary intention

appears in the will or (2) the appointment of executor as trustee is deemed improper or

undesirable, the executor of the will would be named trustee.

4. The language in the will did not give the grandchildren a fee simple title to the realty.

The testator, realizing the nature of the bequest and the age of the beneficiaries,

intended that someone would hold/manage the property until they were of sufficient

age to do so themselves.

v. Jiminez v. Lee (Intent to Create Trust/Language) (p. 558)

vi. F/P

1. The defendant’s mother purchased a $1,000 face value U.S. Savings Bond which was

registered in the names of defendant, “and/or” plaintiff, “and/or” Dorothy Lee,

plaintiff’s mother. One of the defendant’s clients made one $500 gift to the defendant

and two other $500 gifts to the defendant’s two other children. The donor deposited

$1,500 into a savings account in the name of the defendant and his three children. The

defendant cashed the savings bond and invested the proceeds in common stock of the

Commercial Bank of Salem, Oregon. The shares were registered as “Jason Lee,

Custodian under the Laws of Oregon for Betsy Lee.” The defendant closed the joint

savings account containing the client’s gifts to defendant’s children and $1,000 of the

proceeds invested in Commercial Bank stock. The defendant also took title to this stock

as “custodian” for his children. The trial court held that the defendant did not hold the

savings bond or the savings account in trust for the benef

2. it of the plaintiff. At trial, the defendant testified that his mother said that she was going

to supply a bond to help with his daughter’s educational needs and that she was naming

him and his wife to use the funds as may be conducive to the educational needs of his

daughter. The defendant also wrote a letter to his mother in which he stated that his

children, Dave and the plaintiff were aware that the defendant held his mother’s

property in trust for them.

vii. I

1. Whether a trust exists where the specific term “in trust” is not used to transfer the

property?

viii. R/A/H

Chapter 8 – Trusts: Introduction and Creation 84 of 120

1. HELD: Yes. A trust exists here even though the specific term “in trust” was not used to

transfer the property.

2. RULE: No specific words are required to create a trust, as long as the requisite intent is

expressed in the trust instrument.

3. It is sufficient that the defendant testified that he received property for the benefit of

his own children and he admitted in a letter that he wrote to the owner that he held

property in trust for his children. Even though the defendant purchased bank stock as

the “custodian” for the plaintiff under the Uniform Gift to Minors Act, it was ineffectual

to increase the defendant’s powers over the trust property from that of a trustee to a

custodian.

4. The particular facts revealed during the trial showed that the defendant was aware that

he was holding property in trust for the plaintiff.

ix. Fuzzy Distinctions between Gift and Trust: The intent to create a trust is merely a variation on

the intent to create a gift; sometimes it is hard to tell the difference between the two. Two

particularly difficult scenarios are:

1. Precatory Trust: i.e. language used such as “wish” or “hope.” Precatory language DOES

NOT CREATE A LEGAL OBLIGATION that the property be used for the benefit of another.

A precatory trust is not a trust at all, but merely a “gift with a wish.”

a. Key Analysis: Whether the language indicates the intent to vest the beneficial

interest in the 3rd

party (in which case the transfer is a trust), or whether the

language expresses merely the hope or wish that the recipient of the property

uses it for the 3rd

party’s benefit (in which case, it is a gift to the recipient, who

is not actually a trustee).

b. Failed Gifts: Note – this in in Emanuel’s, but it wasn’t assigned reading, so

fuuuuuuuuck it.

2. Unthank v. Rippstein (Transfer of Property to Trust) (p. 569)

3. F/P

a. Craft handwrote a letter to Iva Rippstein promising to her that he would make

monthly payments in the amount of $200.00 to the appellee for the next five

years, if he lived that long. At the end of the letter, he wrote in his own

handwriting that he bound his estate to the payments and struck out the words

stating “provided I live that long.”

b. The appellee filed an action against the estate of the Craft demanding

payments; she argued that the letter constituted a declaration of trust.

c. The trial court ruled in favor of the appellee. The Court of Civil Appeals

reversed and held that the letter established a voluntary trust.

4. I

a. Whether a donor creates a trust by promising to make monthly payments in

the future for an indefinite amount of time

5. R/A/H

a. HELD: No. A donor does not create a trust by promising to make monthly

payments in the future.

b. RULE: To prove a trust, the facts must show that the donor intended to act as a

trustee or have the property managed by a trustee, not just a sincere desire to

make a gift.

c. Craft wrote that he bound his estate to a promise to make monthly payments

and excluded the clause stating “provided I live that long.” However this

language does not suggest that the donor intended to be a trustee over the

payments. His intent was to bind his estate to such payments but not to create

a trust.

i. He also did not expressly declare that all or any specific portion of his

property/assets would constitute the trust corpus.

Chapter 8 – Trusts: Introduction and Creation 85 of 120

c. The Necessity of Funding (Transferring Trust Property): A trust cannot exist without trust property, often

called the “res” or the “corpus.” The res may be one dollar or one cent or any interest in property that

can be transferred (R.3d Trusts §40, cmt b)

i. Traditional Approach: The act of funding the trust was viewed as completely separate from

creating the trust. Funding was a function of the law of property—the asset had to be delivered

to the trust/trustee, and the Statute of Frauds governed the requirement of written evidence of

the transfer/funding.

1. Transfers of Real Property: Required there to be a grant deed, separate from the

declaration of trust or deed of trust, which transferred the real property from the settlor

to the trust/trustee.

2. Transfers of Personal Property: Required either (1) the item itself to be manually

delivered to the trust/trustee or (2) a written instrument manually delivered to the

trust/trustee that listed the personal property in question and evidenced the

symbolic/constructive delivery of the item(s) to the trust.

ii. Modern Trend Approach: The requirements of Creating and Funding the trust can overlap and

can be one and the same.

1. Written DECLARATION of Trust and Real Property: i.e. Settlor is the trustee; written

declaration of trust IDs the real property the trust holds

a. Majority Rule: The declaration of trust also transfers the real property interest

into the trust, without a separate writing (UTC 401).

b. Minority Rule: Separate writing required to transfer title to the real property

from the settlor as nontrustee to the settlor as trustee (this seems pointless…)

2. Written DECLARATION of Trust and Personal Property: i.e. Settlor is the trustee;

written declaration of trust specifically IDs the personal property the trust holds

a. Majority Rule: Writing will be sufficient to simultaneously create the trust,

transfer the personal property interests into the trust, and transfer the

equitable interest to the beneficiary (UTC 401)

b. Minority Rule: If the trust property is stock or other titled property, the settlor

is required to change the registration of the trust property from the settlor as

nuntrustee to the settlor as trustee to transfer property to the trust.

3. Oral DECLARATION of Trust and Personal Property: i.e. Settlor is the trustee; oral

declaration (by settlor) specifically identifies the personal property the trust holds

a. Majority Rule: The oral declaration will be sufficient to create the trust and to

transfer the personal property interests into the trust.

b. Overlap of Funding with Intent: Because the elements of trust creation and

funding overlap, evidence of creation and of intent may overlap as well. (See

Emanuel’s p. 222??)

c. Note: There is no oral declaration that can specify real property because the

Statute of Frauds requires written evidence of the transfer.

4. Written DEED of trust and Real Property: i.e. Trustee is a 3rd

party (not the settlor);

written deed of trust IDs the real property the trust holds

a. Jurisdictions are split: on whether a reference to the property in a list

attached to the deed of trust is sufficient to constitute delivery in all cases or

whether the settlor must do more if the property is real property (e.g. execute

a separate grant deed transferring title from the settlor to the trust/trustee).

5. Written DEED of trust and Personal Property: i.e. the trustee is a 3rd

party (not the

settlor); written deed of trust specifically includes a list that identifies the personal

property the trust holds

a. Jurisdiction are split: on whether a reference to the property in a list attached

to the deed of trust is sufficient to constitute delivery in all cases or whether

the settlor must do more if the property is stock (e.g. re-register the stock) or

other titled property.

6. Oral DEED of trust and Personal Property: i.e. trustee is a 3rd

party; deed of trust is oral

Chapter 8 – Trusts: Introduction and Creation 86 of 120

a. Majority Rule: Even if the settlor specifically identifies the personal property

the trust holds, oral declaration is insufficient to create the trusts and to

transfer personal property interests into the trust. There must be some other

evidence of delivery, i.e. either actual delivery of the personal property in

question to the trust/trustee or deliver of a written list of personal property

assets being transferred to the trust/trustee.

7. UTC Approach: The Uniform Trust Code expressly PERMITS oral trusts, UNLESS

otherwise prohibited by state law (e.g. Statute of Frauds). The required proof of the

terms is clear and convincing evidence. (UTC 407)

iii. Grantor Trusts—Taxation (see CB p. 576)

1. Grantor Trust: Trusts in which the income is taxable to the settlor (grantor) because the

grantor has retained substantial control and is deemed by the Internal Revenue Code to

still be the owner of the trust assets.

2. In Brainard v. Commissioner (not assigned), the taxpayer was trying to avoide taxes by

shifting the income generated by his stock trading from himself, as settlor, to the

income beneficiaries, who were in lower tax brackets. Such tax avoidance is no longer

available.

a. RULE: Where the settlor/grantor retains sufficient dominion and control over

the trust, under what are known as the “Clifford Regulations,” the trust is

known as a grantor trust and the settlor is deemed owner of the income

generated by the trust for tax purposes.

d. The Necessity of Trust Beneficiaries

i. Introduction/RULE: A Trust MUST have one or more ascertainable beneficiaries (UTC 402(a)(3)),

R.3d Trusts 44). There must be someone to whom the trustee owes fiduciaries; someone who

can call the trustee to account. A private trust must be for the benefit of the beneficiaries.

1. Charitable Trust Exception: A charitable trust (unlike a private trust need not have an

ascertainable beneficiary to be valid – but note—we skipped charitable trusts on the

syllabus, so…. Fuuuuuuckit!!

2. Unborn Children Exception: The beneficiaries of a private trust may be unborn or

unascertained when the trust is created.

a. Eg: A trust created by O, who is childless, for the benefit of her future children

would be valid.

ii. Beneficiaries MUST be ascertainable (except unborn children): “Ascertainable” means that you

must be able to identify the beneficiaries by name. If their names are not expressly set forth in

the trust, the trust must contain a formula or description of the beneficiaries that permits the

court to determine by objective means who they are.

1. Policy: The requirement of ascertainable beneficiaries ensures that it is possible to

determine who has standing to come into court and hold the trustee accountable.

2. Clark v. Campbell (Ascertainable Trust Beneficiaries) (p. 579)

3. F/P

a. A settlor left certain property in trust to “my friends, as they, my trustees, shall

select.” He also wrote, “Each of my trustees is competent by reason of

familiarity with the property, my wishes and friendships, to wisely distribute

some portion at least of said property. The court determines whether the trust

must and does sufficiently identify the beneficiaries of the trust.

4. I

a. Whether the bequest for the benefit of the testator’s “friends” must fail for the

want of certainty of the beneficiaries?

b. Does the clause provide for definite and ascertainable beneficiaries so that the

bequest can be sustained as a private trust?

5. R/A/H

a. HELD 1: Trust fails – though the instrument indicated a clear intent to create a

trust, the trust failed for want of ascertainable beneficiaries.

Chapter 8 – Trusts: Introduction and Creation 87 of 120

b. HELD 2: No—the document does not provide an objectively ascertainable

formula for identify who the beneficiaries should be.

c. RULE: There must be a beneficiary or a class of beneficiaries indicated in the

will capable of coming into court and claiming the benefit of the bequest. This

principle applies to private trusts, but not to public trusts or charities.

d. Even though the settlor described the class of beneficiaries as his friends and

stated the trustee would know who they were, there were no guidelines to

eliminate separate who might be beneficiaries from those that the settlor

intended except what the trustee believed. Therefore the trustee was given

wide discretion and the beneficiaries were not identifiable.

6. Identifying Beneficiaries Using Familial Terms: Courts routinely hold that familial terms

such as “children, issue, nephews, and nieces” are objectively ascertainable. Some

courts have even held that the terms “relatives” and “relations” refer to one’s heirs

under the state’s descent and distribution scheme and thus are objectively

determinable

7. Power of Appointment: Where there is a transfer in trust for members of an indefinite

class of persons (such as to testator’s “friends”), no enforceable trust is created, but the

transferee (trustee) has a discretionary power to convey the property to such members

of the class as he may select.

a. Key Differences between Trust and Power of Appointment: (1) with power of

appointment, the holder owes no fiduciary duty to the possible appointees

(unlike trusts); (2) exercise of the power is purely discretionary; (3) the possible

appointees of a power (analogous to trustees) do not have to be ascertainable;

they just must reasonably meet the description set forth in the power.

i. See 755 ILCS 5/4-2 (blue sup p. 12)

b. Transforming a Failed Trust into a Power: Prof. Scott (top trust law scholar)

argued that anytime a trust in favor of an unascertainable class failed, the trust

should automatically be transformed into a power of appointment.

i. Restatement Adopts Scott’s View: See R.3d Trusts 46 – but note, only

“a handful” of courts have adopted the rule.

c. Application to Clark v Campbell: In Clark, the court ruled that use of the terms

“trustees” and “in trust” evidenced the intent to create a trust, not a power. If

the testator had not used those terms, the more likely construction would have

been that the testator intended to create a power.

8. Honorary Trusts: In an honorary trust, the transferee is not under a legal obligation to

carry out the settlor’s stated purpose, hence the qualifier, “honorary.” But if the

transferee declines, she is said to hold the property upon a resulting trust, and the

property revers to the settlor or the settlor’s successors.

a. Rule: If a trust would otherwise fail for want of ascertainable beneficiaries, but

the purpose is specific and honorable (and not capricious or illegal), the trust

may continue as long as the “trustee” is willing to honor the terms of the

“honorary trust.” If the trustee stops honoring the terms of the honorary trust,

he/she is not permitted to keep the property. In that case, a resulting trust is

imposed, and the property is ordered distributed to the proper takers.

b. Trusts for the benefits of a pet, eg: Trusts for the benefits of a pet or to

maintain one’s gravesites, while honorable, technically fail for want of

ascertainable beneficiaries.

c. In re Searight’s Estate (Honorary Trusts) (p. 582)

d. F/P

i. George Searight’s (testator’s) will left his dog, Trixie, to Florence Hand

(trustee), and he directed his executor to deposit $1,000 in the bank

to be used to pay Florence 75 cents a day to care for the dog for its

life. The

Chapter 8 – Trusts: Introduction and Creation 88 of 120

e. I

i. Is the testamentary bequest for the care of Trixie the dog valid in Ohio

as (a) a proper subject of a so-called honorary trust, and (b) not being

in violation of the rule against perpetuities?

f. R/A/H

i. HELD: Yes, the bequest is a valid honorary trust.

ii. HELD: No, the trust does not violate the rule against perpetuities.

iii. Because the beneficiary is a dog, there is no beneficiary capable of

enforcing the trust. However, because the trust was for a specific,

honorable purpose (to take care of the specific dog, Trixie), and not a

capricious or illegal purpose, the trust was a valid honorary trust, and

could continue as long as Florence was willing to honor the terms of

the trust (i.e. to use the money to care for Trixie).

iv. The trust does not violate the Rule against Perpetuities because the

trust principal ($1,000) would be exhausted within 3-5 years.

g. The Rule Against Perpetuities: If the administration of the trust can continue

for longer than the maximum period allowed under the Rule of Perpetuities,

the trust is invalid from the moment of its attempted creation.

i. Common Law Rule: An honorary trust for a non-charitable purpose is

void if it can last beyond all relevant lives in being at the creation of

the trust + 21 years. Even if the trust is for a pet animal, the pet’s

lifetime is NOT considered “relevant” (e.g. even if a pet bird lives for

80 years, the bird’s lifetime is not part of “relevant” lifetimes).

ii. Honorable Trusts vs Charitable Trusts: Charitable trusts are NOT

subject to the rule against perpetuities. But most Honorary Trusts ARE

subject to the rule against perpetuities, because though they are

“honorable,” they are usually not “charitable.” i.e. Charitable trusts

have to be for the good of society, but most Honorary trusts specify a

specific animal or a specific gravesite, not “all dogs” or something like

that.

iii. Statutory Purpose Trusts (Majority Approach): Most states have

statutes that permit a trust for a pet animal or other non-charitable

purpose for a given amount of time, and for the perpetual care of a

grave site. These statutes are based on the Uniform Trust Code (UTC

408-409) or Uniform Probate Code (UPC 2-907).

e. The Necessity of a Written Instrument:

i. Inter Vivos Trusts:

1. Common Law: Under the common law approach, the requirement for a written trust

instrument is governed by the Statute of Frauds and the Wills Act formalities not trust

law.

a. No Orally Created Trusts Involving Real Property: Under the Statute of Frauds,

the terms of an oral trust involving real property cannot be enforced against

the transferee because oral conditions are not permitted to vary the terms of a

deed.

2. Modern/Majority approach: An inter vivos oral declaration of trust is enforceable.

R.3d of Trusts 20. An oral trust of personal property need not be evidenced by a trust

instrument, but the creation of an oral trust and its terms may be established only by

clear and convincing evidence. (UTC 407)

a. Real Property: Restatement imposes a constructive trust and orders the

purported trustee to distribute the real property to the intended beneficiaries

outright (often the settlor is the intended beneficiary, so the constructive trust

ends up looking like a resulting trust).

Chapter 8 – Trusts: Introduction and Creation 89 of 120

3. Remedial Trusts: The Statute of Frauds does not apply to the remedial trusts

(constructive or resulting trusts) because they are not true trusts in the full sense of the

word, but rather are judicial remedies that arise by operation of law. Under the

remedial trusts, the original trust is not upheld and enforced; the purported trustee is

simply ordered to transfer the real property in question immediately to the appropriate

party or parties.

4. Unclean Hands: If the settlor created the oral trust for real property for improper

reasons (the settlor was trying to hide assets from creditors or from an impending

divorce, etc.), even under the Modern Trend, the courts generally will not help one who

has “unclean hands.” The purported trustee will be permitted to keep the real property

free of any trust.

ii. Testamentary Trusts: Testamentary trusts (usually created in a testator’s will) must be in writing

pursuant to the Wills Act formalities. Where a testamentary trust fails for want of a writing, the

issue is whether the relief should be a constructive trust or a resulting trust. The answer turns on

whether the failed testamentary trust is deemed a secret trust or a semi-secret trust.

1. Testamentary Trusts vs Inter Vivos Trusts:

a. RULE: A trust where the trustee is to deliver personal property at the settlor’s

death, if funded inter vivos, is a an inter vivos trust and does not have to be in

writing.

2. In re Estate of Fournier (Oral Creation of Trust for Personal Property) (p. 589)

3. F/P

a. Fournier gave $400,000 in cash to a couple he was friends with and asked them

to hold it until he died and then to deliver it to his sister Fogarty. He had two

other sisters, Flanigan and Rose, but he explained to the couple that Fogarty

needed the money and the others did not.

b. He died testate, with a will leaving the residue of his estate equally to Fogarty,

Flanigan, and his nephew, Curtis King. Fogarty met with the couple and

received the money. Fogarty petitioned the court for declaratory judgment to

establish that during his lifetime, Fournier had created an oral trust for her

benefit.

c. The trial court reasoned that because Fournier had told Flanigan’s daughter

about the money, he must have intended it to pass through probate, and that

Fogarty was to take the money as personal representative of Fournier.

4. I

a. Had the decedent created an inter vivos oral trust for the benefit of Fogarty?

5. R/A/H

a. HELD: Fournier had created an oral trust for the benefit of Fogarty, in which

the couple was supposed to hold money for Fogarty until Fournier’s death.

b. RULE: Although a trust need not be in writing, the creation of an oral trust

must be established by clear and convincing evidence.

c. The court ruled there was clear and convincing evidence he intended to create

an inter vivos trust for Fogarty. Primarily, the testimony of the couple showed

that Fournier intended for Fogarty to take the money.

d. EPILOGUE: After the initial ruling, Flanigan found a handwritten note in

Fournier’s hosue, written by Fournier and dated after he gave the money to the

couple, that referenced the $400,000 and provided that the money was to

“reimburse” Flanigan, Fogarty, and King – but there was a line through King’s

name. The court ruled that the note tracked the decedent’s residuary clause,

and the note sufficiently evidenced that the inter vivos trust was for all three.

6. Secret Trust: A secret trust is a testamentary trust that fails because, on the face of the

will, the secret trust looks like an outright gift to a devisee—it does not indicate intent

to create a transfer to the ‘devisee’ as a trustee, for the benefit of someone else.

Chapter 8 – Trusts: Introduction and Creation 90 of 120

a. Evidence: Courts admit extrinsic evidence that the devisee was to take

property as a trustee, not as a devisee

b. Majority/Traditional Remedy: The courts remedy unjust enrichment by

imposing a constructive trust for the benefit of the intended beneficiary (see

also R.3d Trusts 18, R.3d Restitution 46)

7. Semi-Secret Trust: A semi-secret trust is a testamentary trust that fails because,

although it identifies a trustee (i.e. it expressly states that the devisee was not intended

to take the property for his/her own benefit), it does not identify a beneficiary.

a. Evidence: Courts DO NOT admit extrinsic evidence to identify the intended

beneficiaries

b. Majority/Traditional Approach: Courts impose a resulting trust and give the

property back to the settlor/testator. Typically, the property falls into the

residuary clause of the will. But if the failed testamentary trust was the

residuary clause, then the property falls to intestacy.

c. Minority/Modern Approach: Constructive trust should be imposed in favor of

the intended beneficiary (see also R.3d Trusts 18, R.3d Restitution 46)

d. Oliffe v. Wells (Secret/Semi-Secret Trusts) (p. 593)

e. F/P

i. Ellen Donovan created a will leaving the residuary estate to the

defendant and wrote, “to distribute the same in such manner as in his

discretion shall appear best calculated to carry out wishes which I have

expressed to him or may express to him.” Wells (defendant) was also

named as executor. Donovan’s heirs brought suit, claiming that the

residue should be distributed to them.

ii. The defendant stated in his answer to the lawsuit that Donovan had

orally expressed to him before and after the execution of the will that

her estate be used for charitable purposes. He also stated that he

desired and intended to distribute the residue of the estate for those

purposes.

f. I

i. Did the will in question create a valid trust, or did the trust fail (leaving

a secret or semi-secret trust)?

g. R/A/H

i. HELD: The trust is too indefinite to enforce; the will indicates that the

devisee take property as trustee, but the beneficiaries cannot be

identified.

ii. RULE: A trust that is not sufficiently declared on its face to be a trust

cannot be used to defeat the rights of heirs at law by extrinsic

evidence of a trust

iii. On the face of the will, the residuary bequest to the defendant gives

him no beneficial interest (i.e. the heirs have the beneficial interest—

the will does not exclude them). As for Wells’ duty as a trustee, the

will declares a trust too indefinite to be carried out, and the testatrix’s

next of kin must take by way of resulting trust.

8. Oral Inter Vivos Trusts of Land: Where S conveys land to T upon an oral trust to pay the

income to B for life, and upon B’s death to convey the land to R, the Statute of Frauds in

virtually every state prevents enforcement of the express trust.

a. Remedy for Transfers Wrongly Obtained: If the transfer was wrongly obtained

by (1) fraud or duress; (2) where the transferee was in a confidential

relationship with the transferor; (3) where the transfer was made in

anticipation of the transferor’s death, then the courts will impose a

constructive trust for the beneficiaries.

b. See also Hieble and Pappas, p 596 (casebook)

Chapter 9 – Rights to Distributions from the Trust Fund 91 of 120

10. See Also IL Statutes

a. 760 ILCS 5/1 to 21

Chapter 9 – Rights to Distributions from the Trust Fund 1. Rights of the Beneficiary to Distributions

a. Mandatory Trust: Trustee must make specified distributions to an identified beneficiary

i. Overview: Mandatory trusts and mandatory interests cause relatively few problems because the

trustee either performs pursuant to the mandatory terms or the trustee does not.

b. Discretionary Trust: Trustee has discretion over distributions. The beneficiary has no right to receive

payments of income and/or principal—any such payments are at the discretion of the trustee (typically

according to a standard set forth in the express terms of the trust). The trustee’s discretion can also be

limited by coupling it with an express standard (e.g. a discretionary support trust).

i. Settlor’s Responsibilities: Settlor can postpone and delegate to the trustee the decisions of to

whom to make distributions, in what amounts, and when.

1. Settlor’s Purpose: Settlor may provide or purpose or standard that the trustee must

keep in mind when exercising his/her discretion.

a. RULE: The standard MUST be set forth in the express terms of the trust.

b. Implications for Trustee: The trustee’s performance is measured against the

standard set forth in the will.

ii. Trustee’s Duties:

1. Fiduciary Obligations: (See Chapter 10, p. 105)

2. Duty to Decide: The trustee MUST exercise his/her discretion pursuant to the terms of

the trust, even if that decision is not to make a payment to the beneficiary. In assessing

the trustee’s decision-making process, the courts have kept in mind the fiduciary duty

that the trustee owes the beneficiary

3. Duty to inquire: Consistent w/ the fiduciary duty the trustee owes the beneficiary,

before the trustee can exercise discretion wrt whether to make a payment to the

beneficiary, the trustee has a duty to inquire as to the beneficiary’s status and needs.

This includes a duty of due diligence in attempting to gather the relevant information,

and a duty to follow up on unsuccessful attempts. If the trustee fails to inquire, the

trustee is deemed to have breached his/her fiduciary duty to the beneficiary.

4. Duty to Act Reasonably: Objective standard to act as a reasonable trustee would act.

a. Extended Discretion: Some trusts authorize the trustee to act in his/her “sole

discretion,” or “sole and absolute discretion.” But courts construe this as not

TRULY meaning sole/absolute discretion—trustees MUST still have a fiduciary

duty. W/o fiduciary duty, the trust would fail. Such language virtually

eliminates the duty to act “responsibly,” but the trustee must still act in good

faith. (UTC 814)

5. Duty to Act in Good Faith: Subjective standard—the trustee acts in good faith as long

as he/she honestly thought that he/she was acting in the best interests of the

beneficiaries and the trust in making decisions.

6. Duty to Consider a Beneficiary’s Other Resources: Whether a trustee should/must

consider a beneficiary’s other resources in deciding whether to make a payment to the

beneficiary is a question of settlor’s intent—i.e. did the settlor intend to provide a

minimum level of income for the beneficiary, regardless of the beneficiary’s other

sources of income, or did the settlor merely intend to provide a ‘safety net’?

a. Presumption of ‘Steady Income’: Where the settlor’s intent is unclear, most

courts presume that the settlor intended to provide for beneficiary regardless

of the beneficiary’s other resources—the trustee is not to consider other

resources UNLESS the trust expressly authorizes it.

7. Marsman v. Nasca (Discretionary Trusts – Trustee’s Breach of Duty) (p. 598)

Chapter 9 – Rights to Distributions from the Trust Fund 92 of 120

8. F/P

a. Sara Marsman created a testamentary trust that provided that the trustees

were to pay the income to her husband (Cappy) at least quarterly, and “after

having considered the various available sources of support for him, my trustees

shall, if they deem it necessary or desirable…, in their sole and uncontrolled

discretion, pay over to him, …such an amount of the principal thereof as they

deem advisable for his comfortable support and maintenance.”

b. During Sara’s life, Sara and Cappy lived well. After Sara died, Cappy lost his

employment, and his standard of living fell substantially. The principal of the

trust was $65,000. When Cappy brought his plight to the trustee’s attention,

the trustee gave Cappy a minimal distribution of principal ($300) and asked

Cappy to explain in writing the need for the principal. Cappy failed to reply,

and the trustee failed to follow up. For approximately ten years, the trustee

never inquired into the plaintiff’s financial state and only paid him $300 during

the nearly 10 year time span. The plaintiff experienced financial difficulty due

to his failing business during the nearly 10 year period

9. I

a. Whether a trustee that holds discretionary power to pay principal for the

“comfortable support and maintenance” of a beneficiary, has a duty to inquire

into the financial resources of the beneficiary to determine his needs?

10. R/A/H

a. HELD: The trustee had breached the duty to inquire into/follow up on the

financial status of the beneficiary.

b. The will that gave the trustee the power to pay principal for the support and

maintenance of the beneficiary had a duty to inquire into the financial

resources of the beneficiary. Despite the broad discretion in the trust, Cappy’s

standard of living had been reduced substantially. In light of the settlor’s intent

that the principal was to be used to maintain Cappy’s comfortable support and

maintenance, the trustee had breached the duty.

c. A trustee who is given discretionary power to pay the principal for the

beneficiary’s support and maintenance must inquire into the financial

resources of the beneficiary

11. Exculpatory Clauses: Discretionary clauses often include an exculpatory clause

protecting the trustee against liability for breach of trust absent “willful neglect” or the

like.

a. Enforceability of Exculpatory Clauses:

i. Abuse of Confidential Relationship: If the ct concludes that the

exculpatory clause was put in the trust because of the trustee’s

overreaching or abuse of fiduciary or confidential relationship, the

clause will be deemed null and void.

ii. Intentional, Bad Faith Breaches: If the ct concludes that the breach of

trust was intentional, in bad faith, or in reckless disregard for the

beneficiary’s interest, the clause will not be enforced.

iii. Traditional/General Rule: A beneficiary challenging the validity of an

exculpatory clause bears the burden of proof (i.e. burden of proving

intentional or bad-faith breach)

iv. UTC Approach: If the trustee drafted the exculpatory clause or caused

it to be drafted, the clause is presumed to be invalid.

1. Rebuttable Presumption: If the trustee proves the

exculpatory clause is fair under the circumstances and that its

existence and contents were adequately communicated to

the settlor, then the clause is valid (UTC 1008(b)).

Chapter 9 – Rights to Distributions from the Trust Fund 93 of 120

v. New York Approach: Exculpatory clauses granting immunity to

trustees to failure to exercise reasonable care are void, because they

violate public policy.

vi. Example of Enforceability: In Marsman above, the TRUSTEE inserted

an exculpatory clause into the trust, protecting the trustee as long as

he didn’t commit “willful neglect or default.” The trustee testified that

he discussed the clause with the settlor during the drafting process,

and the settlor approved the clause.

1. Valid Exculpatory Clause: though the trustee drafted the

clause and suggested the clause, the ct held that the clause

was valid (UTC approach). The ct said the trustee’s conduct

constituted a breach of duty, but the breach was not

intentional, in bath faith, or in reckless disregard for the

beneficiary’s interest.

12. Mandatory Arbitration Clause: Increasingly, trusts contain mandatory arbitration

clauses

a. Policy: Minimizes administration fees and expedites the resolution of disputes

b. Rule: Cases are scarce – Schoneberger v. Oelze (p. 609) allowed the

beneficiary to litigate in court in spite of the trust’s mandatory arbitration

provision

i. But – An argument can be made that the clauses should be subject to

the same analysis as exculpatory clauses, and should not be per se

invalid.

13. Sprinkle/Spray Trust: A sprinkle or spray trust requires the trustee to distribute the

property in question to a group of individuals, and the trustee has discretion as to whom

to make the payments and how much each receives.

a. Hybrid Trust: To the beneficiaries, the trust is discretionary (they don’t know

who will receive how much…). To the trustee, the trust is mandatory (they

MUST distribute the property; but as to whom and how much, the trust is

discretionary).

14. Unitrust: Under a unitrust, a life beneficiary is given a fixed annual percentage interest

in the total worth of the trust, regardless of whether the property needed to satisfy that

fixed interest comes from the income or the principal. The trustee is then free to

pursue any investment that he/she thinks will produce the greatest benefit for the trust,

regardless of the amount of income the investment produces, because the trustee has

the power to disburse income and principal.

15. Perpetual Dynasty Trust: An increasing number of jurisdictions are abolishing the Rule

against Perpetuities. This permits a trust to last forever. Where a settlor creates a trust

that will last forever for the benefit of one’s issue, the trust is called a Perpetual Dynasty

Trust.

a. Distribution: The trustee has discretionary powers over both the income and

the principal; thus the trustee may manage the trust to allow the corpus to

remain intact, if not grow, while creating a stream of income for the settlor’s

descendants.

2. Rights of the Beneficiary’s Creditors: Some trusts offer asset protection features. 3 main ones: (1) Discretionary

trusts, (2) spendthrift trusts, and (3) self-settled asset protection trusts.

a. Creditors’ Rights Generally: A creditor can reach a debtor’s property as long as the property interest in

question is transferable. Absent a special provision in a trust, generally a beneficiary’s interest in freely

transferable, whether the beneficiary’s interest is beneficiary or mandatory. i.e. the creditor can reach a

beneficiary’s interest in the trust.

i. Scope of Creditors’ Rights: Generally, a creditor only receives whatever interest the beneficiary

has in the trust—no more, no less.

Chapter 9 – Rights to Distributions from the Trust Fund 94 of 120

1. Mandatory Trusts: The creditor can force the trustee to distribute the income to the

creditor pursuant to the terms of the trust, just as the beneficiary could have.

b. Discretionary Trust: Two types of discretionary trusts: (1) Pure Discretionary Trusts, and (2) Support

Trusts

i. Pure Discretionary Trust: Trustee has absolute, sole, or uncontrolled discretion over

distributions to the beneficiary

1. Traditional Approach: Creditor has no recourse against the beneficiary’s trust interest.

Creditor cannot compel the trustee to pay him.

a. Policy: Because the beneficiary has no right to compel distribution, neither

does the beneficiary’s creditor (R. 2d Trusts 155, cmt b)

b. But Court Order: Even though the creditor cannot compel the trustee to satisfy

a debt of the beneficiary if the trust is discretionary, the creditor MAY be

entitled to a court order directing that when the trustee does decide to pay the

beneficiary, the trustee must pay the creditor before making any paying the

beneficiary. (Hamilton v. Drogo, p. 610)

2. Modern Approach: Both the R.3d Trusts and UTC collapse the distinction between

discretionary trusts and support trusts. (see below, p. 94)

ii. Support Trust: Trustee is obligated to make distributions as necessary for the beneficiary’s

needs (e.g. support/education).

1. Traditional Rule: The beneficiary of a support trust cannot alienate her interest.

Therefore, creditors of the beneficiary cannot reach the beneficiary’s interest.

a. Basic Necessities Exception: Suppliers of necessaries may recover through the

beneficiary’s right to support (R.2d Trusts 154).

b. Child Support and Alimony Exception: Children and spouses may enforce

claims for child support and alimony against a beneficiary’s interest in a

support trust (R.2d Trusts 157).

2. Construction of Support Trust Language: The key to classifying a trust as a support

trust is the formula that controls how much the trustee can distribute to the beneficiary,

not the use of the word “support” per se.

a. Excample: Where the payment IS limited to the amt necessary for

support/education, the trust qualifies as a support trust. If the trustee is

required to distribute ALL of the income to the beneficiary for his/her support,

the trust is not a support trust because the amount being paid out is not

limited to the amount necessary for the beneficiary’s support.

3. Discretionary Support Trust: Not recognized as a separate category, but still common:

Combines an explicit statement of unfettered discretion with a distribution standard

(e.g. see Marsman, above p. 91).

a. Creditors’ Rights: Wrt to creditors’ rights, courts tend to treat these more as

pure discretionary trusts than as support trusts.

iii. Modern/UTC Approach to Creditors’ Rights: Modern trend and UTC is to abolish the distinction

btwn discretionary and support trusts. Creditors may reach the beneficiary’s trust interest, but

the Restatment and UTC take different approaches.

1. R.3d Trusts 60: “if the terms of a trust provide for a beneficiary to receive distributions

in the trustee’s discretion, a transferee or creditor of the beneficiary is entitled to

receive or attach any distributions the trustee makes or is required to make in the

exercise of that discretion.

2. UTC 504: Subject to an exception for claims by children and spouses for child support

and alimony, a creditor of a beneficiary cannot compel a discretionary distribution, even

if the beneficiary could compel such a distribution. (see casebook p. 612)

iv. Protective Trust: A protective trust is a hybrid asset-protecting trust. The beneficiary’s interest

is mandatory until a creditor attaches a beneficiary’s interest (i.e. because beneficiary falls into

debt). At that point, the beneficiary’s interest automatically becomes discretionary (to protect it

against the claim of the creditor).

Chapter 9 – Rights to Distributions from the Trust Fund 95 of 120

1. Favorable in Jurisdictions that don’t recognize Spendthrift Clauses: Protective trusts

are popular in jurisdictions that don’t recognize spendthrift clauses, and they are the

norm in England (where spendthrift clauses are not recognized—see p. 614).

c. Spendthrift Trust: A spendthrift clause bars a beneficiary’s ability to transfer his/her interest voluntarily

(by sale or gift) or involuntarily (by creditors reaching it). (R.3d Trusts 58)

i. Policy Issue: The spendthrift trust arises from the settlor’s right to condition the terms of her

transfer. The issue becomes whether or not to allow the ‘dead hand’ to restrain alienation of the

beneficial interest.

ii. Majority Rule: A trust is not spendthrift (nontransferable) unless the settlor expressly inserts a

spendthrift clause

iii. New York Rule: A beneficiary’s interest is presumed spendthrift (nontransferable) unless the

trust expressly provides otherwise.

iv. England Rule: England don’t be recognizin’ spendthrift clauses at all, yo. Word, son.

v. UTC Rule: A spendthrift provision is valid ONLY IF it restrains both voluntary AND involuntary

transfer of a beneficiary’s interest (UTC 502(a))

vi. Enforceability of Spendthrift Trusts: Generally, spendthrift clauses are valid and enforceable,

even as applied to remaineder interests in trust

vii. Exceptions: Although spendthrift clauses are generally valid, exceptions apply:

1. Judicial Exceptions: Public Policy � certain categories of creditors are not subject to

spendthrift clauses: (1) ex-spouses entitled to spousal support (alimony); (2) children

entitled to child support; (3) creditors who provide basic necessities; (4) tax claims by

the state or federal govt

a. But Tort Creditors Are Screwed in Most Jurisdictions: Majority of jurisdictions

still apply spendthrift clauses to tort creditors

2. UTC Approach: UTC limits exceptions to (1) children entitled to child support pursuant

to a judgment or court order; (2) spouses/ex-spouses entitled to spousal support

(alimony/maintenance) pursuant to a judgment or court order; (3) a claim by a

state/federal govt; and (4) a judgment creditor who has provided services for the

protection of a beneficiary’s interest in the trust. (UTC 503, see CB p. 620)

a. See also R.3d Trusts 59(a): Agrees w/ UTC § 503.

b. Minority (substantially small) Approach: Spouse/child cannot reach a

spendthrift trust to satisfy a judgment for support.

3. Scheffel v. Krueger (Spendthrift Trusts and Tort Creditors) (p. 616)

4. F/P

a. Krueger’s grandmother set up a trust for his benefit. He had a mandatory

interest in the income, discretionary interest in the principal (for his

maintenance, support, and education), and the trust had a spendthrift clause.

b. Some time later, Krueger sexually assaulted a minor, videotaped it, and

broadcast it over the internet. He was found liable for several sexual assault

charges and also faced criminal charges. He also had a tort judgment of over

$500,000 entered against him.

5. I

a. Whether a trust purpose for support and maintenance may still be fulfilled

where the beneficiary faces a criminal sentence for sexual assault.

b. Whether a statute that bars creditors from claiming an interest in a

beneficiary’s trust makes an exception for tort creditors.

6. R/A/H

a. HELD 1: Yes – the purpose of support and maintenance trust may still be

fulfilled while the beneficiary is incarcerated and after he is released.

b. HELD 2: Yes – the spendthrift clause barred reaching Krueger’s interest in the

trust.

c. RULE: A statute that bars creditors from claiming an interest to a beneficiary’s

trust does not make an exception for tort creditors.

Chapter 9 – Rights to Distributions from the Trust Fund 96 of 120

7. Shelley v. Shelley (Spendthrift Trusts and Child Support Creditors) (p. 618)

8. F/P

a. The settlor created a trust for the benefit of his son. The son’s interest in the

income was to be mandatory (income to be paid for life). The trustee was to

begin distributing corpus to the son after he reached age 30 in amounts that

the trustee (and other named persons) deemed the son to be able to invest

properly. The trust also authorized the trustee to disburse principal to the son,

or his children, in an emergency. The trust had a standard spendthrift clause.

b. The son owed both alimony and child support payments to his ex-spouses and

children. When the son disappeared, the ex-spouses and children sued seeking

to reach the son’s interest in the trust.

9. I

a. Whether a spendthrift provision bars a beneficiary’s former spouse and child

from collecting alimony and child support?

10. R/A/H

a. HELD: No – The creditors were not subject to the spendthrift clause. The

children were entitled to payments of the principal in their capacity as

beneficiaries of the trust, not as creditors.

b. RULE: A spendthrift provision does not bar the claims of a beneficiary’s

children and former spouse for child support and alimony in regards to the

income of a trust. However such claims are barred in regards to discretionary

payments from the trust corpus.

c. The spendthrift clause does not bar the ex-spouses and children from reaching

the son’s mandatory interest in the income.

d. Moreover, the ex-spouses and children were also entitled to reach the son’s

interest in the principal (corpus). But because the son’s interest was

discretionary, they could not force the trustee to make disbursements to them

as creditors. However, because the children were beneficiaries of the principal

in their own right, the trustee’s failure to disburse principal to them as

beneficiaries was an abuse of discretion

viii. Statutory Limitations on Creditor Protection: Some states statutorily limit the amount of the

beneficiary’s interest that can be protected against creditor’s claims by a spendthrift clause.

1. There are 3 statutory main approaches:

a. (1) Limit the amt of a beneficiary’s interest that can be shielded from creditors’

claims by a spendthrift clause to the amount necessary for the beneficiary’s

support and education; allow the creditor to reach the rest

b. (2) Permit a creditor to reach a fixed percentage (usually less than 1/3) of a

beneficiary’s interest in the income;

c. (3) Fixed dollar amount cap on the amt of money that can be shielded from

creditors’ claims by a spendthrift clause; allow the creditor to take the rest

2. Pension Trusts and the Employee Retirement Income Security Act (ERISA): ERISA

requires that each pension plan covered by the act provides that may not be

transferred. Current, non-family creditors MAY NOT reach the beneficiary’s interest.

However, the benefits MAY be reached for child support, alimony, or marital property

rights.

3. Bankruptcy Law and Trust Asset Protection: The Bankruptcy Code provides that a

beneficiary’s interest in a trust passes to the bankruptcy trustee ONLY IF the

beneficiary’s interest is transferable. If the trust has a spendthrift clause, the

beneficiary’s interest is not reachable in bankruptcy.

d. Creditors’ Rights when Settlor is the Beneficiary (Self-Settled Asset Protection Trusts): The above listed

creditors’ rights scenarios assumed that the beneficiary was a different person than the settlor. Here, the

beneficiary IS the settlor (and the trustee is someone else)

Chapter 9 – Rights to Distributions from the Trust Fund 97 of 120

i. Traditional Rule = the settlor cannot use a trust to shield his own assets from his own creditors.

Even if the trust is discretionary, spendthrift, or both, the settlor’s creditors can reach the

maximum amt that under any circumstances the trustee could pay to the settlor or apply for the

settlor’s benefit. (Traditional Rule is carried forward in R.3d Trusts 58(2), 60, cmt f) and UTC

505).

1. Mandatory Interest: If the settlor retained a mandatory interest in the trust, creditors

of the settlor can reach the mandatory interest in the trust. If the trustee fails to make

the payment to them, the creditors can compel the trustee to make the payment to

them

2. Discretionary Interest: If the settlor retained a discretionary interest in the trust,

creditors of the settlor can reach the discretionary interest to the full extent that the

trust permits the trustee to use the trust for the benefit of the settlor

3. Spendthrift Clause: Generally, spendthrift clauses are null and void as applied to

creditors of a beneficiary who is also the settlor. It is against public policy to permit one

to shield one’s assets in a spendthrift trust.

ii. Modern Trend: In an attempt to attract trust business some states have adopted statutes

authorizing self-settled Asset Protection Trusts (APTs)

1. Alaska/Delaware: Settlor’s creditors have no recourse against the settlor’s interest in a

self-settled spendthrift trust, as long as the initial transfer was not fraudulent

a. Rules: APTs are permitted in favor of a beneficiary who is also the settlor if (1)

the trust is irrevocable, (2) the trust interest is discretionary, and (3) the trust

was not originally created to defraud creditors

b. Locale: Settlors in any state can take advantage of these exceptions as long as

the trusts are created in the states that allow such trusts.

c. This approach is followed in 11 states: AK, DE, MO, NH, NV, OK, RI, SD, TN, UT,

WY

i. ILLINOIS DOES NOT ALLOW APTs

2. Offshore: e.g. Bahamas, Barbados, etc. islands allow self-settled trusts against which

settlors’ creditors have virtually no recourse

iii. Federal Trade Commission v. Affordable Media, LLC (Self-Settled Asset Protection Trusts) (p.

628)

iv. F/P

1. Denyse and Michael Anderson created a telemarketing scheme that constituted a Ponzi

scheme to defraud investors of millions of dollars, which they tucked away in a “Cook

Islands” trust (an offshore self-titled spendthrift trust). The Andersons were co-trustees,

along with AsiaCiti Trust Ltd.

2. The FTC brought suit against the Andersons to recover as much money as possible for

the defrauded investors and was granted a preliminary injunction ordering the

Andersons to account for the funds held offshore and to repatriate the funds. The

Andersons faxed AsiaCiti for an accounting and to repatriate the funds to the states, but

AsiaCiti concluded that the district court’s preliminary injunction constituted duress

voiding the Anderson’s request, and under the terms of the trust, authorized AsiaCiti to

remove the Andersons as co-trustees.

3. The dist ct held the Andersons in civil contempt for failure to account and repatriate the

funds and took them into custody.

4. As a general rule, a party’s inability to comply with a judicial order constitutes a defense

to a charge of civil contempt. The Andersons argued that the terms of the trust

prevented them from complying with the court order. The court expressed skepticism

as a legal matter as to the applicability of the defense where the inability to comply is

the intended result of the defendant’s own actions (here, the creation of the offshore

APT to protect the assets from the reach of US courts).

5. The court ruled that the Andersons had not met their burden of showing “categorically

and in detail” that they were unable to comply with the court order. The court was not

Chapter 9 – Rights to Distributions from the Trust Fund 98 of 120

convinced that the Andersons had put the money beyond their control, particularly

because the Andersons retained the position of “protectors” of the trust. Protectors

have significant control of offshore trusts, including the appointment of successor

trustees and the power to make the anti-duress provisions of the trust subject to their

control.

v. I

1. Whether a party demonstrates categorically and in detail that he us unable to comply

with the repatriation section of a preliminary injunction to transfer to the United States

all assets under their control directly and indirectly because the assets are in trust under

a trustee that refuses to relinquish the proceeds?

vi. R/A/H

1. HELD: No. The defendants were protectors of the trust and could have forced the

trustees to turn over the proceeds. Furthermore, the defendants showed they were

aware of their ability to force the trustees to transfer the money. After they stated that

they could not comply with the order, the Commission revealed that the defendants

were the protectors of the trust. Thereafter, the defendants attempted to resign as

protectors of the trust.

2. RULE: A party petitioning for an adjudication that another party is in civil contempt

does not have the burden of showing that eh other party has the capacity to comply

with the court’s order, but the party asserting the impossibility defense must show

categorically and in detail why he is unable to comply. There is a high burden on the

defendant to prove impossibility as a defense to a contempt charge

3. United States courts will penalize trustees of trusts in foreign lands if the trustees are

domiciled or residents of the United States and do not comply with orders in regards to

the overseas funds.

4. EPILOGUE: The district court freed the Andersons six months after finding them in

contempt. The FTC sued AsiaCiti in the Cook Islands for $20 million, but settled for only

$1.2 million

vii. In re Lawrence (p. 634)

viii. F/P

1. Lawrence created and funded an offshore asset protection trust in Mauritius with $7

million dollars. After Lawrence lost a securities law arbitration proceeding, he was

punished with a $20.4 million judgment against him. Lawrence filed for bankruptcy and

the court ordered him to turn over the bankruptcy trustee assets held in the offshore

trust. Lawrence did not comply and the court held him in contempt and jailed him

pending compliance with the turnover order.

ix. I

1. Whether a person charged with contempt is responsible for his inability to comply with

an order to turn over assets in an offshore account if he was removed as a beneficiary

under the account because he declared for bankruptcy?

x. R/A/H

1. HELD: Yes. Lawrence is responsible for his inability to comply because even though he

was removed as a beneficiary as a result of his bankruptcy filing, he still retained the

authority to appoint trustees who could have exercised the authority to reinstate

Lawrence as a beneficiary. The new trustees could have distributed the entire trust to

Lawrence once he was reinstated. Therefore Lawrence is responsible for his inability to

comply. Furthermore, it appears as if the sole purpose of the provision that extinguishes

his inters tint the event of bankruptcy is to aid the settlor in evading contempt.

2. The court did not accept impossibility as a defense here because settlor still had the

power to transfer the assets if he appointed new trustees.

xi. Transfers in fraud of creditors: Asset transfers made with the intent to hinder or defraud a

creditor’s claim (as opposed to transfers made before a creditor’s claim arises) constitute actual

fraud and are recoverable under the widely adopted Uniform Fraudulent Transfer Act (UFTA).

Chapter 9 – Rights to Distributions from the Trust Fund 99 of 120

xii. Professional Responsibility: Lawyers have an ethical obligation not to counsel a client to engage,

or assist a client, in conduct that the lawyer knows is criminal or fraudulent (Model Rule of

Professional Conduct 1.2(d)). Lawyers must be careful in creating an asset protection trust that

would delay, hinder, or defraud a client’s existing or foreseeable creditors.

3. Trusts for the State Supported (eg Public Health Benefits): Public health benefits, such as Medicaid and state-

sponsored health programs, are usually limited to people who cannot pay for their own health services. People

have to use trusts (esp discretionary trusts) to shield their assets from their medical expenses, thereby qualifying

for public assistance.

a. Issue: Whether such trusts are successful turns mainly on whether the applicant who is the beneficiary

contributed to the creation of the trust or whether someone other than the applicant/beneficiary created

the trust.

b. Applicant-Created Trusts (“self-settled trusts”):

i. Definition: For Medicaid purposes, a trust is self-settled if (1) “assets of the individual were used

to form all or part of the corpus of the trust” and (2) the trust was established by the individual,

the individual’s spouse, or by a person or court w/ legal authority to act on behalf of, or by

request of, the person or spouse.

ii. General Rule: Rules favor including the trust property among the applicant’s resources for

determining the applicant’s eligibility for Medicaid and state-sponsored health programs.

iii. Revocable Trusts: The whole trust is considered the applicant’s property for the purposes of

determining eligibility

iv. Irrevocable Trusts: The trust property is considered the applicant’s property to the full extent

that any part of the trust could be used for the applicant’s benefit.

v. Exceptions: Two important exceptions to this rule:

1. Testamentary Discretionary Trust: Testamentary discretionary trusts created by the

applicant’s spouse (by will) are not included as individual assets for Medicaid

2. Trusts Established for a Disabled Individual by a parent, guardian, or court: If (1) the

trust is established for a disabled individual from the individual’s property, by a parent,

grandparent, or guardian of the individual or by a court, and (2) the trust provides that

the state will receive all unreimbursed medical costs upon the applicant’s death.

c. Third-Party Created Trusts: If the applicant had no role in creating the trust, the beneficiary’s interest in

the trust is considered part of the applicant’s resources only to the extent the beneficiary could compel

the trustee to make a payment of income or principal (typically in a mandatory trust, but not in a

discretionary trust).

i. Spendthrift Trusts: As a provider of basic necessities, the govt generally is not subject to a

spendthrift clause.

ii. UTC Approach: Under UTC, providers of basic necessities generally have no right to reach a

beneficiary’s interest in a trust, BUT if the State is the plaintiff, then it can. (UTC 503(b)(3))

iii. Discretionary Trusts: When a trust is a discretionary trust created by a 3rd

party, the state’s

ability to reach the beneficiary’s interest turns on the settlor’s intent generally.

1. General Rule: The state cannot reach discretionary trusts, nor consider them when

determining eligibility for public benefits.

2. BUT – Hybrid Discretionary Trusts w/ Support Purposes: Where the discretionary trust

is a hybrid with a support standard under which the beneficiary could force distribution,

the State can also force distribution

3. Supplemental Needs Trusts: Where the settlor’s intent was to provide benefits only

that the state is unwilling or unable to provide, the state cannot reach the beneficiary’s

interest.

iv. See also IL Statutes

1. 760 ILCS 35/1 (“Trusts and Dissolutions of Marriage Act”)

2. 760 ILCS 15/1 to 17 (“Principal and Income Act”)

3. 760 ILCS 30/1 (“Instruments Regarding Adopted Children Act”)

4. Modification and Termination of Trusts

Chapter 9 – Rights to Distributions from the Trust Fund 100 of 120

a. Introduction: A trust ends naturally pursuant to its terms. A trust ends when all of the trust res is

completely disbursed. A trust’s terms will provide for when the trust res (principal) is to be disbursed.

i. Issue: If the settlor is dead or does not consent, when may the trust beneficiaries modify or

terminate the trust prematurely?

ii. Irrevocable Trusts: If the settlor and all beneficiaries consent, an irrevocable trust may be

modified or terminated. The trustee cannot object. Such a right exists even if the trust contains

a spendthrift clause.

iii. Revocable Trusts: If the trust is revocable, the settlor can single-handedly terminate the trust.

The power to terminate implicitly includes the power to modify—settlor can revoke the trust and

create a new trust with modified terms and conditions.

1. Note: The settlor must comply with the requirements for revoking the trust. The

doctrines of trust modification and termination presume an irrevocable trust.

b. Interest in Trust (and ability to modify/terminate trust):

i. All three parties consent: if all three parties to a trust (settlor, trustee, beneficiaries) agree to

modify or terminate the trust, the trust can be modified or terminated. If any of the parties later

changes his/her mind and sues any of the other parties, the suing party will be estopped based

on his/her initial consent.

ii. Settlor and Beneficiaries Consent (but not trustee): Even if the trustee objects, the trust can be

modified/terminated.

1. The trustee must assert the settlor’s intent. The trustee has no beneficial interest in the

trust.

iii. Trustee and Beneficiaries Consent (but not settlor): Assuming the settlor has no interest in the

trust (an irrevocable trust), if all the beneficiaries consent and the trustee consents, the trust can

be modified/terminated.

1. The beneficiaries will be estopped from suing later for breach of duty, because they

consented earlier to the change/terminate the trust.

iv. Beneficiaries Consent, Trustee Objects (Settlor is Dead): If all the beneficiaries consent, but the

trustee objects and the settlor is dead, the jurisdictions are split.

1. English Approach: If all the beneficiaries consent, the trust is modified or terminated

regardless of the terms of the trust or the trustees’ objections

a. “Dead hand” control generally is not permitted. After the death of the settlor,

the beneficiaries are deemed the owners of the trust property for purposes of

modification and termination of the trust.

2. Traditional American Approach: The trustee has the right (and to some degree, the

duty) to object to a modification or termination. If all the beneficiaries consent, the

trust is modified/terminated regardless of the terms in the trust or the trustee’s

objections.

a. Rationale: By protecting a deceased settlor’s intent as expressed in the trust,

future settlors will be encouraged to create trusts, because they know that as a

general rule, the courts will protect and uphold their intent even after their

death (if the trustee objects).

c. Trust Modification (Deviation and Changed Circumstances):

i. Settlor’s Intent: Under the common law doctrine of modification, the assumption is that the

modification is to further the settlor’s intent.

ii. R.2d Trusts 167 – The court may permit the trustee to deviate from a term of the trust if

circumstances not known to the settlor and not anticipated by him would defeat or substantially

impair the accomplishment of the purposes of the trust

iii. Unforeseen Change: Fact-intensive inquiry.

1. Common Law: Courts tend to: Courts protect settlor’s intent, even against attempts at

modification. Courts apply a rather high threshold for what constitutes an unforeseen

change in circumstances

Chapter 9 – Rights to Distributions from the Trust Fund 101 of 120

2. Modern Trend: There is a noticeable shift toward giving the beneficiaries greater

control over the property in the trust after the settlor’s death (i.e. lower threshold for

what constitutes an unforeseen change in circumstances)

a. Eg: An unusually high rate of inflation or increased medical costs can be

enough to constitute an unforeseen change

3. Beneficiary’s Advantage: The mere fact that the proposed modification would be more

advantageous to one or more beneficiaries is not enough to warrant modifying a trust,

even if all beneficiaries agree.

a. In re Trust of Stuchell (p. 643)

b. F/P

i. The Petitioner sought to modify a trust in regards to one of the

remainder beneficiaries because he is mentally repaired. Without the

amendment, if the disabled beneficiary survived the other two life-

income beneficiaries, he would receive the remainder of funds

outright. The petitioner, with the consent of all of the other income

beneficiaries and remainder men, sought to modify the trust so that it

would not be distributed directly to him if he survived the others, but

rather continue to exist as a secondary source of funds to supplement

the money he receives from pubic assistance.

c. I

i. Whether a trust may be amended where all but one of the

beneficiaries approves and the trust as amended, but only makes the

trust more advantages to the beneficiaries.

d. R/A/H

i. HELD: No. The trust may not be amended where the proposed

amendment would only be more advantageous to the beneficiaries.

Here the proposed amendment would allow the trust to continue only

as a secondary supplement to the current income and benefits that

the beneficiary receives from public assistance.

ii. RULE: A court will not permit or direct the trustee to deviate from the

terms of the trust merely because such deviation would be more

advantageous to the beneficiaries than a compliance with such

direction.

iii. Here the modification only benefited the other beneficiaries. Even

though the handicapped beneficiary would not receive less money

than he was currently receiving through public assistance, the other

beneficiaries would receive more money from the trust.

iv. Substantially Impair: Whether an unforeseen change in circumstances “defeats or substantially

impairs” the settlor’s intent is a very soft, fact-sensitive inquiry

1. Common Law Approach: Protect settlor’s intent: Apply a high threshold for finding that

the change (in circumstances) “defeats or substantially impairs” settlor’s intent.

2. Modern Trend/UTC Approach: Favors granting beneficiaries greater power over the

trust (low threshold for what constitutes “defeating/substantially impairing” settlor’s

intent.

a. UTC: Ct may modify the administrative or dispositive provisions of a trust if,

because of circumstances not anticipated by the settlor, modification would

further the purposes of the trust.

v. Beneficiaries’ Consent: General Rule: Even if an unforeseen change in circumstances defeats or

substantially impairs the settlor’s intent, before a court directs or permits modification, ALL of

the beneficiaries must consent. There are doctrines to get consent from beneficiaries who lack

the capacity to consent or from future beneficiaries who might not be born yet.

Chapter 9 – Rights to Distributions from the Trust Fund 102 of 120

1. Guardian ad litem: Method of getting consent from minors or unborn beneficiaries –

petition the court for an appointment of a guardian ad litem to represent the interests

of the minor or unborn beneficiaries

a. Traditional Approach: Guardians ad litem take a strict/conservative approach

to representing the minor or unborn beneficiary, asking only whether the

proposed modification would increase or decrease the economic value of the

interest the guardian was appointed to protect.

b. Modern Trend: The courts have encouraged guardians ad litem to take into

consideration of non-economic factors, such as family harmony and the

settlor’s apparent primary intent to take care of other family members.

2. Virtual Representation: Some courts and the UTC held that under the doctrine of

virtual representation, if the interests of the minor or unborn beneficiaries are virtually

identical to those of living adult beneficiaries are deemed to speak not only for

themselves, but also for the interests of the minor or unborn beneficiaries by virtual

representation.

3. Modern Statutory Trends: Make it easier to get consent of all beneficiaries by reducing

the pool of beneficiaries who have to consent or by permitting the court to order

modification, even in the absence of all the beneficiaries consenting.

a. UTC: UTC requires the consent of only “qualified beneficiaries” for the removal

of a trustee. A “qualified beneficiary” is one who would be entitled to receive

property if the trust was terminated on the day the petition was filed.

i. UTC Also authorizes the COURT to order modification or termination,

without requiring the consent of all beneficiaries if (1) the trust could

have been modified if all the beneficiaries had consented, and (2) the

interests of the non-consenting beneficiaries are adequately

protected. (UTC 411(e))

b. Modifying Trusts to Achieve Tax Benefits: A handful of states, and both the

R.3d of Property, Donative Transfers and the UTC authorize modification of

trusts to further a settlor’s apparent tax minimizing objectives. (see CB p. 651)

vi. Administrative Modification: Generally, under the unforeseen change in circumstances

doctrine, courts are more willing to modify administrative provisions than they are to modify

distributive provisions.

1. Rationale: Modifying administrative provisions changes only how to administer the

settlor’s intent, while still honoring that intent. Modifying distributive provisions may go

against the settlor’s intent.

2. UTC: UTC authorizes a court to modify the administrative provisions of a trust if

continuing the current administrative procedures would be impractical or wasteful or

impair the trust’s administration. (UTC 412) – see CB p. 645

3. Restatement: R.3d Trusts 66 authorizes courts to modify administrative AND

distributive provisions (see CB p. 645).

4. In re Riddell (Court-ordered Trust Modification) (p. 645)

5. F/P

a. George and Irene Riddell created multiple trusts to benefit their only son Ralph

and his wife Beverly; and upon the death of the latter of them, to provide

benefits to Ralph’s children until they reach the age of 35, when the principal

would be distributed outright to them. Ralph had 2 kids (Donald and Nancy),

both of whom were over age 35. Nancy suffered from schizophrenia and lives

in a state hospital – George and Irene were not aware of this.

b. Ralph was the trustee. Ralph filed a petition to consolidate the trusts and

create a special needs trust for Nancy (instead of disbursing her payment

outright) that would (1) manage Nancy’s funds for her benefit, (2) avoid the

state seizing the funds to be reimbursed for medical expenses, and (3) avoid

Nancy’s mismanagement of the money.

Chapter 9 – Rights to Distributions from the Trust Fund 103 of 120

c. Trial granted motion to consolidate trusts, but not to modify

6. I

a. Should the trustee be allowed to modify the trusts?

7. R/A/H

a. HELD: Ralph (trustee) should be allowed to modify the trusts.

b. RULE: Equitable Deviation (R.3d Trusts 66): The court may modify an

administrative or distributive provision if (1) because of circumstances not

anticipated by the settlor and (2) the modification or deviation will further the

purposes of the trust.

c. The court found that (1) Nancy’s special needs constituted circumstances not

anticipated by the settlors because George and Irene were unaware of them,

(2) the special needs modification will further the purposes of the trust, and (3)

the modification was not against public policy.

d. George and Irene’s intent was not to lose the money to the state and prevent

Nancy from passing the money to her son.

vii. Trust Protectors: A “Trust Protector” can be given powers and control over a trust similar to

those held by the settlor of a revocable trust (e.g. protector may terminate or modify the

administrative or distributive provisions of the trust in response to changed circumstances).

1. Fiduciary Duty: Whether the Trust Protector owes the beneficiaries a fiduciary duty is

up for debate

a. UTC 808(d): A person who has the power to direct the trustee is presumptively

a fiduciary with a duty

i. HOWEVER, this rule is not included in the schedule of mandatory rules

in UTC 105, so it may be overridden by the terms of the trust

b. R.3d Trusts 75: Trust protector is generally a fiduciary

c. Alaska State Law: Trust protector is NOT presumptively a fiduciary

d. Delaware State Law: Trust Protector is presumptively a fiduciary, but the

trustee has no duty to monitor the protector or to notify the beneficiary if the

trustee disagrees w/ protector’s judgment.

d. Trust Termination: Premature termination where all the beneficiaries consent, but the trustee objects,

troubled courts. On one hand, courts wanted to protect settlor’s intent on the theory that the trust, not

the beneficiaries, owned the trust property. On the other hand, the courts were suspicious of the

trustee’s objections because of the trustee’s vested interest in receiving trustee fees.

i. The Claflin Doctrine: Doctrine handles conflict of interest between beneficiaries and trustee.

1. Policy: Favors settlor’s intent more than beneficiaries’ rights.

2. Rule: If continuance of the trust without modification or termination is necessary to

carry out a material purpose of the settlor, then the beneficiaries cannot compel

modification or termination. However, if there is no remaining unfulfilled purpose, and

all the beneficiaries, they can compel modification/termination.

3. Unfulfilled Material Purpose: What constitutes an unfulfilled material purpose is a

question of fact – centers on the language and apparent purpose of each trust.

a. Some trusts inherently have unfulfilled purposes: (1) discretionary trusts, (2)

spendthrift trusts, (3) support trusts, (4) trusts where the property is not to be

disbursed until the beneficiary reaches a specific age.

b. If the court determines that the dispositive provisions of the trust constitute

merely a succession of interests that have no material purpose, premature

termination is ordered, if all the beneficiaries consent.

c. In re Estate of Brown (Trust Termination – Claflin Doctrine/Material Purpose)

(p. 653)

d. F/P

i. Brown created a trust to be used for the education of the children of

his nephew, Woolson S. Brown. Upon completion of that purpose, the

trust income and principal were to be used for the care, maintenance,

Chapter 9 – Rights to Distributions from the Trust Fund 104 of 120

and welfare of his nephew, Woolson S. Brown and his wife Rosemary

Brown, so that they would be able to live in the style and manner to

which they were accustomed for the rest of their lives. Upon the

death of the survivor, the trust res was to be distributed to their then-

living children equally.

ii. When the educational purpose had been fulfilled, all the beneficiaries

to terminate the trust early. The trustee objected.

e. I

i. Whether a trust is a support trust where the trustee must distribute all

of the remainder income to specified beneficiaries after the initial

purpose of the trust is fulfilled?

ii. Whether the material purpose of a trust was fulfilled after the

education of the settlor’s nephew’s children?

f. R/A/H

i. HELD: No—court declined to terminate the trust.

ii. RULE: The Claflin rule (see above, p. 103)

iii. The trustee must pay an amount to the remainder beyond the extent

necessary for their support; the trustee must pay the amount of the

remainder of the trust income to Woolson and Rosemary Brown as is

as needed for them to live in the style and manner to which they are

accustomed for the remainder of their lives.

iv. The material purpose of a trust that provides for the education of a

beneficiary’s children ,and then for the beneficiary and his wife to live

in a lifestyle to which they were accustomed, is not satisfied after the

beneficiary’s child’s education is complete. The settlor did not merely

name successive beneficiaries, but expressed intent to provide for the

lifelong income of the beneficiaries. Therefore this the second purpose

is material

g. Modern Trend/UTC Approach:

i. State Law: Some states have relaxed conditions under which a trust

may be terminated/modified by request of the beneficiaries

ii. R.3d Trusts 65: Beneficiaries may compel the modification or

termination if all of the beneficiaries consent. The modification must

be consistent with a material purpose of the trust; otherwise, the

beneficiaries need (1) the settlor’s consent, or (2) if the settlor is dead,

court authorization. (see CB p. 656)

iii. UTC 411: Court may order

1. (1) termination if all beneficiaries consent and continuance of

the trust is not necessary to achieve any material purpose of

the trust;

2. (2) modification if all beneficiaries consent and modification

is consistent with a material purpose;

3. (3) termination or modification without the consent of all

beneficiaries if:

a. The trust could have been terminated if all the

beneficiaries had consented AND

b. The interests of the non-consenting beneficiaries are

adequately protected

h. Spendthrift Trusts and Material Purpose: In UTC 411(c) and R.3d Trusts 65,

spendthrift clauses do NOT give rise to material purpose. Spendthrift clauses

merely provide some indication that settlor had a material purpose, but they

are not of themselves enough to establish material purpose.

Chapter 10 – Trust Administration: The Fiduciary Obligation

105 of 120

i. Termination of Trusts because of Probate Settlement: Where there is

litigation during probate, and the heirs and trust beneficiaries reach a

settlement that includes terminating the trust:

i. Majority Approach: Most (but not all) courts enforce the settlement

and terminate the testamentary trust, despite its terms (even if there

is an unfulfilled material purpose).

ii. Minority Approach: Book didn’t say – probably honors the trust

j. Trust Revocability vs Irrevocability Default Rule:

i. Majority Rule: A trust is IRREVOCABLE unless the trust expressly

provides otherwise

ii. Minority/UTC Rule: The trust is REVOCABLE, unless the trust

expressly provides otherwise (UTC 602)

k. Trust Revocation by Will: Question arises—May an inter vivos trust be

revoked by a will (especially if the inter vivos trust is used as a will substitute?)

i. Traditional Approach: No—a will cannot revoke an inter vivos trust

unless the trust expressly provides otherwise.

ii. UTC/Restatement Approach: UTC 402, R.3d Trusts 63, R.3d Wills 7.2

– expressly authorize a “subsequent will” (a.k.a. codicil) to revoke a

revocable trust, or a provision in a trust.

1. Rule: The trust may be revoked (in full or in part) by a codicil

where (1) the will expressly devises the property that

otherwise would have passed under the trust, (2) the trust

does not provide (i) for how it is to be revoked, or (ii) that the

method provided is not stated to be exclusive

e. Removal of Trustees: Traditional Approach to removing trustees is that the settlor’s intent controls. If

the settlor selected a particular trustee, that trustee cannot be removed, even if all the beneficiaries

consent, unless the trustee is unfit to serve or commits a serious breach of trust.

i. UTC Approach (UTC 706): UTC expands on Traditional Approach. (see p. 660)

1. Under UTC, a settlor, a co-trustee, or a beneficiary may request the court to remove a

trustee

2. A trustee can be removed if

a. (1) there is a material breach of trust;

b. (2) infighting among co-trustees substantially impairs the trust’s

administration;

c. (3) the trust has underperformed persistently and substantially, relative to

comparable trusts; OR

d. (4) there has been a substantial change of circumstances OR all beneficiaries

request a change of trustee, and the court finds that the removal best serves

the interests of all the beneficiaries and is not inconsistent with a material

purpose of the trust.

ii. Davis v. U.S. National Bank Association (p. 660) – not assigned!! HaHA, bitch. I ain’t readin’ that.

Chapter 10 – Trust Administration: The Fiduciary Obligation 1. Introduction: The trustee has fiduciary obligations to the beneficiary – if the trustee manages the trust property

poorly, the beneficiaries bear the loss.

a. Historically: Trusts were funded primarily w/ real property—the primary duty of the trustee was to

preserve the real property for the beneficiaries. The trustee had no inherent powers over the trust

property; 3rd

parties who dealt with the trustee had a high duty to inquire into the propriety of

transactions

b. Modern Trend: Trusts are typically funded w/ intangible assets (stocks, bonds, etc)—the primary duty of

the trustee is to manage the fund of wealth. The trustee is granted by law all the powers a reasonable

person would need to manage the trust, and 3rd

parties interested in dealing w/ the trustee have no duty

to inquire absent suspicious circumstances.

Chapter 10 – Trust Administration: The Fiduciary Obligation

106 of 120

c. Agency Theory: The risks inherent in a trust are analogous to the risks inherent in the use of an agent. It

is impossible for the parties (principal and agent; settlor and trustee) to anticipate and contract with

respect to every conceivable scenario so the relationship is governed primarily by the more flexible

fiduciary principle. But in the trust context, the beneficiaries do not have the same control over the

trustee that a principal typically has over an agent. So, traditionally, courts were stricter in their

application of fiduciary principles to trustees.

2. Trustee’s Powers:

a. Common Law: At common law, trustee has no inherent powers. Trustee possesses only those powers

either expressly granted in the terms of the trust or those necessarily implied in light of the trust

purposes.

b. Judicial Authorization: A trustee can petition a court of equity for authorization to undertake an action

not expressly or implicitly authorized under the terms of the trust. In essence, the court may grant the

trustee the requested additional power.

c. Modern Trend: The modern trend is to facilitate the granting of powers to the trustee. Two approaches:

i. Statutory List: Jurisdiction adopts a statute that sets forth a long list of powers it presumes a

trustee would need. Thus, the drafter may specify the trustee’s powers in the trust instrument

by referring to the statutory list of trustee’s powers (i.e. incorporation by reference) (see also p.

674, for UTC 815, UTC 816)

ii. Inherent Powers: Statutorily grant the trustee a broad set of basic powers, unless the settlor

expressly provides that the trustee is NOT to have one or more of the granted powers. (Typically

the statute provides that a trustee is presumed to have all the powers a reasonable person

would need to perform the acts necessarily in light of the purposes of the trust)

d. Third Parties’ Liability: A trustee’s powers to act are only effective if third parties are willing to deal w/ a

trustee. Because 3rd

parties may be held liable for participating in a breach of trust, the protection given

3rd

parties who deal w/ a trustee affects the practical scope of the trustee’s powers.

i. Common Law: Imposes virtually strict liability on 3rd

parties if the transaction constitutes a

breach of trust.

1. Policy: Generally presumed that the purpose of the trust is to preserve trust property.

If the 3rd

party knows or should know he/she is dealing w/ a trustee, the party has a

duty to inspect the trust instrument to see if the transaction is authorized and is charged

w/ proper interpretation of the trust

ii. Modern Trend: The trustee has greater powers, and 3rd

parties dealing with a trustee have

greater protection.

1. Uniform Trustee’s Powers Act: Eliminates the duty to inquire into the terms of the trust

and protects 3rd

parties unless they have actual knowledge that the transaction

constitutes a breach of trust. (UTPA 7, see p. CB 674)

2. UTC: Requires 3rd

parties to act in good faith and give valuable consideration. (UTC

1012(a)-(b), see CB p. 675)

3. The Duty of Loyalty

a. Introduction: The most fundamental principle of the fiduciary obligation in trust law is the duty of

undivided loyalty to the beneficiary. The trustee must administer the trust SOLELY in the interest of the

beneficiary.

i. Test/Application: The duty of loyalty breaks down into 2 parts:

1. Duty to act Reasonably: Objective standard—permits judicial review and supervision of

the trustee’s actions even where the trustee acted in good faith.

2. Duty to act in Good Faith: Subjective standard—addresses the trustee’s state of mind;

the trustee must have thought that what he/she was doing was in the beneficiaries’ best

interests

b. Duty Against Self-Dealing: Self-dealing arises where the trust and the trustee engage in a transaction.

The trustee has a conflict of interest—he has a personal interest in the transaction while at the same time

the trustee has a duty to act only in the best interests of the beneficiary. The duty against self-dealing is

usually construed broadly to include transactions involving other members of the trustee’s family

(spouse, children, parents).

Chapter 10 – Trust Administration: The Fiduciary Obligation

107 of 120

i. Presumption of Breach of Trust (The No Further Inquiry Rule): If a trustee engages in self-

dealing, an IRREBUTTABLE presumption of breach of the duty of loyalty arises. NO FURTHER

INQUIRY OF GOOD-FAITH OR REASONABLENESS IS NECESSARY/APPROPRIATE

ii. Hartman v. Hartle (Trustee’s Duty Against Self-Dealing) (p. 675)

iii. F/P

1. The testatrix had 5 children. Her will appointed two of her sons-in-law executors of her

estate and directed that her real property was to be sold and divided equally among the

five children. The land was sold for $3,900 at public auction, and one of the testatrix’s

sons bought it for his sister, who was the wife of one of the executors. Two months

later, the sister sold the land for $5,500.

iv. I

1. Whether a trustee may sell property to his wife without permission by an order of the

court?

v. R/A/H

1. HELD: No. A trustee’s wife may not sell property from himself at his own sale.

Nonetheless, a resale may not be ordered because the property is currently owned by

innocent purchasers. Instead, the executors hold an account for the complainant’s one-

fifth share of the profits made on the resale of the property.

2. RULE: A trustee nor his wife may purchase property from himself at his own sale unless

leave to do so has been previously obtained under an order of the court.

3. The duty against self-dealing applied to the spouse of the fiduciary. Absent court

approval of the transaction, the sale was inappropriate. The sale could not be rescinded

because of the subsequent sale to a bona fide purchaser w/o notice of the breach of

trust, but the sister was forced to share 1/5 of the profit upon resale with the

complaining beneficiary.

4. Because the executor sold the property to his wife, the beneficiaries did not receive a

fair amount of the testator’s estate.

vi. In re Gleeson’s Will (p. 676)

vii. F/P

1. In 1950, Mary Gleeson leased 160 acres of farm land to the petitioner. In 1951, Gleeson

renewed the lease with the petitioner for another year. Gleeson died just two weeks

before the lease was to expire in 1952. Gleeson devised the land to the petitioner, as

trustee, for the benefit of her three children. After Gleeson’s death, with the expiration

of the second lease imminent, the petitioner remained on the land for another year

until March 1, 1953. He increased his rent payments from $6 per acre to $10 per acre

plus a share of the crops. He leased the land to another tenant after the holdover year.

In the preceding fall of 1951, the petitioner sowed part of the 160 acres in wheat to be

harvested in 1952. The petitioner held over in an open manner.

viii. I

1. Whether the petitioner breached his duty of loyalty by holding over before the date that

the lease expired to a year later?

ix. R/A/H

1. HELD: Yes. The petitioner dealt individually with the farm land as a tenant and breached

his duty of loyalty to the trust. An exception is not made for the petitioner because he

held over shortly before the lease expired and farm land tenant are not easy to find.

Though the petitioner’s holdover did not damage the property, he is still liable for

breach. The petitioner must turn over the profits he made while he remained a tenant

at the time that he was the trustee over the farm land, to the trust.

2. RULE: A trustee may not deal in his individual capacity with the trust property.

3. The fact that the petitioner’s actions did not harm the land did not absolve him of his

duty to refrain from self-dealing.

Chapter 10 – Trust Administration: The Fiduciary Obligation

108 of 120

x. Traditional Exceptions to the Duty Against Self-Dealing: The duty against self-dealing can be

waived either by (1) the settlor in the terms of the trust or (2) by all the beneficiaries, following a

full disclosure of the proposed transaction

1. Judicial Review: Even where the self-dealing is authorized, the transaction must still be

reasonable and fair. If it is not, the trustee is liable for breaching the duty of loyalty.

(UTC 802(b))

xi. Modern Trend Exceptions to the Duty Against Self-Dealing: Many states have statutes

permitting a bank trust dept to deposit trust assets in its own banking department; institutional

trustees are increasingly authorized to combine separate trust accounts into a common trust

fund or mutual fund; trustees are authorized to charge a reasonable compensation. (UTC 802(f),

802(h); R.3d Trusts 78 cmts 4, 6, 8)

xii. Trust Pursuit Rule (Remedy): One of the remedies available to trust beneficiaries in equity for a

breach of trust is the trust pursuit rule. (see p. 679)

1. Rule: If the trustee, in wrongfully disposing of trust property, acquires other property,

the beneficiary is entitled to enforce a constructive trust on the property so acquired,

treating it as part of the trust.

2. Rule: The trust pursuit rule also applies where property ends up in the hands of a 3rd

person, UNLESS the 3rd

person is a bona fide purchaser for value and w/o notice of the

breach of trust

c. Duty to Avoid Conflicts of Interest: A conflict of interest arises where the trust deals with another party

w/ whom the trustee has an interest that may affect the trustee’s assessment of the proposed

transaction. If the transaction involves a possible conflict of interest, but not self-dealing, the “no further

inquiry” rule does NOT apply. Instead, the transaction is assessed to see if it is reasonable and fair under

the circumstances.

i. In re Rothko (Trustee’s Duty to Avoid Conflicts of Interest) (p. 679)

ii. F/P

1. Rothko was an abstract expressionist painter who had an international reputation of

greatness. He died on February 25, 1970. His will was admitted to probate on April 27,

1970. Reis, Stamos, and Morton Levine were the executors. Within a period of three

weeks, the executors hastily dealt with Rothko’s 798 paintings. By a contract of sale, the

executors agreed to sell to Marlborough A.B. (hereinafter MAG) 100 Rothko paintings

and to Marlborough Gallery, (hereinafter MNY) 700 paintings. The petitioner attempted

to remove the executors, enjoin MNY and MAG from disposing of the paintings, and to

rescind the agreements between the executors and said corporations, for a return of

the paintings still in possession of those corporations, and for damages. Reis was the

director, treasurer, and secretary of MNY, the consignee gallery. The testator had a

1969 inter vivos contract with MNY to sell Rothko’s work at commission of only 10

percent. Reis’s family had an extensive art collection through the Marlborough interests.

Stamos was an unsuccessful artist under contract with Marlborough. Marlborough

purchased a Stamos painting from a third party for $4,000 during the week in May 1970

when the estate contract negotiations were pending. Levine was aware of the

transactions.

iii. I

1. Whether executors fail to act unfairly in the transactions they entered into on behalf of

the estate?

2. Whether an executor who acting prudently on the advice of counsel may be liable for

the co-executors breach of trust?

3. Whether an executor who is liable for making an improper transfer where he had duty

to retain property but chose to sell the property, is liable for appreciation damages?

iv. R/A/H

1. HELD 1: Yes. The executors not only held an interest that conflicted with the interests of

the estate, but they acted unfairly because their interests conflicted with the interests

of the estate. Reis was induced to act in the MNY’s favor in conducting the transactions

Chapter 10 – Trust Administration: The Fiduciary Obligation

109 of 120

with the estate. As the director, secretary, and treasure of MNY, Reis was induced to

favor the interests of MNY, including his own financial aggrandizement of status and

financial advantage through sales of almost one million dollars for items from his own

collection and his family’s extensive private art collection by the Marlborough interests.

Stamos was also induced to act in favor of the Marlborough interests because he was an

artist under contract with Marlborough and the latter bought one of Stamos’s paintings

during a week when contract negotiations were pending. Stamos breached his duty not

to accept employment from a company that was conducting business with the estate.

2. HELD 2: Yes. Though Levine acted on the advice of counsel, he is liable for damages

because he failed to exert efforts directed towards prevention but acceded to the

breaches.

3. HELD 3: Yes. Because the paintings cannot be returned to the trust estate, the estate is

entitled to appreciation damages. This case involves wrongful transfers that should

make the estate whole. The damage award is not punitive.

4. Executors may be held to the same standard as trustees. Trustees may have an interest

in a transaction with the estate but they must not engage in the transaction unless they

can show that they will not be improperly influenced by those interests when dealing

with the estate.

v. Damages:

1. Rule: Where a trustee is authorized to transfer trust property, but improperly sells it for

too low a price, the trustee is liable for the difference in the actual sale price and the

price that should have been realized.

2. Appreciation Damages: Where a trustee sells property he/she was NOT authorized to

sell, appreciation damages are appropriate

a. Definition: Appreciation damages are the difference between the sale price

and the value of the property as of the date of the court’s decree (which puts

the beneficiaries back in the position they would have been in but for the

unauthorized sale.

b. Application to Rothko: In Rothko (above, p. 108), the court imposed

appreciation damages on the two executors who acted with the conflict of

interest, as well as the art gallery.

d. Co-Trustee Liability:

i. Traditional Rule: If there is more than one trustee of a private trust, the trustees must act as a

group and with unanimity, UNLESS the trust instrument provides to the contrary. One of Several

trustees does not have the power alone to transfer or deal with the property. Since co-trustees

must act jointly, a co-trustee is liable for the wrongful acts of a co-trustee to which she has

consented, or which by her negligence through inactivity or wrongful delegation, she has

enabled the co-trustee to commit.

ii. Modern Trend: The traditional rule of unanimity is on its way out. In many states, UTC 703(a)

(or other statute) allows a majority of trustees to act if there are 3 or more trustees (see also

R.3d Trusts 39). But note: Co-trustees still have a duty to take reasonable steps to prevent a

breach of trust by her co-trustees, even by bringing suit if need be.

iii. Charitable Trusts: Unanimity of action is not required of the trustees – action by a majority is

valid.

iv. Trustee’s Right to Contribution from Other Trustees: Because co-trustees are jointly liable, a

trustee generally has a right to contribution from co-trustees where he/she is found liable.

1. R.2d Trusts: The right to contribution may be limited if the trustee was either more at

fault or benefited personally from the breach; and the right to contribution is eliminated

if the trustee acted in bad faith.

e. See also IL Statutes

i. 755 ILCS 5/10-1 to 5 (“Administrators to Collect”)

ii. 755 ILCS 5/12-1 to 15 (“Bonds—Oaths—Acceptance of Office”)

iii. 755 ILCS 5/16-1 to 3 (“Recovery of Property and Discovery of Information”)

Chapter 10 – Trust Administration: The Fiduciary Obligation

110 of 120

iv. 755 ILCS 5/18-1 to 5 (“Claims against Estates”)

v. 755 ILCS 5/22-1 to 6 (“Nonresident Representative”)

vi. 755 ILCS 5/23-1 to 8 (“Resignation and Removal of Representative”)

4. The Duty of Prudence: Imposes on the trustee an objective standard of care.

a. Introduction: The trustee has a duty the trust with such skill and care as a person of ordinary prudence

would use in dealing with his/her own property. The UTC adopts this approach (see UTC 804)

b. Trust investments—overview: The issue of what constitutes an appropriate trust investment goes

directly to the question of what is the purpose of a trust. The rules limiting trust investments to “safe”

investments have given way to permitting an acceptable level or risk to ensure an adequate return on the

trust property.

i. Background: Historically, the most common approach to what constituted an appropriate trust

investment was a judicial and/or statutory list of appropriate investments. The courts and/or

legislature would identify categories of investments that were presumptively appropriate, but

even then, an investment in a particular entity or activity on the list had to be otherwise

reasonable and proper.

ii. Common Law Approach: Each investment decision is viewed separately. (if one out of 100

investments is deemed inappropriate, the trustee is liable for any loss caused by the one

inappropriate investment.

1. Settlor’s Authorization Exception: If the settlor expressly authorizes investments that

are not on the jurisdiction’s statutory list, such investments are appropriate as long as

they are otherwise reasonable and proper.

2. Judicial construction: Generally, courts tend to construe narrowly provisions

authorizing a trustee to invest in otherwise inappropriate investments – the trustee

must still act reasonably and properly.

c. Model Prudent Man Investment Act (The Old Prudent “Man” Rule): The Model Prudent Man Investment

act abolished statutory lists

i. RULE: Permits any investment that a “prudent” man would make. “Speculative” investments are

not allowed.

ii. Objective Standard: The most common statement of the prudent person std is that the trustee

should invest with the same care as a prudent man would make of his own property, taking into

consideration the dual goals of preservation of property (principal) and generation of income

iii. Criticism: The prudent man rule has been criticized for putting too much emphasis on how risky

investments are. Return on investments corresponds directly with risk. Permitting only non-

speculative investments restricts the potential return for beneficiaries. Moreover, “safe”

investment may have little to no risk of complete loss, but may subject the trust to a substantial

risk of inflation in that if inflation exceeds the rate of return, the real value of the trust property

will fall.

d. Uniform Prudent Investor Act: The UPIA, adopted in 1994, builds on the prudent person approach. The

R.3d of Trusts (§ 90) and the Trustee Act of 2000 have adopted the prudent investor standard (see CB p.

694 for UPIA)

i. Prudent Investor Standard: A trustee shall invest and manage trust assets as a prudent investor

would, by considering the purposes, terms, distribution requirements, and other circumstances

of the trust. In satisfying this standard, the trustee shall exercise reasonable care, skill, and

caution. (UPIA 2(a))

ii. Pooling Trust Funds:

1. The common law rule strictly requires each trust fund to be segregated from both the

trustee’s own funds and other trust funds.

2. The Modern Trend and Majority Rule permits pooling of trust funds to achieve

efficiencies of scale and to facilitate diversifying trust investments. The modern trend

also permits investment in mutual funds.

iii. Portfolio Approach: Trustee’s investment and management decisions are no longer assessed in

isolation, but in the context of the trust portfolio (UPIA 2(b))

Chapter 10 – Trust Administration: The Fiduciary Obligation

111 of 120

1. A key to assessing the propriety of an investment under the portfolio approach is

whether it is a compensated or uncompensated risk (Modern Portfolio Theory)

2. Compensated Risks: Compensated Risks are investments that are riskier than others,

but have a corresponding higher rate of possible return associated w/ them. The

investor is compensated appropriately for the enhanced risk. Compensated risks are

appropriate investments under the portfolio approach as long as the overall risk level of

the trust’s investment portfolio is acceptable relative to the trust purposes.

a. Example: Putting an appropriate amount of a trust’s funds into a start-up

company with great growth potential is an example of a compensated risk.

3. Uncompensated Risks: Uncompensated risks are those investments that are risky and

do not have a corresponding market-enhanced compensation to reward the investor for

taking the risk

a. Example: Putting all of one’s investments into one stock (regardless of the

level of risk associated w/ the stock) is an uncompensated risk (i.e. lack of

diversification = uncompensated risk)

4. Investment Decisions: Arguably the key considerations in assessing a trustee’s

investments under the portfolio theory approach are (1) the trustee’s investigations and

decision-making process in determining the trust’s acceptable level of compensated

risk, and (2) how that level is achieved through the combination of trust investments.

5. Duty to Delegate: The UPIA permits trustees to delegate investment and management

functions. Delegating the investment process to an expert is viewed with favor, though

the trustee still has a duty (1) to properly investigate to whom the power should be

delegated, (2) to consult with the agent to ensure that he/she properly understands the

trust’s terms, purposes, and acceptable level of compensable risk, and (3) to monitor

the activities and decisions of the investment agent.

iv. Duty to Diversify: The trustee must diversify the investments of the trust, UNLESS the trustee

reasonably determines that, because of special circumstances, the purposes of the trust are

better served without diversifying (UPIA 3)

1. A well-diversified portfolio spreads the risk of loss across all the investments so that the

aggregate level of risk is acceptable in light of the trust purposes.

e. Adequate Diversification: The question of how much diversification is necessary is not addressed in the

Act. (it is a fact-sensitive issue to be determined on a trust-by-trust basis, considering the purpose of the

trust and the particular investments in question.

i. In re Estate of Janes (Risk, Return, and Diversification in Practice) (p. 702)

ii. F/P

1. The testator died (May 1973) with a probate estate of approximately $3.5 mil, approx.

$2.5 mil of which was held in stock. 71% of that $2.5 mil (approx. $1.79 mil) consisted

of 13,232 shares of Kodak common stock. His will bequeathed most of his estate to 3

trusts. Trust 1 was a marital deduction trust for the benefit of his wife, Cynthia; Trust 2

was for the benefit of selected charities. Trust 3 was for Cynthia’s benefit during her

life, and upon her death, the principal was to pour over to the charitable trust.

2. Cynthia and Lincoln Rochester Trust Co were appoint co-executors. By August 1973, the

Trust Co’s trust and estate officers had determined the estate’s expenses and how many

shares of stock needed to be sold to cover the expenses. At that meeting the trust

officers recommended holding the remaining shares until the trusts were funded. The

memo did not otherwise discuss investment strategy. In Sept 1973, Cynthia consented

to the sale of an additional 1,200 shares of Kodak stock. At that time, the stock was

$139/share. The last time the trust officers discussed the retention of the stock or other

investment issues w/ Cynthia. Afterwards, Kodak stock declined steadily every year, to

$41/share by Mar 1978.

3. In 1980, the trust co filed its initial accounting covering most of the pd in question, and

sought judicial settlement of it. Cynthia and the atty gen objected and sought to

Chapter 10 – Trust Administration: The Fiduciary Obligation

112 of 120

surcharge the trust co for its imprudent retention of the high concentration of Kodak

stock, in violation of the prudent investor rule.

4. The surrogate court found the trust co had acted imprudently – the trust co should have

sold the stock by Aug 1973. The court imposed a $6.1 mil surcharge (including a “lost

profit” on the money which would have been reinvested if there had been proper

divestment.

5. The Appellate Division upheld the finding of imprudence and the date used to calculate

the damages, but not the inclusion of lost profits – surcharge was reduced to $4.1 mill.

6. On appeal, trust co argued there was no duty to diversify absent additional elements of

hazard, and that a list of factors indicated that no additional elements of hazard existed

in this case.

7.

iii. I

1. Whether a fiduciary’s duty of investment prudence may be limited to the opinion of

investment bankers and analysts who follow the company’s stock, and an overall

determination of the investment quality determined by (1)the capital structure of the

company, (2) the competency of its management, (3) whether the company is a

seasoned issuer of stock with a history of profitability, (4) whether the company has a

history of paying dividends, (5) whether the company is an industry leader, and (6) the

expected future direction of the company’s business?

2. Whether, under all of the facts and circumstances of this case, the fiduciary violated the

prudent person standard in maintaining a concentration of the Kodak stock?

3. Whether August 9, 1973 was a reasonable time by which the petitioner should have

divested the estate of the stock?

4. Whether the proper measure of damages for breach of a duty to act prudently is “lost

profits” or the amount that the proceeds of the stock would have yielded, up to the

time of trial, had they been invested in petitioner’s own diversity equity fund on August

9, 1973.

iv. R/A/H

1. HELD 1: No. A fiduciary’s duty of investment prudence may not be limited to the

opinion of investment bankers and analysts who follow the company’s stock and other

factors offered by the petitioner. The prudent person rule dictates against any absolute

rule that immunizes a fiduciary from its failure to diversify based upon such factors.

Also, omits other factors to be considered under the prudent investor rule like, the

amount of the trust estate, the situation of the beneficiaries, the trend of prices and of

the cost of living, and the prospect of inflation and deflation.

2. HELD 2: Yes. The petitioner here acted imprudently by failing to divest the estate of the

Kodak stock by August 9, 1973 because the petitioner jeopardize the interests of the

primary income beneficiary. The Kodak stock dropped in value, decreasing the amount

of income that would be available to the primary income beneficiary, the testator’s 72

year old widow for whom the support testamentary trusts were created. Furthermore,

the petitioner failed initially to undertake a formal analysis of the estate and establish

an investment plan consistent with the testator’s primary objectives; (2) failed to follow

petitioner’s own internal trustee review protocol during the administration of the

estate, which advised special caution and attention in cases of portfolio concentration

of as little as 2%; and (3) failed to conduct more than routine reviews of the Kodak

holding in this estate, without considering alternative investment choices, over a seven-

year period of steady decline in the value of the stock.

3. HELD 3: Yes. August 9, 1973 was a reasonable time by which the petitioner should have

divested the estate of the stock The petitioner’s internal documents and

correspondence, as well as the testimony of Patterson, Young and objectants’ experts

establish that the petitioner had all the information a prudent investor would have

needed to conclude that the percentage of Kodak stock in the estate’s stock portfolio

Chapter 10 – Trust Administration: The Fiduciary Obligation

113 of 120

was excessive, and should have been significantly reduced in light of the estate’s over-all

investment portfolio and the financial requirements of Janes and the other charitable

beneficiaries.

4. HELD 4: No. The proper measure of damages is the value of the capital that was lost,

the difference between the value of the stock at the time it should have been sold and

its value when ultimately sold. In this case which involves faithless transfers as oppose

to deliberate self-dealing, the proper measure of damages is not “lost profits.”

5. RULE: A trustee must diversity assets unless the trustee reasonably determines that it is

in the interests of the beneficiaries not to diversify, taking into account the purposes

and terms and provision of the governing instrument. In imposing liability upon a

fiduciary on the basis of the capital lost, the court should determine the value of the

stock on the date it should have been sold, and subtract from that figure the proceeds

from the sale of the stock, or, if the stock is still retained by the estate, the value of the

stock at the time of the accounting. The court has discretion on whether interest should

be awarded. Dividends and other income attributable to the retained assets should

offset any interest awarded.

6. No precise formula exists for determining the prudent person standard—each case

turns on its own facts and circumstances. The trustee’s investment decisions are to be

measured in light of the business and economic circumstances at the time they were

made

7. The high concentration of Kodak stock (with the other shares primarily in other stocks)

failed to take into adequate consideration the needs of the testator’s 72 yr-old widow.

8. The trust co failed to exercise the due care and skill of a corporate fiduciary by (1) failing

to establish an investment plan upon funding; (2) failing to follow its own internal

policies of special caution and attention to cases of portfolio concentration exceeding

20%; and (3) failing to conduct more than routine reviews of the account in the face of

declining values.

v. Duty of Care – Professional/Corporate Trustees vs Individual Trustees: Professional and

corporate trustees are usually held to a higher standard of care in investing due to their

presumed expertise. Individual trustees are usually held to a lower standard.

vi. Exceptions to Duty to Diversify:

1. It may be prudent not to diversify or to delay diversification when the tax or other costs

of reorganizing the portfolio are likely to outweigh the benefits of diversification (UPIA

3, cmt).

2. Or if a trust holds a family business, esp if the business is closely held and not readily

marketable

3. Or if the trust is just one component of a larger scheme such that the beneficiary’s

financial interests are diversified overall

4. Investment in a mutual fund may constitute adequate diversification if the fund is

diversified.

vii. Inception Assets: Many jurisdictions permit a trustee to have a preference for retaining the

trust’s “inception assets”—the assets used to fund the trust that the settlor recommends the

trustee retain.

1. Requirement vs Authorization: Even if a trust authorizes the trustee to retain inception

assets irrespective of diversification, courts may still find the trustee violated fiduciary

duty (see First Ala. Bank of Huntsville, N.A. v. Spragins (CB p. 717)). (see also Emanuel

p. 267)

2. The right is NOT absolute: It is subject to the trustee’s more general fiduciary duty of

prudent administration

a. Power vs Duty: Even if the trustee has the POWER to retain certain assets, the

exercise of the power must accord w/ the trustee’s duties of prudence and

loyalty (R.3d Trusts 91)

Chapter 10 – Trust Administration: The Fiduciary Obligation

114 of 120

viii. Authorization to Retain vs Duty to Sell: Even where the trust instrument authorizes the trustee

to retain the trust assets in question, where failure to diversify is inconsistent with the modern

portfolio approach, the trustee has a duty to sell the trust property in a timely manner (w/in a

reasonable time period)

ix. Direction to Retain vs Duty to Sell: Where the trust instrument directs the trustee to retain the

trust assets in question, the issue is more complicated.

1. General Rule: Settlor’s intent controls – some courts have ruled that the trustee must

comply w/ the retention order

2. Modern Portfolio Approach: Some courts have approved diversification if there are

“changed circumstances,” i.e. trustee doesn’t have a duty to follow the settlor’s

directions

x. Calculating Damages: Where there is a breach of trust, the trust beneficiaries are entitled to be

made whole. Three ways to do this: (1) Charge the trustee with any resulting loss; (2) charge the

trustee with any profit made; or (3) charge the trustee with any profit that would have accrued

but for the breach. (See UTC 1002) – see CB 709

1. The Make-Whole Approach: The “Total return/make whole” damages approach holds

the trustee liable for any losses incurred and gains forgone as a result of the breach.

The trustee is also liable for any profit made by the trustee through the breach of trust.

(UTC 1002).

a. Restatement Approach: Where there are several plausible investment

strategies, the R.3d Trusts favors application of the most favorable unless the

trustee can justify why it should not apply

2. The Capital Lost plus Interest Approach:

a. This approach takes the value of the trust’s stock on the date when the trustee

should have divested, plus compound interest through the [another date],

minus the actual value of the trust.

b. This approach does not punish the trustee for lost investment opportunities,

awarding interest to the trust beneficiaries rather than possible profits from

prudent investment of the lost capital. Under this approach, the rate of

interest is critical. Different possible rates include the historic average rate of

inflation; the annual return on long-term government bonds; and the legal rate

applied to money judgments.

3. The Total Return Approach: The “total return” approach calculates damages by

awarding the difference between how the particular, imprudently managed, portfolio

actually performed vs how a hypothetical matching portfolio, prudently managed,

would have performed (taking into account taxkes, expenses, and distributions).

a. This approach is fact-intensive; requires expert testimony; is somewhat

speculative.

xi. Social Investing: “Social Investing” is choosing to invest (or not invest) based on social issues

(e.g. choosing not to invest in a tobacco manufacturing company’s stock

1. RULE: UPIA §5 says trustees should ALWAYS invest in a manner that best serves the

trust beneficiaries – i.e. “no form of ’social investing’ is consistent with the duty of

loyalty if the investment activity entails sacrificing the interests of trust beneficiaries in

favor of the interests of the persons supposedly benefitted by pursuing the particular

social cause.”

f. Delegation: At common law, a trustee was not allowed to delegate matters that the trustee could

reasonably be required to perform (R.2d Trusts 171).

i. Rationale for Common Law: Settlor put great trust in the trustee and assumed that the trustee

would personally hold/manage the property. So, delegation violated the settlor’s intent

ii. Exception—Ministerial Responsibilities: Even at common law, there was an exception for

ministerial responsibilities—those activities that do not require the exercise of discretion (e.g.

cutting grass, making repairs, etc.)

Chapter 10 – Trust Administration: The Fiduciary Obligation

115 of 120

iii. Modern Trend (Duty to Delegate): Recognize that some trustees are unqualified to undertake

certain responsibilities in holding/managing trust property, esp the duty to invest trust property

properly.

1. Rules: UPIA §9, R.3d Trusts §171, and UTC §807 all agree – trustee may have a duty to

delegate responsibilities if a prudent person would delegate under similar circumstances

(see CB p. 720). Trustee must act in the best interests of the beneficiaries in deciding

whether to delegate discretionary responsibilities, including investment-making

responsibilities, and to whom to delegate them.

2. Duty to Supervise: The trustee has an ongoing duty of care in

a. (1) selecting agents to whom to delegate;

b. (2) defining the agent’s role and giving proper instructions; and

c. (3) monitoring/supervising the actions of the agent(s) to whom the trustee

delegates the responsibilities to ensure that the agent(s) act(s) w/in the

delegated authority and in the best interest of the beneficiaries

d. The trustee cannot abdicate or delegate unreasonably

e. (UPIA §9, See CB p. 722 for longer-winded versions of this :-D)

iv. Directed vs Delegated Trusts:

1. Delegated Trust: The TRUSTEE decides which tasks should be delegated and to whom

they should be delegated. The trustee is subject to the UPIA duties inherent in

selecting, instructing, and supervising the agents

2. Directed Trust: The SETTLOR directs the trustee (through the terms of the trust) to

follow the instructions of others. i.e. the settlor selects the agents to whom certain

tasks are to be delegated, and the trustee must follow the 3rd

parties’ instructions.

3. Rules:

a. R.3d Trusts §75 – trustee has a DUTY to follow a direction of a person if the

trust so provides, UNLESS the direction is “contrary to the terms of the trust”…

or the “trustee knows or has reason to believe that the direction violates a

fiduciary duty that the power holder owes to the beneficiaries.”

b. UTC §808 – The trustee must follow a direction unless it is “manifestly”

contrary to the terms of the trust or would constitutes a “serious” breach of

fiduciary duty.

c. Delaware Code Title 12 §3313: The one who directs (called a “trust adviser”) is

presumptively a fiduciary, but the governing instrument can provide otherwise,

and the trustee has no duty to monitor or to notify the beneficiary if the truste

has concerns about direction

i. i.e. The trustee is protected from liability except in cases of the

trustee’s own willful misconduct

5. Impartiality – Allocating Principal And Income

a. Introduction: Perhaps the most important sub-rule in trust fiduciary law is the duty of impartiality. This

duty is implicated when a trust has two or more beneficiaries.

b. Rule: In investing, managing, and distributing the trust property, the trustee must strike a balance

between the beneficiaries, giving due regard to their respective interests (e.g. strike a balance between

investing the property to produce reasonable income, while preserving the principal for the

remaindermen) (UTC §803, R.3d Trusts §79)

i. Policy: Different beneficiaries have different interests (e.g. present interests – life estate income

interests vs future interests – remainder in the principal). Because the beneficiaries have

different property interests, their personal interests often conflict.

ii. Note: “Impartiality” is not truly impartial – the trustee must take into account any preferences

that the settlor may have expressed in the governing instrument (or some other manner), and

must also consider the various/conflicting beneficiaries’ interests.

iii. Note—we skipped the case in the reading (Howard v. Howard).

c. Accounting for Principal and Income

i. 1962 Principal and Income Act:

Chapter 10 – Trust Administration: The Fiduciary Obligation

116 of 120

1. Income: Money generated on a regular (or irregular) basis as a result of the trust

property or trust investments constitutes income (eg interest, rent, cash dividends on

stock

2. Principal: Money generated as part of a conveyance (voluntary or involuntary) of trust

property is considered principal (eg sales proceeds, insurance proceeds)

a. In Addition: Stock splits and stock dividends are considered principal because

such property has to be retained as principal to maintain the trust’s percentage

interest in the company. Bond principal payments and part of royalty

payments are also considered principal

ii. 1997 Principle and Income Act – the power of equitable adjustment: MAJORITY/MODERN

Approach – Focuses on the total return to the trust portfolio. As long as the trust achieves an

acceptable rate of return on its investments, it irrelevant whether that return came from income

or principal as traditionally defined.

1. Trustee’s Power: Trustee has power & discretion, the power of equitable adjustment,

to reallocate the total return between the income and principal beneficiaries to ensure

that the two groups are treated fairly while paying particular attention to the larger rate

of return regardless s of how the return is classified

2. Settlor’s Intent: The settlor may expressly provide that the trustee does not have

power to reallocate principal and income.

iii. Unitrust: Under a unitrust, the life (income) beneficiaries are entitled to a specified percentage

of the value of the trust principal each year, so there is no need to distinguish income from

principal.

1. Rule: All property generated by the trust is assigned to principal, and at the appropriate

intervals, the specified percentage of the trust principal is distributed to the appropriate

beneficiaries

2. Purpose of Unitrust: Permit the trustee to focus on investing the trust portfolio to

maximize total return, as opposed to worrying about investing to ensure an appropriate

income stream for the income beneficiary.

iv. Unitrust Election (Adjustment Powers)

1. Introduction: Many states have statutes authorizing a trustee to convert a traditional

trust into a unitrust (adjustment powers).

2. Trustee-beneficiary: Some states deny this power to a trustee-beneficiary; others

permit it but scrutinize the trustee’s actions more closely (See UPIA §104(c)(7))

3. Settlor’s Intent: This statutory adjustment power is a default rule—it only is operational

if the settlor does not provide otherwise (see also R.3d Trusts §113)

4. In re Matter of Heller (Unitrust) (p. 731)

5. F/P

a. Jacob Heller created a testamentary trust for the benefit of his wife Bertha and

his children from a previous marriage. Bertha was to receive income from the

trust each year, and after her death, the principal was to be distributed to his

children. Jacob’s two daughters were to receive a 30% share each. His two

sons, who were also trustees, were to get 20% each (they were trustee-

beneficiaries).

b. Following Jacob’s death, the income to Bertha averaged $190,000/yr for

several years. In 2001, to facilitate overall portfolio performance, NY enacted

legislation that created an optional unitrust provision. It allows a trustee to

calculate the income to be distributed according to a fixed formula and based

on the net fair market value of the trust assets.

c. In 2003, the brothers elected to apply this option retroactively, reducing

Bertha’s annual income to $70,000. Jacob’s sons were then able to make

investments, although they produced low dividend yields, would outperform

the alternative in the long-term, creating a better overall return.

6. I

Chapter 10 – Trust Administration: The Fiduciary Obligation

117 of 120

a. Does the trustee-beneficiary’s election of unitrust violate fiduciary duties to

beneficiaries?

7. R/A/H

a. HELD: No

b. RULE: When an interested trustee elects a unitrust, it does not per se violate

fiduciary duties.

c. Though the brothers were beneficiaries and trustees, they could elect unitrust

treatment.

d. NOTE: The court ruled that the lower courts should scrutinize the unitrust

election with special care in such cases to ensure that the election is not a

violation of the trustee’s fiduciary duties to any of the beneficiaries.

6. Sub-Duties Relating to Care of Trust Property

a. Introduction: At the macro level, a trustee has a duty to care for the property as a prudent person would

care for the property of another.

i. Duty to Collect and Protect: A trustee has a duty to collect and protect trust property w/o

unnecessary delay (UTC §809; R.3d Trusts §76(2)(b))

1. How long of a delay is reasonable: W/ testamentary trusts, trustee should collect the

assets from the executor as soon as circumstances permit

2. Duty to Examine Property: W/ testamentary trusts, trustee owes a duty to the

beneficiaries to examine the property tendered by the executor to make sure it is what

the trustee ought to receive

ii. Duty to Care For and Maintain: Trustee should treat the property as an ordinary owner would

treat similarly situated property (e.g. insure the property, keep the property in good repair, etc)

iii. Duty to Earmark: Where the trust property is personal property (esp fungible assets, such as

money, stocks, etc.), the trustee has a duty to separate the trust property from all other assets

and to properly designate the property as trust assets. This ensures that a trustee cannot

“switch” trust assets and personal assets after the fact where the trust assets outperform the

personal assets (UTC 810)

1. “Fungible” means – a description applied to items of which each unit is identical to

every other unit, such as in the case of grain, oil, or flour.

2. Fungible goods are those that can readily be estimated and replaced according to

weight, measure, and amount.

3. Exception—Assets Not Subject To Registration: Assets not subject to registration, such

as bearer bonds, fall within an established exception to the earmarking requirement. In

such cases, the trustee must keep records showing that the property belongs to the

trust and should be kept separate from the trustee’s own property (R.3d Trusts 84, cmt

d)

4. Common law—strict liability: At common law, if a trustee breached the duty to

earmark, the trustee was strictly liable for any damage the trust property sustained

(even if the damage is not caused by the breach).

5. Modern Trend—Causation: The modern trend is that a trustee is not liable for a breach

unless the breach of the duty causes the damage to the trust property.

iv. Duty not to Commingle Trust Funds with the Trustee’s Own:

1. RULE: A trustee is guilty of breach of trust if the trustee commingles the trust funds

with his own, even if he does not use the trust funds for his own purposes. (UTC 810(b);

R.3d Trusts 84, cmt b)

2. Policy: Commingled trust funds become difficult to trace, and are subject to the risk

that personal creditors of the trustee can reach them (i.e. the trustee would be able to

improperly pay off debts to his own creditors using trust property. This is a no-no). This

defeats the purpose of trust law, which is that trustee’s creditors generally cannot reach

the trust property.

3. Modern Trend: Keeps the general commingling rule, but permits commingling with

other trust funds (but not the trustee’s own funds) to achieve economies of scale and to

Chapter 10 – Trust Administration: The Fiduciary Obligation

118 of 120

improve the efficiency of trust administration. The trustee must maintain records clearly

indicating the respective interests. (UTC 810(d), see p. 737)

4. Standard of Liability:

a. Common Law: Strict liability if the trustee breaches the duty not to commingle

b. Modern Trend: Trust beneficiaries must prove the breach caused the damage

to the trust property

7. Duty to Inform and Account

a. Duty to Inform/Disclose: The trustee has a duty to inform the beneficiaries of the existence of the trust

and significant developments related to trust administration, and to respond promptly to requests by

beneficiaries for information reasonably related to the beneficiaries’ interests in the trust. (UTC 813) (See

CB p. 738)

i. Settlor Authorizes Withholding Information: The law is not clear about what should happen

when the settlor expressly provides in the trust that the terms of the trust or information about

the trust property are to be withheld from the beneficiary.

1. UTC: Default rule is that the trustee must promptly provide a copy of the trust

instrument to the beneficiary if the beneficiary requests a copy, UNLESS the settlor

provides otherwise (UTC 813(b)(1))

2. California—Right to Receive: Cali provides that upon the death of a settlor of a

revocable trust, all beneficiaries AND HEIRS of the settlor have the right to request a

complete copy of the trust instrument.

ii. Fletcher v. Fletcher (Trustee’s Duty to Disclose/Inform) (p. 739)

iii. F/P

1. Fletcher executed a revocable inter vivos Trust Agreement. The agreement contained

specific provisions for the establishment of a number of trusts upon the grantor’s death,

including three separate trusts for the respective benefit of certain children. The three

separate trusts were to be in the amount of $50,000 each. The trustees were authorized

in their discretion to expend of the benefit of James N. Fletcher, Jr., an adult child of the

grantor, such amounts of the net income an principal, of the $50,000 trust as may be

necessary to provide him adequate medical insurance and medical care during his

lifetime, or until such time as the trust is depleted. They were also authorized to

expend, in their discretion, for the benefit of Fletcher’s children such amounts of the

income and principal for the benefit of Fletcher’s children.

2. The plaintiff alleged that one of the trust instruments recites that the Grantor

transferred, assigned, and set over certain cash and securities which were described in a

schedule entitled “A” attached to the trust agreement. The plaintiff also alleged that on

his m other’s death, he was advised that the trust assets had been transferred to a new

trust with the defendants as trustees. The plaintiff requested the details of both trusts

from the trustees but the trustees refused to comply with his request. The plaintiff also

alleged that the trustee, Henry L. Fletcher, justified his failure to disclose the requested

information by stating that his mother requested that the trust terms and dealings be

kept confidential, even from the beneficiaries.

3. The trial court issued an order that the plaintiff had an absolute right to complete copies

of the Trust Agreement and all amendments referred to in the pleadings and associated

documents. The court ordered the Trustees to provide the plaintiff with full and

complete copes of the trust instruments that are referred to in the Bill of Complaint

iv. I

1. Whether the trustees had a duty to disclose the terms of the trust agreement to the

beneficiaries?

v. R/A/H

1. HELD: Yes. The trustees had a duty to disclose the terms of the trust agreement to the

beneficiaries, even though the terms of the trust itself did not require such a disclosure.

2. RULE: The terms of a trust may regulate the amount of information which the trustee

must give and the frequency with which it must be given, but the beneficiary is always

Chapter 10 – Trust Administration: The Fiduciary Obligation

119 of 120

entitled to such information as its reasonably necessary to enable him to enforce his

rights under the trust or to prevent or redress a breach of trust.

3. Without access to the agreement, the beneficiaries were not able to assure the Trustees

were discharging their duty to deal impartially with all the beneficiaries in regards to the

trust agreement, in regards to their duty to use reasonable care and skill to make the

trust property productive, or in regard to whether the trustees’ investment decisions

made with respect to the assets revealed on Schedule “A.” Though the claim that the

grantor did not want the beneficiaries to see the trust agreements in full was

unsubstantiated, the Grantor could not prevent the beneficiaries from having access to

the trust agreements.

4. Though the trustee does not have a duty in statute to turn over the trust documents,

the beneficiaries did have a right to see the trust documents.

b. Duty to Account:

i. Introduction: The law protects a trustee from liability to the beneficiary if (1) the facts

underlying the beneficiary’s claim are fairly disclosed in an accounting filed with the court, (2)

notice of the accounting is properly served on the beneficiary, and (3) the beneficiary does not

timely object to the accounting.

1. Problems arise when the facts underlying a subsequent claim by the beneficiary may

not have been fairly disclosed in the accounting.

ii. Testamentary Trusts: Trustees have a duty to account to the probate court so that the court can

assess the trustee’s performance.

1. “No Judicial Accounting” Clauses (Settlor’s Intent): Some courts permit a provision in

the trust releasing a trusting from his/her duty to account to the probate court (because

doing so is slow and expensive) as long as the trustee accounts directly to the

beneficiaries (typically the income beneficiaries).

a. BUT: some courts hold that such “no judicial accounting” clauses violate public

policy because they fail to adequately protect the interests of the

remaindermen.

iii. Inter Vivos Trusts: Inter vivos trusts are not created as part of the probate process, so they are

not naturally subject to probate court supervision, though judicial accounting is still possible.

1. “No Judicial Accounting” Clauses: Usually held valid with inter vivos trusts, because the

trusts don’t pass through probate.

iv. Uniform Trust Code: UTC drops the reference to “accounting” – calls it “reporting” instead.

1. Settlor may waive the duty to report to the beneficiaries; (UTC 813(c))

2. Beneficiaries may likewise waive their right to receive a report or other information

(UTC 813(d))

3. A beneficiary may not ex ante and irrevocably waive rights to all reports and information

a. The term “ex ante” is a Latin word which means based on assumption and

prediction. It also means beforehand or before the event

b. This means that beneficiaries cannot pre-emptively waive ALL rights to receive

reports—only on a one-off basis

c. Also, waiver does not relieve a trustee from liability for misconduct that a

report would have disclosed.

v. Beneficiaries’ Duty to Review Accounting: When a trustee makes an accounting (to court or

directly to beneficiaries), the beneficiaries have a duty to check the accounting and to object in a

timely manner. If the beneficiaries fail to object timely, they may be barred from complaining

later.

vi. Fraudulent Accounting: When a trustee files a fraudulent accounting, and the beneficiaries later

discover the fraud, the beneficiaries MAY re-open the account

vii. Constructive Fraud: Where an accounting makes factual representations that turn out to be

false, if the trustee made the representations without undertaking reasonable efforts to

ascertain the accuracy of the factual representations, the representations constitute

Chapter 10 – Trust Administration: The Fiduciary Obligation

120 of 120

“constructive” or “technical” fraud. Such fraud is grounds for reopening an otherwise properly

allowed accounting

1. Investigation: The doctrine of technical fraud does not make trustees guarantors of all

factual representations in an accounting. If the representations were made in good

faith and follow reasonable efforts to ascertain the accuracy of the representations, the

trustee has fulfilled his/her duty.

2. Constructive Fraud Only Applies Factual Representations: Not to statements of

judgment or discretion

3. Discoverability: Constructive Fraud does not apply if the factual falsehood is

discoverable from an inspection of all the trust accounts, the trust terms, and the law.

viii. Improper Payments: Where an accounting reveals that a trustee has improperly distributed

trust property to one who is NOT entitled to receive such property, the trustee is liable fro

breach of trust UNLESS the court approves the accounting. Where the court’s approval is based

on fraudulent accounting, reopening such accounting voids the court’s approval of the

accounting.

ix. Example: National Academy of Sciences v. Cambridge Trust Co. (Not assigned)

8. See also IL Statutes

a. 755 ILCS 5/14-1 to 3 (“Inventory and Appraisal”)

b. 755 ILCS 5/19-1 to 14 (“Administration of Personal Estate”)

c. 755 ILCS 5/20-1 to 24 (“Administration of Real Estate”)

d. 755 ILCS 5/21-1 to 2 (“Investments by Representative”)

e. 755 ILCS 5/24-1 to 22 (“Accounts”)